Sei sulla pagina 1di 131

1/1

The Official SAT Online Course


Help | Profile | My Organizer | My Bookmarks | Logout

Answers and Explanations

Back to Score Report

Test Sections

Section 1

Essay

Section 2

Online - Practice Test #2

Section 3

Section 4

Section 6

Section 7

These sample essays were originally handwritten by students but are shown typed here for ease
of reading. The essays are displayed exactly as students wrote them, without any corrections to
spelling, punctuation, or syntax. One handwritten sample essay is provided to illustrate the need
for legible and clear handwriting.

Section 8

Section 9

Section 10

Exemplars:

Essay Prompt

Think carefully about the issue presented in the following excerpt and the assignment below.

ed

For a variety of reasons, people often make choices that have negative results. Later, they
regret these choices, finding out too late that bad choices can be costly. On the other hand,
decisions that seem completely reasonable when they are made may also be the cause of
later disappointment and suffering. What looks like a wonderful idea at one time can later
seem like the worst decision that could have been made. Good choices, too, can be costly.

is
te
r

Are bad choices and good choices equally likely to have negative consequences? Plan and
write an essay in which you develop your point of view on this issue. Support your position
with reasoning and examples taken from your reading, studies, experience, or observations.

nR
eg

Copyright 2006 The College Board. All rights reserved.

Back to Score Report

Privacy Policy

Terms of Use

Contact Us

file://E:\\b2.htm

2006-11-12

1/17

The Official SAT Online Course

Help | Profile | My Organizer | My Bookmarks | Logout

Answers and Explanations

Back to Score Report

Test Sections

Section 1

View Answers and Explanations

Section 2

Online - Practice Test #2

Section 3

Section 4

what is the value of

If

and

when

Section 6

Section 7

(A)

Section 8

(B)

Section 9

(C)

Section 10

(D)
(E)

AND EXPLANATIONS

ed

ANSWERS

Explanation for Correct Answer A :


Choice (A) is correct. Substituting the given values for

results in the equation

into the

which simplifies to

er

equation

and

is
t

or

Explanation for Incorrect Answer B :


Choice (B) is not correct. If

eg
or

nR

and

the equation reads

which is a false statement.

or

Explanation for Incorrect Answer D :


Choice (D) is not correct. If

and

the equation reads

which is a false statement.

or

Explanation for Incorrect Answer E :


Choice (E) is not correct. If

and

the equation reads

which is a false statement.

or

the equation reads

which is a false statement.

Explanation for Incorrect Answer C :


Choice (C) is not correct. If

and

then

If
(A)
(B)
(C)
(D)
(E)

ANSWERS

file://E:\\b3.htm

AND EXPLANATIONS
2006-11-12

2/17

The Official SAT Online Course

Explanation for Correct Answer C :


Choice (C) is correct. Since
Therefore,

it follows that

Explanation for Incorrect Answer A :


Choice (A) is not correct. If

equals

and

then

Explanation for Incorrect Answer B :


Choice (B) is not correct. If

and

then

Explanation for Incorrect Answer D :


Choice (D) is not correct. If

then

Explanation for Incorrect Answer E :


Choice (E) is not correct. If

then

and

and

re

is
te

On the number line above, the tick marks are equally spaced. What is the value of

nR
eg

(A)

(B)

(C)

(D)

(E)

ANSWERS

AND EXPLANATIONS

Explanation for Correct Answer E :


Choice (E) is correct. Since the tick marks divide the segment between
and into four equal parts, the distance between any two adjacent tick

marks must be

and the value of

Therefore

corresponds to

corresponds to

is

Explanation for Incorrect Answer A :

Choice (A) is not correct.

is the value of

The question asks for the

value of

Explanation for Incorrect Answer B :


Choice (B) is not correct. The tick marks represent quarter units on the

file://E:\\b3.htm

2006-11-12

3/17

The Official SAT Online Course

must be a whole number multiple of

number line, so

Explanation for Incorrect Answer C :

but the question asks for the

is the value of

Choice (C) is not correct.

and

difference between

Explanation for Incorrect Answer D :


Choice (D) is not correct. The tick marks represent quarter units on the

must be a whole number multiple of

number line, so

Questions 4-5 refer to the following graph.

re

te

is

that was

Which of the following regions in Africa had an elephant population in

of its elephant population in

approximately

eg

I.

Central

II.

Eastern

nR

Southern

III.

(A)

I only

(B)

II only

(C) III only

(D) I and III only

(E)

I, II, and III

ANSWERS

AND EXPLANATIONS

Explanation for Correct Answer B :


Choice (B) is correct. By looking at the graph, you can see that in region II

was approximately

(Eastern Africa) the elephant population in

of

In each of the regions I and III (Central


the elephant population in
Africa and Southern Africa, respectively), the elephant population in

was greater than

of the elephant population in

region II was the elephant population in

file://E:\\b3.htm

Therefore, only in

approximately

of that in

2006-11-12

4/17

The Official SAT Online Course

Explanation for Incorrect Answer A :


Choice (A) is not correct. In Central Africa, the elephant population in

of the elephant population in

was greater than

Explanation for Incorrect Answer C :


Choice (C) is not correct. In Southern Africa, the elephant population in

of the elephant population in

was greater than

Explanation for Incorrect Answer D :


Choice (D) is not correct. In each of the regions Central Africa and Southern

of the elephant

was greater than

Africa, the elephant population in

population in

Explanation for Incorrect Answer E :


Choice (E) is not correct. In Eastern Africa, the elephant population in

of the elephant population in

was approximately

but in each of the

regions Central Africa and Southern Africa, the elephant population in

of the elephant population in

was greater than

re

the total elephant population in the four regions of Africa


to
From
decreased by approximately what percent?

is
te

(A)

(B)
(C)

(E)

eg

(D)

AND EXPLANATIONS

nR

ANSWERS

Explanation for Correct Answer C :


the total elephant population was
Choice (C) is correct. In
In
approximately
the total elephant population was approximately
Since the population
fell by about half, the population decrease was approximately

Explanation for Incorrect Answer A :


was
Choice (A) is not correct. The total elephant population in
If the total elephant population in the four regions
approximately
then the
to
from
had decreased by approximately
would have been approximately
population in

Explanation for Incorrect Answer B :


was
Choice (B) is not correct. The total elephant population in
If the total elephant population in the four regions
approximately
then the
to
from
had decreased by approximately
would have been approximately
population in

Explanation for Incorrect Answer D :


was
Choice (D) is not correct. The total elephant population in
If the total elephant population in the four regions
approximately
then the
to
from
had decreased by approximately
would have been approximately
population in

file://E:\\b3.htm

2006-11-12

5/17

The Official SAT Online Course

Explanation for Incorrect Answer E :


was
Choice (E) is not correct. The total elephant population in
If the total elephant population in the four regions
approximately
then the
to
from
had decreased by approximately
would have been approximately
population in

the function
For all numbers
following has a negative value?

Which of the

is defined by

(A)
(B)
(C)
(D)
(E)

ANSWERS

AND EXPLANATIONS

is

at

re

Explanation for Correct Answer C :


Choice (C) is correct. The value of the function

is
te

Explanation for Incorrect Answer A :


Choice (A) is not correct. The value of the function

at

is

This is not a negative value.

Explanation for Incorrect Answer B :


Choice (B) is not correct. The value of the function

eg

nR

at

is

This is not a negative value.

Explanation for Incorrect Answer E :


Choice (E) is not correct. The value of the function

is

This is not a negative value.

Explanation for Incorrect Answer D :


Choice (D) is not correct. The value of the function

at

at

is

This is not a negative value.

In the figure above,

If

divides square

into two rectangles, and

bisects

what is the area of

(A)
(B)
(C)
(D)

file://E:\\b3.htm

2006-11-12

6/17

The Official SAT Online Course

(E)

ANSWERS

AND EXPLANATIONS

Explanation for Correct Answer B :


Choice (B) is correct. The area of a triangle is given by the formula

is

Therefore, the area of

is

of

Explanation for Incorrect Answer A :


of
Choice (A) is not correct. The base

re

Explanation for Incorrect Answer C :


of
Choice (C) is not correct. The base

If the area of

is
te

so its length is

eg

If the area of

bisects

nR

the side length of square

so its length is

to the side length of square

While driving on a
hours. In terms of

is one side of square

were

its height would be

must also be equal to

which is

If the area of

bisects

However,

must also be equal

so the height of

Explanation for Incorrect Answer E :


of
Choice (E) is not correct. The base

its height would be

which is

Explanation for Incorrect Answer D :


of
Choice (D) is not correct. The base

However,

were

to the side length of square

so its length is

is one side of square

so the height of

bisects

However,

which is

to the side length of square

its height would be

must also be equal

so the height of

bisects

However,

is one side of square

were

If the area of

so its length is

and the height

it follows that

bisects

is a

into two rectangles, it follows

divides

Since

Since

that

because

is

case, the length of the base of

square of side

is the height. In this

is the length of the base and

where

so the height of

is one side of square

were

its height would be

must also be equal

which is

miles per hour for the first


-mile trip, Mr. Smith averages
how many miles remain to be traveled?
where

(A)

(B)

(C)
(D)

(E)

ANSWERS

AND EXPLANATIONS

Explanation for Correct Answer B :


Choice (B) is correct. Since Mr. Smith drives at an average speed of
miles. Thus,
hours, he will have traveled
miles per hour for the first

file://E:\\b3.htm

2006-11-12

7/17

The Official SAT Online Course

miles remaining to be traveled.

there are

Explanation for Incorrect Answer A :


Choice (A) is not correct. The number of miles driven,
subtracted from the total length of the trip,

must be

Explanation for Incorrect Answer C :


miles, not
Choice (C) is not correct. The total length of the trip is
miles, not
hours, Mr. Smith has driven
miles. Also, after
miles.

Explanation for Incorrect Answer D :


hours, Mr. Smith has driven
Choice (D) is not correct. After

miles, remain to be

miles, not

miles. Therefore,

miles, not

traveled.

Explanation for Incorrect Answer E :


Choice (E) is not correct. This expression represents the ratio

The question asks for number of miles

re

remaining, not the amount to be traveled relative to the distance traveled so


far.

is
te

eg

In the figure above, the average (arithmetic mean) of the numbers in each column is
If the three circled numbers are moved from the left to the right column, which of
the following combinations of numbers can then be moved from the right to the left
remains the average of the numbers in each column?
column so that

nR

(A)

(B)

(C)

(D)
(E)

ANSWERS

AND EXPLANATIONS

Explanation for Correct Answer E :


numbers, and the average of each
Choice (E) is correct. Each column has
so the sum of the numbers in the left column is equal to the
column is
circled numbers,
sum of the numbers in the right column. When the
are moved from the left to the right column, a list of
whose sum is
needs to be moved from the right column to the left.
numbers with sum
has a sum of
Of the given options, only the list

Explanation for Incorrect Answer A :


Choice (A) is not correct. If these numbers were switched with the circled
numbers, then the average of the numbers in the left column would be

but the average of the numbers in the right column would be

file://E:\\b3.htm

2006-11-12

8/17

The Official SAT Online Course

Explanation for Incorrect Answer B :


Choice (B) is not correct. If these numbers were switched with the circled

numbers, then the average of the numbers in the left column would be

but the average of the numbers in the right column would be

Explanation for Incorrect Answer C :


Choice (C) is not correct. If these numbers were switched with the circled

numbers, then the average of the numbers in the left column would be

but the average of the numbers in the right column would be

Explanation for Incorrect Answer D :


Choice (D) is not correct. If these numbers were switched with the circled

numbers, then the average of the numbers in the left column would be

P.M.,

By

P.M.,

of the junior class had arrived at a school dance. By

te
r

10

ed

but the average of the numbers in the right column would be

more juniors had arrived, raising attendance to

people are in the junior class?


(A)

of the junior class. How many

is

(B)

(C)

nR
eg

(D)

(E)

ANSWERS

AND EXPLANATIONS

Explanation for Correct Answer D :

Choice (D) is correct. If the difference between

of the junior class is

students, then

of the junior class and

students make up exactly

of the junior class. Therefore, the total number of students in the

junior class is

Explanation for Incorrect Answer A :


students in the entire junior
Choice (A) is not correct. If there were only
which
and
class, the whole class would have arrived between

contradicts the statement in the problem that

of the class had arrived by

Explanation for Incorrect Answer B :


Choice (B) is not correct. If there were only
and when
would have shown up by

people in the junior class,


more students had arrived

by

the fraction of the class present would have been

Explanation for Incorrect Answer C :


Choice (C) is not correct. If there were only

file://E:\\b3.htm

not

people in the junior class,

2006-11-12

9/17

The Official SAT Online Course

more students had arrived

and when

would have shown up by

not

the fraction of the class present would have been

by

Explanation for Incorrect Answer E :


Choice (E) is not correct. If there were
and when
would have shown up by

people in the junior class,


more students had arrived by

not

the fraction of the class present would have been

11

above, what is the value of

re
d

In the equilateral triangle


(A)

(B)

te

(C)

(D)

eg
is

(E)

ANSWERS

AND EXPLANATIONS

Explanation for Correct Answer D :


is equilateral, the measure of
Choice (D) is correct. Since triangle
Since the measure of
and
thus,
is
the three angles of the small triangle on the left measure

nR

and

gives

Solving for

therefore,

is

Explanation for Incorrect Answer A :


Choice (A) is not correct. Since triangle
angles in the small triangle on the left are

is equilateral,
and

and the
If

and the sum of the angles in the small


then
were
triangle on the left would be greater than

Explanation for Incorrect Answer B :


Choice (B) is not correct. Since triangle
angles in the small triangle on the left are

is equilateral,
and

and the
If

and the sum of the angles in the small


then
were
triangle on the left would be greater than

Explanation for Incorrect Answer C :


Choice (C) is not correct. Since triangle
angles in the small triangle on the left are

is equilateral,
and

and the
If

and the sum of the angles in the small


then
were
triangle on the left would be greater than

Explanation for Incorrect Answer E :


Choice (E) is not correct. Since triangle

angles in the small triangle on the left are

were

file://E:\\b3.htm

then

is equilateral,

and

and the

If

and the sum of the angles in the small triangle

2006-11-12

10/17

The Official SAT Online Course

on the left would be less than

12

what is the value of

In the table above, if


(A)

(B)
(C)
(D)
(E)

AND EXPLANATIONS

ed

ANSWERS

Explanation for Correct Answer D :


Choice (D) is correct. The values in each row of the table must satisfy the
Substitution of the values from the second row of the
equation

er

Substitution of the values


table into the equation yields
from the third row of the table into the equation yields

is
t

eg

Explanation for Incorrect Answer A :


is the -coordinate of the point with
Choice (A) is not correct.
coordinate

nR

Explanation for Incorrect Answer B :


is the value of
Choice (B) is not correct.
value of

but the question asks for the

Explanation for Incorrect Answer C :


but this does not give the value of
Choice (C) is not correct.

Explanation for Incorrect Answer E :


but this does not give the value of
Choice (E) is not correct.

13

is twice the radius of the


In the figure above, the radius of the circle with center
What is the radius of the circle with center
circle with center

file://E:\\b3.htm

2006-11-12

11/17

The Official SAT Online Course

(A)

(B)

(C)

(D)

(E)

ANSWERS

AND EXPLANATIONS

Explanation for Correct Answer B :


is equal to
Choice (B) is correct. The radius of the circle with center
Since the
is equal to
The radius of the circle with center
is twice the radius of the circle with center
radius of the circle with center

This simplifies to the equation

it follows that

Therefore, the radius of the circle with

and further to

center

is

Explanation for Incorrect Answer A :


were
Choice (A) is not correct. If the radius of the circle with center
and the ratio of the radii of the circles would be
would equal
then

te
re

Explanation for Incorrect Answer C :


were
Choice (C) is not correct. If the radius of the circle with center
and the ratio of the radii of the circles would be
would equal
then

eg
is

Explanation for Incorrect Answer D :


were
Choice (D) is not correct. If the radius of the circle with center
and the ratio of the radii of the circles would be
would equal
then

nR

Explanation for Incorrect Answer E :


were
Choice (E) is not correct. If the radius of the circle with center
and the ratio of the radii of the circles would be
would equal
then

14

A weather-watch camera is set so that its shutter opens every

seconds. If

represents the number of times the cameras shutter opens in

hours, which of the

following defines
(A)
(B)
(C)
(D)
(E)

ANSWERS

AND EXPLANATIONS

Explanation for Correct Answer D :


Choice (D) is correct. Since there are

file://E:\\b3.htm

seconds in an hour, the number

2006-11-12

12/17

The Official SAT Online Course

hours, the shutter will

If this goes on for

of openings per hour is

times. Hence,

open

Explanation for Incorrect Answer A :


Choice (A) is not correct. This equation corresponds to the situation in which
times each hour.
the shutter opens

Explanation for Incorrect Answer B :


Choice (B) is not correct. This equation corresponds to the situation in which
times per
times each hour, or
the shutter opens
second.

Explanation for Incorrect Answer C :


Choice (C) is not correct. This corresponds to a situation in which the camera
shutter opens less than once an hour.

Explanation for Incorrect Answer E :


Choice (E) is not correct. According to this equation, the more time that
passes, the fewer times the shutter opens. Since the shutter opens every
seconds, this does not make sense.

Marble was red.


was not red.
Marble
was blue.
Marble
was the same color as marble
Marble
was the same color as marble
Marble
marblessome red, some white, and some blue. The information
A jar contained
is the total number of
marbles that were drawn from the jar. If
above is about
blue marbles drawn, which of the following statements must be true?

te

15

re

The only possible value of

is

(B)

The only possible value of

is

nR
eg

is

(A)

(C) The only possible value of

(D) The only possible values of

are

and

are

and

(E)

is

The only possible values of

ANSWERS

AND EXPLANATIONS

Explanation for Correct Answer E :


Choice (E) is correct. Either the five colors, listed in order, were red, white,
were both white) or red, blue,
and
blue, red, and white (if marbles
were both blue). In the first case,
and
blue, red, and blue (if marbles
there was only one blue marble, in the second case there were three blue
or
marbles so

Explanation for Incorrect Answer A :


Choice (A) is not correct. is a possible value for
value.

but

is also a possible

Explanation for Incorrect Answer B :


was blue. If
were red and
Choice (B) is not correct. Marbles and
were two then the given information would be contradicted since marbles
were the same color (either both white or both blue).
and

Explanation for Incorrect Answer C :


is a possible value for
Choice (C) is not correct.
value.

Explanation for Incorrect Answer D :


Choice (D) is not correct. Marbles and

file://E:\\b3.htm

but

were red and

is also a possible

was blue, so

2006-11-12

13/17

The Official SAT Online Course

is not a possible value because the


is a possible value. However,
were either both
and
given information would be contradicted (marbles
white or both blue).

16

which of the following must be equal to

If

In the figure above,


(A)

(B)

(C)

(D)

AND EXPLANATIONS

te

ANSWERS

re

(E)

Explanation for Correct Answer E :


are supplementary, so
and
Choice (E) is correct. The angles labeled
are corresponding angles, so
and
The angles labeled
substitution gives
Since
and
and
from which it follows that
are supplementary, as are the angles
and
The angles labeled
Now that the measures of all the
and
so
and
labeled
labeled angles have been found, substitution of the correct values in the five
options shows that only

is

eg

nR

Explanation for Incorrect Answer A :


but
Choice (A) is not correct.

Explanation for Incorrect Answer B :


but
Choice (B) is not correct.

Explanation for Incorrect Answer C :


but
Choice (C) is not correct.

Explanation for Incorrect Answer D :


but
Choice (D) is not correct.

17

If the integer
divided by

is divided by

the remainder is

What is the remainder if

is

(A)
(B)
(C)
(D)
(E)

file://E:\\b3.htm

2006-11-12

14/17

The Official SAT Online Course

ANSWERS

AND EXPLANATIONS

Explanation for Correct Answer C :


Choice (C) is correct. This problem can be solved quickly by choosing a value
is
that satisfies the if clause of the first sentence. For example, if
is divided
then the remainder is , and when
divided by
is a counterexample to options (A),
Thus,
the remainder is
by
(B), (D), and (E), so option (C) is the only possibility for the correct answer.

can be proven as follows.


More generally, that the remainder must be
it follows that
is divided by
when
Since there is a remainder of
is some integer. Therefore, the expression
where
must equal
the only
is divided by
When
is equal to
goes evenly into
because
part that contributes to the remainder is
it follows that the
is
is divided by
. Since the remainder when
is also
is divided by
remainder when

Explanation for Incorrect Answer A :


when
then the remainder is
is equal to
Choice (A) is not correct. If
is
and the remainder when
However,
is divided by
not
is
divided by

ed

Explanation for Incorrect Answer B :


when
then the remainder is
is equal to
Choice (B) is not correct. If
is
and the remainder when
However,
is divided by
not
is
divided by

er

Explanation for Incorrect Answer D :


when
then the remainder is
is equal to
Choice (D) is not correct. If
is
and the remainder when
However,
is divided by
not
is
divided by

is
t

nR
eg

Explanation for Incorrect Answer E :


then the remainder is
is equal to
Choice (E) is not correct. If
and the remainder when
However,
is divided by
not
is
divided by

18

when
is

minutes
cents for the first
The daytime telephone rate between two cities is
percent on
cents for each additional minute. The total charge is reduced
and
-minute call made at
P.M. The cost, in dollars, of a
calls made after
P.M. between these two cities is

(A)

(B)
(C)

(D)

(E)

ANSWERS

AND EXPLANATIONS

Explanation for Correct Answer B :


Choice (B) is correct. First, consider the cost of the phone call without the

reduction. The first

There are

or

minutes cost

minutes remaining, and each minute costs

dollars, for a subtotal of

cost of the call is

Now, consider the

dollar.

cents, or

cents,

dollars. Therefore, the full

dollars.

reduction in price. Subtracting

from the

original price yields

file://E:\\b3.htm

2006-11-12

15/17

The Official SAT Online Course

Explanation for Incorrect Answer A :


Choice (A) is not correct. Here, only the cost of the first
should be
Also, the term
are reduced by
the expression being in terms of dollars.

minutes of the call


to account for

Explanation for Incorrect Answer C :


should be
Choice (C) is not correct. Here, the term
minutes of the call in terms of dollars.
the cost of the last

to account for

Explanation for Incorrect Answer D :


of the
Choice (D) is not correct. Here, the total cost of the call is
should
Also,
the
term
daytime rate, rather than being reduced by
to account for the expression being in terms of dollars.
be

Explanation for Incorrect Answer E :


of the
Choice (E) is not correct. Here, the total cost of the call is
minutes are
Also,
daytime rate, rather than being reduced by
cents per minute, which would make the entire call
charged at
minutes.

or

as their

re

(A)

as their first digit and

How many positive four-digit integers have


last digit?

19

(B)

te

(C)

(D)
(E)

is

AND EXPLANATIONS

eg

ANSWERS

Explanation for Correct Answer C :


ways of choosing the hundreds digit and
Choice (C) is correct. There are
or
digits
ways of choosing the tens digit (any of the
ways of filling the ones
may go into either of these positions) There are
or a ), and only way to fill the thousands
digit (it can only be either a
Therefore, the number of integers satisfying the
digit since it must be
stated conditions is

nR

Explanation for Incorrect Answer A :


Choice (A) is not correct. The list
numbers of this form but there are more:
contains

Explanation for Incorrect Answer B :


Choice (B) is not correct. This is the number of positive three-digit numbers
as their last digit.
or
that have

Explanation for Incorrect Answer D :


Choice (D) is not correct. This is the number of positive three-digit number
as their first digit.
or
that have

Explanation for Incorrect Answer E :


Choice (E) is not correct. The complete list of numbers of this form is
numbers.
and contains only

20

file://E:\\b3.htm

2006-11-12

16/17

The Official SAT Online Course

squares.

units is divided into


In the figure above, a square with sides of length
What is the area of the circle (not shown) that passes through the points

which are the centers of the four corner squares?

and

(A)

square units

(B)

square units

(C)

square units

(D)

square units

(E)

square units

AND EXPLANATIONS

ANSWERS

re

Explanation for Correct Answer B :


Choice (B) is correct. Since the sides of the large square have a length of
units. The points
units, each of the small squares has sides of length
are separated by a full side of a small square, and two half sides of a
and

te

small square, or a total of

units. Similarly,

and

and
units, and so on. Moreover, the points
are separated by
units. The
are the vertices of a square that has sides of length
are diameters of the circle that passes through
diagonals of square

is

and

the points

Since the sides

and

and the diagonal

form a right isosceles triangle, the length of the

of square

eg

diagonal (and the diameter of the circle) is

and therefore the radius of

Therefore, the area of the circle is

nR

the circle is

square units.

Explanation for Incorrect Answer A :


Choice (A) is not correct. The area of the circle would be

the diameter of the circle,

had length

square units if

units, but the length of

units.

is

Explanation for Incorrect Answer C :


Choice (C) is not correct. The area of the circle would be

the diameter of the circle,

had length

square units if

units, but the length of

is

units.

Explanation for Incorrect Answer D :


Choice (D) is not correct. The area of the circle would be

the diameter of the circle,

is

had length

square units if

units, but the length of

units.

Explanation for Incorrect Answer E :


Choice (E) is not correct. Since the circle is contained inside the large square,
the area of the circle must be smaller than the area of the large square, but
square units.
square units is greater than

file://E:\\b3.htm

2006-11-12

17/17

The Official SAT Online Course

Back to Score Report

Privacy Policy

Copyright 2006 The College Board. All rights reserved.

Terms of Use

Contact Us

re

is
te

eg

nR

file://E:\\b3.htm

2006-11-12

1/22

The Official SAT Online Course

Help | Profile | My Organizer | My Bookmarks | Logout

Answers and Explanations

Back to Score Report

Test Sections

Section 1

View Answers and Explanations

Section 2

Online - Practice Test #2

Section 3

Section 4

Section 6

Although visitors initially may find touring the city by subway to be -------, they are
pleased to discover that subways are an inexpensive and ------- way to get around.

Section 7

(A)

wasteful. . generous

Section 8

(B)

daunting. . efficient

Section 9

(C) extravagant. . prohibitive

Section 10

(D) convenient. . solitary

enjoyable. . easy

ANSWERS

AND EXPLANATIONS

ed

(E)

Explanation for Correct Answer B :


Choice (B) is correct. "Daunting" means intimidating. "Efficient" means effective. If
one were to insert these terms into the text, the sentence would read "Although
visitors initially may find touring the city by subway to be daunting, they are
pleased to discover that subways are an inexpensive and efficient way to get
around." The first part of the sentence describes how the visitors feel "initially," or
at first, about traveling by subway. The second part of the sentence describes a
way in which the visitors' views change. It is likely that visitors to a city would at
first find its subways intimidating. It is also likely that once visitors discover that
subways are an inexpensive and effective way to travel, they would be pleased.

er

is
t

eg

nR

Explanation for Incorrect Answer A :


Choice (A) is incorrect. "Wasteful" means inclined to waste, or uneconomical.
"Generous" means giving. If one were to insert these terms into the text, the
sentence would read "Although visitors initially may find touring the city by subway
to be wasteful, they are pleased to discover that subways are an inexpensive and
generous way to get around." It is not likely that visitors to a city would find its
subways to be uneconomical, since subways are stated to be inexpensive. In
addition, the word "generous" doesn't describe a mode of transportation.

Explanation for Incorrect Answer C :


Choice (C) is incorrect. "Extravagant" means excessive. "Prohibitive" means tending
to discourage. If one were to insert these terms into the text, the sentence would
read "Although visitors initially may find touring the city by subway to be
extravagant, they are pleased to discover that subways are an inexpensive and
prohibitive way to get around." It is not likely that visitors to a city would find its
subways to be excessive, since subways are said to be inexpensive. It is also not
likely that visitors would be pleased by something tending to discourage them.

Explanation for Incorrect Answer D :


Choice (D) is incorrect. "Convenient" means easy or suitable. "Solitary" means
alone. If one were to insert these terms into the text, the sentence would read
"Although visitors initially may find touring the city by subway to be convenient,
they are pleased to discover that subways are an inexpensive and solitary way to
get around." It is likely that visitors to a city would find its subways to be
"convenient," or an easy means of traveling. However, since the subway is a type
of public transportation, travel by subway is, by definition, not "solitary."

Explanation for Incorrect Answer E :


Choice (E) is incorrect. "Enjoyable" means pleasing or satisfying. "Easy" means
simple or not difficult. If one were to insert these terms into the text, the sentence
would read "Although visitors initially may find touring the city by subway to be
enjoyable, they are pleased to discover that subways are an inexpensive and easy
way to get around." If visitors initially find touring by subway to be pleasing, then
discovering such travel to be both inexpensive and simple doesn't reflect the

file://E:\\b4.htm

2006-11-12

2/22

The Official SAT Online Course

contrast implied by the term "although."

One critic asserts that modern urban architecture causes sensory deprivation because
it fails to provide visual and tactile -------.
(A)

latency

(B)

stimulation

(C) complacence

(D) confusion

(E)

extension

ANSWERS

AND EXPLANATIONS

Explanation for Correct Answer B :


Choice (B) is correct. "Stimulation" means excitement or arousal. If one were to
insert this term into the text, the sentence would read "One critic asserts that
modern urban architecture causes sensory deprivation because it fails to provide
visual and tactile stimulation." In an environment of sensory deprivation, the
senses are deprived of "stimulation," or arousal. People in such an environment
would have nothing to see (visual arousal) or feel (tactile arousal).

re

Explanation for Incorrect Answer A :


Choice (A) is incorrect. "Latency" is the state of being dormant or inactive. If one
were to insert this term into the text, the sentence would read "One critic asserts
that modern urban architecture causes sensory deprivation because it fails to
provide visual and tactile latency." Architecture is accused of failing to provide
something interesting to the senses, not failing to provide a state of inactivity.

te

is

Explanation for Incorrect Answer C :


Choice (C) is incorrect. "Complacence" means self-satisfaction. If one were to insert
this term into the text, the sentence would read "One critic asserts that modern
urban architecture causes sensory deprivation because it fails to provide visual and
tactile complacence." Although people in an environment of sensory deprivation
might feel a lack of self-satisfaction, the environment is not necessarily the cause.
Architecture is accused of failing to provide something interesting to the senses, not
failing to provide self-satisfaction.

eg

nR

Explanation for Incorrect Answer D :


Choice (D) is incorrect. "Confusion" means bewilderment or puzzlement. If one
were to insert this term into the text, the sentence would read "One critic asserts
that modern urban architecture causes sensory deprivation because it fails to
provide visual and tactile confusion." Failing to puzzle has nothing to do with
sensory deprivation caused by architecture.

Explanation for Incorrect Answer E :


Choice (E) is incorrect. "Extension" means something additional. If one were to
insert this term into the text, the sentence would read "One critic asserts that
modern urban architecture causes sensory deprivation because it fails to provide
visual and tactile extension." Architecture may cause sensory deprivation, but that
has nothing to do with its ability to provide something additional.

Because little rain falls in the district during summer, municipalities are necessarily
------- to ------- water from winter storms.
(A)

ready. . squander

(B)

reluctant. . retain

(C) free. . absorb

(D) careful. . store

(E)

unwilling. . conserve

ANSWERS

file://E:\\b4.htm

AND EXPLANATIONS

2006-11-12

3/22

The Official SAT Online Course

Explanation for Correct Answer D :


Choice (D) is correct. "Careful" means showing concern. "Store" means to put aside
for future use. If one were to insert these terms into the text, the sentence would
read "Because little rain falls in the district during summer, municipalities are
necessarily careful to store water from winter storms." In order to have enough
water during the summer, when little rain falls, "municipalities," or cities and towns,
must take advantage of water that is available during the winter. After collecting
water during the winter, they must show concern to put it aside so it can be
available during the dry summer.

Explanation for Incorrect Answer A :


Choice (A) is incorrect. "Ready" means prepared. "Squander" means to waste. If
one were to insert these terms into the text, the sentence would read "Because little
rain falls in the district during summer, municipalities are necessarily ready to
squander water from winter storms." In order to have enough water during the
summer, municipalities would have to be prepared to collect water that is available
during the winter. Wasting it wouldn't ensure sufficient water during the summer.

Explanation for Incorrect Answer B :


Choice (B) is incorrect. "Reluctant" means hesitant or disinclined. "Retain" means to
keep. If one were to insert these terms into the text, the sentence would read
"Because little rain falls in the district during summer, municipalities are necessarily
reluctant to retain water from winter storms." If the municipalities were hesitant to
keep water from winter storms, it is unlikely that they would have water for use in
the dry summer.

re

Explanation for Incorrect Answer C :


Choice (C) is incorrect. "Free" means not controlled. "Absorb" means to take in
something. If one were to insert these terms into the text, the sentence would read
"Because little rain falls in the district during summer, municipalities are necessarily
free to absorb water from winter storms." Although cities and towns may be free to
make choices, such as to collect water, it doesn't make sense to speak of cities or
towns absorbing water. The word "absorb" describes a physical process of taking in
something (for example, the way sponges take in water).

te

is

Explanation for Incorrect Answer E :


Choice (E) is incorrect. "Unwilling" means not willing or disinclined. "Conserve"
means to save. If one were to insert these terms into the text, the sentence would
read "Because little rain falls in the district during summer, municipalities are
necessarily unwilling to conserve water from winter storms." It is likely that the lack
of water would make municipalities more willing to save water in winter, rather than
making them "unwilling" to do so.

eg

nR

Toni Cade Bambaras novels are engrossing because the protagonists, in striving to
achieve goals, are not simply ------- characters.
(A)

passive

(B)

tangible

(C) abandoned

(D) autonomous

(E)

redundant

ANSWERS

AND EXPLANATIONS

Explanation for Correct Answer A :


Choice (A) is correct. "Passive" means inactive. If one were to insert this term into
the text, the sentence would read "Toni Cade Bambara's novels are engrossing
because the protagonists, in striving to achieve goals, are not simply passive
characters." This sentence makes sense because inactive characters would not
strive to achieve goals and would thus be less "engrossing," or interesting to read
about.

Explanation for Incorrect Answer B :


Choice (B) is incorrect. "Tangible" refers to something truly real, something capable

file://E:\\b4.htm

2006-11-12

4/22

The Official SAT Online Course

of being physically handled or grasped. If one were to insert this term into the text,
the sentence would read "Toni Cade Bambara's novels are engrossing because the
protagonists, in striving to achieve goals, are not simply tangible characters." An
author can create characters who seem real, but it is impossible to make a
character in a book physically real.

Explanation for Incorrect Answer C :


Choice (C) is incorrect. "Abandoned" means left behind. If one were to insert this
term into the text, the sentence would read "Toni Cade Bambara's novels are
engrossing because the protagonists, in striving to achieve goals, are not simply
abandoned characters." Characters who have been left behind might be
"engrossing," or interesting to read about, but such characters would not
necessarily be trying hard to achieve goals.

Explanation for Incorrect Answer D :


Choice (D) is incorrect. An "autonomous" character is self-directing or independent.
If one were to insert this term into the text, the sentence would read "Toni Cade
Bambara's novels are engrossing because the protagonists, in striving to achieve
goals, are not simply autonomous characters." It is unclear why "striving to achieve
goals" would show a character to be not independent.

Explanation for Incorrect Answer E :


Choice (E) is incorrect. "Redundant" means unnecessary. If one were to insert this
term into the text, the sentence would read "Toni Cade Bambara's novels are
engrossing because the protagonists, in striving to achieve goals, are not simply
redundant characters." There is no logical connection between "redundant"
characters and characters who try hard to achieve goals.

te
re
d

Once his integrity had been -------, the mayoral candidate was quick both to ------these attacks and to issue counterattacks.
(A)

debunked . . buttress

(B)

restored . . recommence

(C) revoked . . relinquish

is

(D) impugned . . repudiate

vitiated . . avoid

eg

(E)

ANSWERS

AND EXPLANATIONS

nR

Explanation for Correct Answer D :


Choice (D) is correct. "Impugned" means attacked by words or arguments.
"Repudiate" means to reject as untrue or unjust. If one were to insert these terms
into the text, the sentence would read "Once his integrity had been impugned, the
mayoral candidate was quick both to repudiate these attacks and to issue
counterattacks." The phrase "these attacks" indicates that the first missing term will
have a meaning similar to "attacked." When one's "integrity," or honesty, is
attacked, it is reasonable to reject the attacks by claiming them to be unjust or in
error.

Explanation for Incorrect Answer A :


Choice (A) is incorrect. "Debunked" means to expose something as false. "Buttress"
means to strengthen or support. If one were to insert these terms into the text, the
sentence would read "Once his integrity had been debunked, the mayoral candidate
was quick both to buttress these attacks and to issue counterattacks." People might
indeed try to "debunk" a candidate's integrity, or expose his integrity as false.
However, the candidate wouldn't want to strengthen the attacks on himself.

Explanation for Incorrect Answer B :


Choice (B) is incorrect. "Restored" means renewed or brought back to an original
state. "Recommence" means to begin again. If one were to insert these terms into
the text, the sentence would read "Once his integrity had been restored, the
mayoral candidate was quick both to recommence these attacks and to issue
counterattacks." If the candidate's integrity had just been "restored," then he
probably would not want to begin the attacks all over again.

Explanation for Incorrect Answer C :


Choice (C) is incorrect. "Revoked" means taken back. "Relinquish" means to give
something up. If one were to insert these terms into the text, the sentence would

file://E:\\b4.htm

2006-11-12

5/22

The Official SAT Online Course


read "Once his integrity had been revoked, the mayoral candidate was quick both to
relinquish these attacks and to issue counterattacks." "Integrity," or honesty, can't
be taken back. Also, a candidate could not "relinquish," or give up, attacks made on
him by another person.

Explanation for Incorrect Answer E :


Choice (E) is incorrect. "Vitiated" means corrupted. "Avoid" means to stay away
from something. If one were to insert these terms into the text, the sentence would
read "Once his integrity had been vitiated, the mayoral candidate was quick both to
avoid these attacks and to issue counterattacks." The candidate might try to stay
away from attacks. However, a candidate's "integrity," or honesty, could not be
corrupted by attacks, as the sentence implies. Attacks might declare a person to be
dishonest, but such attacks cannot make him dishonest.

Passage 1

Before silent film star Charlie Chaplin


(1899-1977)
came along, tramps and hoboes had long been
a part of
the Anglo-American cartoon and comic strip
tradition.
But Chaplin was to raise the tramp figure to
Line
heights of
poetic and mythic power. Chaplins famous
5
Tramp is a
human being down and out on his luck but full
of passion
for life and hope that things will get better. He
is complex
and many-sided, thereby touching most human
beings at
one or more points in our character and
makeup. There is
a good deal in his nature that most of us
10
identify with in
our secret selves, apart from what we are in
the public
world we inhabit.

ed

er

is
t

eg

nR

Passage 2

Chaplin was very forthcoming during a 1957


interview about how much the early comic strips
Weary
Willie and Tired Tim influenced his creation of
15
his
own Tramp character. Theres been a lot
said about
how I evolved the little tramp character who
made my
name, said Chaplin. Deep, psychological
stuff has
been written about how I meant him to be a
symbol
file://E:\\b4.htm

2006-11-12

6/22

The Official SAT Online Course

of all the class war, of the love-hate concept,


the deathwish, and what-all. But if you want the simple
Chaplin
truth behind the Chaplin legend, I started the
little tramp
simply to make people laugh and because those
other
tramps, Weary Willie and Tired Tim, had always
made
25 me laugh.

20

Given Chaplins statement in lines 22-25 (I . . . laugh), he would most likely


view Passage 1s portrayal of the famous Tramp (line 5) as
(A)

misleading readers about his creative intention

(B)

disregarding his effort to render social commentary through humor

(C) implying that the Tramp was derived from a comic strip

(D) asserting that the Tramp was the only character he portrayed

assuming that few could embrace his ideas

ANSWERS

ed

(E)

AND EXPLANATIONS

er

Explanation for Correct Answer A :


Choice (A) is correct. Passage 1 makes assertions regarding the Tramp's complex
and various human qualities that lines 22-25 in Passage 2 contradict. Passage 2
highlights Chaplin's creative use of the figure of the Tramp to make people laugh.
Laughter is never mentioned in Passage 1.

st

eg
i

Explanation for Incorrect Answer B :


Choice (B) is incorrect. In Passage 2 Chaplin states that he wanted nothing more
than to make people laugh. Chaplin mentions that others have attempted to see his
work as offering a social commentary, but he makes it clear that this was not his
intention.

nR

Explanation for Incorrect Answer C :


Choice (C) is incorrect. Passage 1 states that "tramps and hoboes had long been a
part of the Anglo-American cartoon and comic strip tradition" (lines 2-3), but it
does not imply that Chaplin's tramp was "derived from a comic strip."

Explanation for Incorrect Answer D :


Choice (D) is incorrect. Neither passage asserts or denies that Chaplin played other
roles in his films.

Explanation for Incorrect Answer E :


Choice (E) is incorrect. Passage 1 suggests that, on the contrary, most people were
moved by Chaplin's Tramp.

Compared to the description of Chaplins Tramp in Passage 1, the account of the


Tramp in Passage 2 is less
(A)

optimistic

(B)

ambiguous

(C) sincere

(D) complicated

(E)

humorous

ANSWERS

file://E:\\b4.htm

AND EXPLANATIONS

2006-11-12

7/22

The Official SAT Online Course

Explanation for Correct Answer D :


Choice (D) is correct. In Passage 2 Chaplin says the Tramp was solely intended to
make people laugh, whereas Passage 1 assigns a variety of characteristics to the
Tramp: "he is complex and many-sided" (lines 7-8). Consequently, the account of
the Tramp in Passage 2 is less complicated.

Explanation for Incorrect Answer A :


Choice (A) is incorrect. Neither passage is optimistic or pessimistic.

Explanation for Incorrect Answer B :


Choice (B) is incorrect. Both passages make their points clearly, and therefore
neither can be labeled ambiguous.

Explanation for Incorrect Answer C :


Choice (C) is incorrect. Both passages appear sincere and honest in their opinions.

Explanation for Incorrect Answer E :


Choice (E) is incorrect. Neither passage relies on humor to make its points.

(A)

more defensive

(B)

more laudatory

In comparison to Passage 2, the tone of Passage 1 is

re

(C) more sentimental

(D) less analytical

te

less pretentious

eg
is

(E)

ANSWERS

AND EXPLANATIONS

Explanation for Correct Answer B :


Choice (B) is correct. In Passage 1 the writing is quite "laudatory," or full of praise.
According to Passage 1, "Chaplin was to raise the tramp figure to heights of poetic
and mythic power" (lines 4-5). Passage 2, on the other hand, consists largely of
quotes from Chaplin that show the actor's humility.

nR

Explanation for Incorrect Answer A :


Choice (A) is incorrect. Neither passage contains anything that could be described
as defensive; both discuss the Tramp in very positive, open manner.

Explanation for Incorrect Answer C :


Choice (C) is incorrect. Although both passages deal with emotions, neither is
"sentimental," or overly emotional.

Explanation for Incorrect Answer D :


Choice (D) is incorrect. Passage 1 attempts to analyze the "poetic and mythic
power" (line 5) of the Tramp figure, whereas Passage 2 reveals that he was created
"simply to make people laugh" (line 23). Passage 1 should not be described as "less
analytical" than Passage 2.

Explanation for Incorrect Answer E :


Choice (E) is incorrect. If anything, Passage 1 is "more pretentious" than Passage
2. Lines such as "there is a good deal in his nature that most of us identify with in
our secret selves" (lines 9-11) could certainly be described as "pretentious," or
exaggerated.

Which best describes the relationship between Passage 1 and Passage 2?


(A)

file://E:\\b4.htm

Passage 1 explains the profound effect of Chaplins Tramp on audiences;


Passage 2 describes how Chaplin created the Tramp.

2006-11-12

8/22

The Official SAT Online Course

(B)

Passage 1 explores how Chaplin expanded the Tramps character;


Passage 2 analyzes the Tramps impact on audiences.

(C) Passage 1 examines the origin of the Tramp figure; Passage 2 traces the
comedic evolution of the Tramp.

(D) Passage 1 illustrates how Chaplin gained fame as the Tramp; Passage 2
discusses Chaplins love of comic figures like the Tramp.

(E)

Passage 1 argues that Chaplin added depth to the Tramp; Passage 2


focuses on Chaplins purpose in developing the Tramp.

ANSWERS

AND EXPLANATIONS

Explanation for Correct Answer E :


Choice (E) is correct. Lines 4-12 of Passage 1 tell how Chaplin enlarged the comicstrip figure of the Tramp and raised the Tramp "to heights of poetic and mythic
power." In Passage 2 Chaplin reveals his thinking about the character and what his
purpose was in creating the character: "simply to make people laugh" (line 23).

Explanation for Incorrect Answer A :


Choice (A) is incorrect. Passage 2 describes Chaplin's purpose in creating the
Tramp. It does not discuss how Chaplin created the Tramp, which would involve
information about the first film in which the Tramp appeared. Passage 1 discusses
the appeal of the Tramp, but it focuses more on the Tramp's qualities than on the
character's effect on audiences.

ed

Explanation for Incorrect Answer B :


Choice (B) is incorrect. Passage 1 does indeed show how Chaplin enlarged the
Tramp's character, but there is no discussion of audiences in Passage 2.

er

is
t

Explanation for Incorrect Answer C :


Choice (C) is incorrect. Passage 1 only briefly examines the Tramp's history before
Chaplin, and Passage 2 does not analyze how the Tramp changed over time.

Explanation for Incorrect Answer D :


Choice (D) is incorrect. Although Passage 2 does discuss some of Chaplin's favorite
comic figures, Passage 1 merely states that Chaplin was a "silent film star" (line 1).
Passage 1 does not show how Chaplin gained fame.

eg

nR

The following passage is from a 1994 novel about a young woman named Sophie who
at age eleven had left Haiti to join her mother in New York.

I was eighteen and going to start college in


the fall. We
moved to a one-family house in a tree-lined
neighborhood;
my mother continued working her two jobs, but
she put in
Line even longer hours.
Before the move, I had been going to a
5
Haitian Adventist
school. They guaranteed that they would get
me into college
and they had lived up to their pledge. My
mother couldnt
have been happier. Her sacrifices had paid off.
I never told my mother that I hated the
Bilingual
Institution. It was as if I had never left Haiti. All
10
the
lessons were in French, except for English

file://E:\\b4.htm

2006-11-12

9/22

The Official SAT Online Course

composition
and literature classes.
When my mother was home, she made me
read out
loud from the English Composition textbooks.
The first
words I read sounded like rocks falling in a
15
stream. Then
very slowly things began to take on meaning.
There were
words that I heard often. Words that jump out
of New York
1

re

20

Creole conversations, like the last kernel in a


cooling popcorn machine. Words like TV, building, and
feeling. There
were other words that helped too, words that
looked almost
the same in French, but were pronounced
differently in
English: nationality, alien, race, enemy.
Eventually, I
began to read better. I answered swiftly when
my mother
asked me a question in English.
Sophie, there is a great responsibility that
comes with
knowledge, to study hard, my mother would
say. I spent
six years doing nothing but that. School, home,
and prayer.
And then, I fell in love with Joseph. He
broke the monotony of my life when he moved next door. He
was the color
of ground coffee, with a voice like molasses
that turned to
music when he held a saxophone to his lips.
One day, he came to our door and asked if
he could use
the phone. After his call, he announced that he
had gotten a
job. I am a musician.
I know, I said. Sometimes I hear you
playing at night.
Does it bother you?
Non, its very pretty.
I detect an accent, he said.
Oh please, say a small one, I thought. After
seven years

te

is

25

eg

nR

U
30

35

40 in this country, I was tired of having people


file://E:\\b4.htm

2006-11-12

10/22

The Official SAT Online Course

detect my
accent. I wanted to sound completely American,
especially
for him.
Where are you from? he asked.
Haiti.
45
Ah, do you speak Creole?

50

Oui, oui, 2 I ventured, for a laugh.


We, we, he said, pointing to me and
him, We have
something in common. I speak a form of
Creole, too. I am
from Louisiana. My parents considered
themselves what
we call Creoles. Is it a small world or what?
Later that week, Joseph brought me a
sandwich to thank
me for letting him use the phone. He stayed
while I ate.
What are you going to study in college?
he asked.
I think I am going to be a doctor.
You think? Is this something you like?
I suppose so, I said.
You have to have a passion for what you
do.
My mother says its important for us to
have a doctor
in the family.
What if you dont want to be a doctor?
Theres a difference between what
people want and
whats good for them.
You sound like you are quoting
someone, he said.
My mother.
What would Sophie like to do? he asked.
That was the problem. Sophie really wasnt
sure. I had
never really dared to dream on my own.
It is okay not to have your future on a
map, he said.
That way you can flow wherever life takes
you.
That is not Haitian, I said. Thats
very American.
What is?
Being a wanderer. The very idea.
I am not American, he said. I am
African American.

55

te
re

is

60

nR
eg

65

70

file://E:\\b4.htm

2006-11-12

11/22

The Official SAT Online Course

What is the difference?


The African. Most Haitians are of African
75
descent. So
you see, it means that you and I, we are
already part of each
other.
1 The French-derived language of Haiti
2 French word for yes, pronounced we

10

Sophie hated (line 9) her school because


(A)

she resented how hard her mother had to work to send her there

(B)

she had little exposure to English

(C) it was in a neighborhood that seemed foreign and unfriendly

(D) the courses were too difficult

ANSWERS

AND EXPLANATIONS

the teachers were intolerant of her language errors

re

(E)

Explanation for Correct Answer B :


Choice (B) is correct. Although Sophie now lives in an English-speaking country,
she receives most of her instruction in French, the language of Haiti. It is this lack
of exposure to English that leads Sophie to hate her school.

is
te

eg

Explanation for Incorrect Answer A :


Choice (A) is incorrect. Although Sophie is aware of her mothers two jobs and
long hours, she never expresses resentment about her mothers working to send
her to school.

nR

Explanation for Incorrect Answer C :


Choice (C) is incorrect. No mention is made of the neighborhood where the school
is located.

Explanation for Incorrect Answer D :


Choice (D) is incorrect. Although Sophie says that things[in English class] take
on meaning slowly, she does not indicate that the courses are difficult. On the
contrary, she suggests that she would like them to be in English and thus more
difficult for her.

Explanation for Incorrect Answer E :


Choice (E) is incorrect. Although we infer from her comments about pronunciation
that Sophie makes mistakes, no mention is made of teachers or their expectations.

11

The comparison in line 15 emphasizes the


(A)

halting way in which Sophie thought she read

(B)

powerful impact of the words Sophie read aloud

(C) feeling Sophie had about her native language

(D) obstacles to Sophies writing in a new language

(E)

strength of Sophies commitment to learn English

ANSWERS

AND EXPLANATIONS

Explanation for Correct Answer A :

file://E:\\b4.htm

2006-11-12

12/22

The Official SAT Online Course

Choice (A) is correct. Sophies notion that her first words sound like "rocks falling
in a stream" shows that she hears the words as discrete sounds, unconnected to
one another. Consequently, the comparison is used to describe the halting or
choppy pace of Sophie's speech.

Explanation for Incorrect Answer B :


Choice (B) is incorrect. Sophie does not feel that her words have impact. It is not
until later in the passage that Sophie's words "take on meaning."

Explanation for Incorrect Answer C :


Choice (C) is incorrect. Here Sophie is speaking not French but English, a language
still unfamiliar to her.

Explanation for Incorrect Answer D :


Choice (D) is incorrect. Sophie is speaking, not writing, the words that sound like
"rocks falling in a stream."

Explanation for Incorrect Answer E :


Choice (E) is incorrect. The comparison to "rocks falling in a stream" suggests
Sophies hesitant and difficult beginning, not the strength of her commitment.

Sophies manner of answering her mother (lines 23-24) shows her

ed

12

(A)

struggle with an anger that she has tried to conceal

(B)

impatience with her mothers persistent questioning

er

(C) growing command of a new language

(D) need to revert to French to express her feelings

eagerness to return to her reading as quickly as possible

ANSWERS

is
t

(E)

AND EXPLANATIONS

eg

Explanation for Correct Answer C :


Choice (C) is correct. Because of Sophies command of English or progress in
English, she is able to swiftly respond to her mother. Sophie also indicates her
growing command of English when she says Eventually, I began to read better.

nR

Explanation for Incorrect Answer A :


Choice (A) is incorrect. The speed of Sophies answers is a sign of growing
proficiency, not anger.

Explanation for Incorrect Answer B :


Choice (B) is incorrect. Although Sophies mother does ask [her]question[s]
in English, there is no indication that her mother is persistent or that Sophie is
impatient.

Explanation for Incorrect Answer D :


Choice (D) is incorrect. All of Sophies observations in lines 13-24 have to do with
speaking English, not French.

Explanation for Incorrect Answer E :


Choice (E) is incorrect. There is no indication that Sophie wants to return to her
reading; she is simply answering her mother's questions in English.

13

The use of italics in line 55 serves to emphasize


(A)

Josephs idealism as contrasted with Sophies cynicism

(B)

Josephs unsuccessful attempt to cheer Sophie

(C) Josephs incredulity at Sophies approach to her future

the unlikelihood that Sophie will be able to pursue her goal

file://E:\\b4.htm

2006-11-12

13/22

The Official SAT Online Course

(D)
(E)

the extent to which Sophie has underestimated her talents

ANSWERS

AND EXPLANATIONS

Explanation for Correct Answer C :


Choice (C) is correct. By repeating the word think with emphasis, Joseph
shows his incredulity, or disbelief, at what she has just told him about her thoughts
of becoming a doctor. Consequently, the italics indicate that Joseph cannot believe
how little Sophie has questioned her plans for the future.

Explanation for Incorrect Answer A :


Choice (A) is incorrect. Joseph expresses no idealism as he repeats the word
think, nor does Sophie give any evidence of cynicism, or scorn.

Explanation for Incorrect Answer B :


Choice (B) is incorrect. Josephs repetition of Sophies word choice is not an
attempt to cheer her, nor is she in a situation to need cheering. Rather, Joseph is
highlighting the tentative quality of her prior claim.

re

Explanation for Incorrect Answer D :


Choice (D) is incorrect. Joseph does not challenge Sophies ability to attain her
goal; rather, he underlines the tentative quality of what she has just said.

Explanation for Incorrect Answer E :


Choice (E) is incorrect. Josephs emphasis on the word think calls attention to
Sophies tentative statement, not any underestimation of her own talent.

te

eg
is

14

Sophies response in line 56 reveals that she


(A)

is anxious to impress others

(B)

is reluctant to confess her deepest fears

(C) is single-minded in her dedication to a medical career

nR

(D) has apparently decided that she should hide her heritage from Joseph

(E)

has rarely questioned the decisions others have made for her

ANSWERS

AND EXPLANATIONS

Explanation for Correct Answer E :


Choice (E) is correct. In using the tentative phrase I suppose so, Sophie
indicates that she has not questioned her mother, who says its importantto
have a doctor in the family. Her hesitance suggests that her mother decided she
should go to medical school and Sophie never really thought she could question
that decision.

Explanation for Incorrect Answer A :


Choice (A) is incorrect. Sophies use of the word suppose shows a lack of
certainty, not a wish to impress.

Explanation for Incorrect Answer B :


Choice (B) is incorrect. Nowhere in the passage does the author provide evidence
that Sophie experiences deep fears.

Explanation for Incorrect Answer C :


Choice (C) is incorrect. The hesitant nature of Sophies word choice makes clear
that she is not strongly dedicated to the idea of becoming a doctor.

Explanation for Incorrect Answer D :


Choice (D) is incorrect. Although Sophie wishes to speak without an accent, she has
already told Joseph that she comes from Haiti.

file://E:\\b4.htm

2006-11-12

14/22

The Official SAT Online Course

15

In line 61, the difference is between


(A)

selfishness and altruism

(B)

desire and practicality

(C) intuitive knowledge and learned knowledge

(D) love for family and love for friends

(E)

duty to the past and fear of the future

ANSWERS

AND EXPLANATIONS

Explanation for Correct Answer B :


Choice (B) is correct. Sophie is speaking of the contrast between what she might
want for herself and what she has been led to believe is good for her. If Sophie
follows what she wants, she follows her desire. If she follows whats good for
[her], she is being practical.

te
re

Explanation for Incorrect Answer A :


Choice (A) is incorrect. Although what people want can be seen as selfishness,
whats good for them cannot necessarily be seen as altruism, or devotion to
the welfare of others.

Explanation for Incorrect Answer C :


Choice (C) is incorrect. Although what people want can be seen as intuitive
knowledge, whats good for [people] cannot be identified as learned
knowledge in this context. This context is not referring to a difference between
types of knowledge but a difference between motivationsthe difference between
motivation according to personal desire and motivation according to practicality.

eg
is

Explanation for Incorrect Answer D :


Choice (D) is incorrect. For Sophie whats good for [her] may involve a sense
of duty to her mother, but nothing in the passage suggests any conflict between
this and a love for friends.

Explanation for Incorrect Answer E :


Choice (E) is incorrect. There is no indication that either what people want or
whats good for them is rooted in either duty to the past or future.

By using Sophies name (line 65) instead of you, Joseph is attempting to

16

nR

(A)

pose as a narrator of a story

(B)

approach a frightening topic gradually

(C) make Sophie consider a new perspective

(D) appear unconcerned about Sophies attitude

(E)

pretend that he is unaware of Sophies presence

ANSWERS

AND EXPLANATIONS

Explanation for Correct Answer C :


Choice (C) is correct. By referring to her in the third person as Sophie rather
than in the second person as you, Joseph asks her to consider herself from a
new point of view, as someone autonomous whose wishes are separate from those
of her mother.

Explanation for Incorrect Answer A :


Choice (A) is incorrect. Joseph is asking questions, not posing as any sort of
narrator.

Explanation for Incorrect Answer B :

file://E:\\b4.htm

2006-11-12

15/22

The Official SAT Online Course

Choice (B) is incorrect. The narrative provides no evidence that the topic of choice is
frightening to Sophie or that using her name is meant to avoid frightening her.

Explanation for Incorrect Answer D :


Choice (D) is incorrect. Joseph shows that he is indeed concerned about Sophies
submissive approach by persisting in his questions and by asking her to consider
herself an independent individual, capable of choice.

Explanation for Incorrect Answer E :


Choice (E) is incorrect. Since he directs the question at Sophie, Joseph clearly is not
pretending he does not know she is there.

17

The problem (line 66) for Sophie is that


(A)

what she wants and what her mother wants are radically different

(B)

medical school would require her mother to make even more financial
sacrifices

(C) Joseph expects her to follow his dreams instead of her own

(D) she is uncomfortable with the long years of schooling that becoming a
doctor entails

she never considered her own needs as important

ANSWERS

AND EXPLANATIONS

(E)

er
e

Explanation for Correct Answer E :


Choice (E) is correct. Sophie says, "I had never really dared to dream on my own"
instead, she had taken cues from her mother. Sophie is clearly stating that the
"problem" is that she is not sure what she wants because she has never thought
about what she wants for her future.

is
t

Explanation for Incorrect Answer A :


Choice (A) is incorrect. Although what her mother wants is clear, Sophie is not sure
what she wants, nor has she ever "dared to dream on [her] own."

eg

Explanation for Incorrect Answer B :


Choice (B) is incorrect. The mother's financial sacrifices are mentioned earlier in the
passage but not in this conversation. In addition, it is the mother who wants Sophie
to become a doctor.

nR

Explanation for Incorrect Answer C :


Choice (C) is incorrect. Joseph has already been identified as a musician. He is not
trying to get Sophie to join him, but to make her own decisions.

Explanation for Incorrect Answer D :


Choice (D) is incorrect. Sophie does not address the length of time it would take to
complete medical training.

18

Josephs statement in lines 68-69 (It is . . . you) primarily shows him to be


(A)

more tenacious than Sophies mother is

(B)

more tolerant of ambiguity than Sophie is

(C) more cynical about the future than Sophie is

(D) unsentimental about family and heritage

(E)

incapable of making commitments

ANSWERS

AND EXPLANATIONS

Explanation for Correct Answer B :


Choice (B) is correct. By saying it's "okay not to have your future on a map,"
Joseph is urging Sophie to see her future in a less restricted way than she has
previously seen it. Josephs suggestions that Sophies future does not need to
be mapped and that she can flow wherever life takes [her] indicate his ability

file://E:\\b4.htm

2006-11-12

16/22

The Official SAT Online Course


to handle uncertainty, or ambiguity.

Explanation for Incorrect Answer A :


Choice (A) is incorrect. By indicating that ones future need not be mapped out,
Joseph shows himself to be less tenacious, or single-minded, than Sophies
mother seems to be.

Explanation for Incorrect Answer C :


Choice (C) is incorrect. Nothing in Josephs comments about the flexibility of the
future suggests cynicism.

Explanation for Incorrect Answer D :


Choice (D) is incorrect. Joseph addresses only the ways Sophie might manage her
future. He does not discuss or allude to family or heritage.

Explanation for Incorrect Answer E :


Choice (E) is incorrect. Joseph does not suggest that he dislikes commitments. He
simply indicates that the future need not be rigidly planned.

ed

The following passage is from an essay written in 1991 about fences in suburban
culture.

In the United States, the traditional view


embraced
by society is that fences are European, out of
place in the
American landscape. This notion turns up
repeatedly in
nineteenth-century American writing about the
Line
landscape.
One author after another denounces the
5
Englishmans
insultingly inhospitable brick wall, topped with
broken
bottles. Frank J. Scott, an early landscape
architect who
had a large impact on the look of Americas
first suburbs,
worked tirelessly to rid the landscape of fences,
which he
derided as a feudal holdover from Britain.
10
Writing in 1870,
he held that to narrow our own or our
neighbors views of
the free graces of Nature was selfish and
undemocratic. To
drive through virtually any American suburb
today, where
every lawn steps right up to the street in a
gesture of openness and welcome, is to see how completely
15
such views
have triumphed. After a visit to the United
States, British

er

is
t

eg

nR

file://E:\\b4.htm

2006-11-12

17/22

The Official SAT Online Course

20

25

novelist Vita Sackville-West decided that


Americans . . .
have no sense of private enclosure.
In many American suburbs such as the one
where I grew
up, a fence or a hedge along the street meant
one thing: the
family who lived behind it was antisocial,
perhaps even had
something to hide. Fences and hedges said:
Ogres within;
skip this place on Halloween. Except for these
few dubious
addresses, each little plot in our development
was landscaped
like a miniature estate, the puniest expanse
of unhedged
lawn was made to look like a public park. Any
enjoyment
of this space was sacrificed to the conceit of
wide-open land,
for without a fence or hedge, front yards were
much too
public to spend time in. Families crammed their
activities
into microscopic back-yards, the one place
where the usefulness of fences and hedges seemed to
outweigh their
undemocratic connotations.
But the American prejudice against fences
predates the
suburban development. Fences have always
seemed to us
somehow un-American. Europeans built walled
gardens;
Americans from the start distrusted the hortus

re

te

is

30

eg

nR

U
35

conclusus .
If the space within the wall was a garden, then
what was
that outside the wall? To the Puritans the whole
American
landscape was a promised land and to draw
lines around
sections of it was to throw this paramount idea
40
into question.
When Anne Bradstreet, the Massachusetts
colonys first
poet, set about writing a traditional English
garden ode, she

tore down the conventional garden wallor (it


file://E:\\b4.htm

2006-11-12

18/22

The Official SAT Online Course

comes to
the same thing) made it capacious enough to
take in the
45 whole of America.
The nineteenth-century transcendentalists,
too, considered
the American landscape Gods second
book and they
taught us to read it for moral instruction.
Residues of this
idea persist, of course; we still regard and write
about nature
with high moral purpose (an approach that still
50
produces a
great deal of pious prose). And though, in our
own nature
writing, guilt seems to have taken the rhetorical
place of
nineteenth-century ecstasy, the essential
religiosity remains.
We may no longer spell it out, but most of us
still believe
the landscape is somehow sacred, and to
55
meddle with it
sacrilegious. And to set up hierarchies within
itto set off
a garden from the surrounding countryside
well, that
makes no sense at all.

ed

er

is
t

eg

nR

* A Latin phrase that means confined garden

In line 1, embraced most nearly means

19

(A)

caressed

(B)

adopted

(C) enfolded

(D) included

(E)

encircled

ANSWERS

AND EXPLANATIONS

Explanation for Correct Answer B :


Choice (B) is correct. "Adopted" means taken. In line 1, the author states, "the
traditional view embraced by society is that fences are European." In this context,
"embraced" signifies "taken" or "adopted."

Explanation for Incorrect Answer A :


Choice (A) is incorrect. "Caressed" means hugged. It is not possible to hug a point
of view.

Explanation for Incorrect Answer C :


Choice (C) is incorrect. "Enfolded" means enclosed. It is not possible to enclose

file://E:\\b4.htm

2006-11-12

19/22

The Official SAT Online Course

something non-material like a point of view.

Explanation for Incorrect Answer D :


Choice (D) is incorrect. "Included" means involved. In line 1, the author states, "the
traditional view embraced by society is that fences are European." "Included" does
not mean the same thing as "embraced" in this context.

Explanation for Incorrect Answer E :


Choice (E) is incorrect. "Encircled" means surrounded. Encircled does not refer
to taking in or assuming an idea as ones own.

20

In lines 10-12, Frank J. Scotts observation implies that nature


(A)

is graceful and beautiful only in areas uninhabited by humans

(B)

should be available for all to enjoy without hindrance

(C) must be incorporated into the design of American suburbs

(D) exerts a more powerful effect on the British than on Americans

(E)

is less evident in American suburbs than in the British countryside

ANSWERS

AND EXPLANATIONS

te
re
d

Explanation for Correct Answer B :


Choice (B) is correct. In these lines, Scott describes erecting fences as "selfish and
undemocratic." In addition, he describes the presence of fences as something that
would "narrow our own or our neighbor's views of the free graces of Nature."
Clearly, Scott believes that nature "should be available for all to enjoy without
hindrance."

is

Explanation for Incorrect Answer A :


Choice (A) is incorrect. Scott writes only about inhabited areas, where people may
choose to erect fences. In addition, his observations imply that nature can be
graceful and abundant in areas that are inhabited.

eg

Explanation for Incorrect Answer C :


Choice (C) is incorrect. Although the passage does state that Scott had an impact
on the design of early suburbs, lines 10-12 address only his dislike of the way
fences obstruct one's view.

nR

Explanation for Incorrect Answer D :


Choice (D) is incorrect. Although Scott does compare the use of fences in Britain
and America, he does not compare the effects of nature on the inhabitants of the
two countries.

Explanation for Incorrect Answer E :


Choice (E) is incorrect. In lines 10-12, Scott does not compare the presence of
nature in American suburbs and the British countryside.

21

In lines 12-15, To drive . . . welcome suggests that suburban lawns


(A)

represent the American preoccupation with appearances

(B)

epitomize the values of capitalism

(C) reflect a particular American attitude

(D) emulate the inviting character of formal British gardens

(E)

reveal the distinctive personality of a homeowner

ANSWERS

AND EXPLANATIONS

Explanation for Correct Answer C :


Choice (C) is correct. The author suggests that the "openness and welcome" of the
lawns reflect the attitude of the people who own them. This attitude is described as
characteristically American.

file://E:\\b4.htm

2006-11-12

20/22

The Official SAT Online Course

Explanation for Incorrect Answer A :


Choice (A) is incorrect. The author calls attention to "a gesture of openness and
welcome," not to any "preoccupation," or concern, with appearance.

Explanation for Incorrect Answer B :


Choice (B) is incorrect. The author does not mention capitalism or its values.

Explanation for Incorrect Answer D :


Choice (D) is incorrect. The passage makes it clear that British gardens are not
open, welcoming, or inviting.

Explanation for Incorrect Answer E :


Choice (E) is incorrect. The statement refers to the lawns in "virtually any American
suburb," linking them in a common "gesture of openness and welcome." The author
does not mention individual homeowners or their distinctive personalities.

22

In line 27, conceit most nearly means


(A)

grandiose ideal

(B)

extreme vanity

(D) ornate article

ed

(C) ingenious expression

extravagant edifice

ANSWERS

is
te
r

(E)

AND EXPLANATIONS

Explanation for Correct Answer A :


Choice (A) is correct. "Conceit" means idea. A "grandiose ideal," an ideal more
elaborate than necessary, is a fitting way to describe the "conceit" of tiny parcels of
lawn pretending to look like "wide-open land."

eg

nR

Explanation for Incorrect Answer B :


Choice (B) is incorrect. In this context "conceit" does not refer to "vanity," or an
overly favorable opinion of ones own ability.

Explanation for Incorrect Answer C :


Choice (C) is incorrect. It would be odd to speak of an "expression" of wide-open
land, or to describe maintaining wide-open land as "ingenious," or inventive.

Explanation for Incorrect Answer D :


Choice (D) is incorrect. The passage is about an idea, not a material thing. In
addition, the ideal of wide-open land with fences is the opposite of "ornate," or
elaborate.

Explanation for Incorrect Answer E :


Choice (E) is incorrect. An edifice is a building, not "wide-open land."

23

The discussion in lines 41-45 (When Anne . . . America) implies that Anne
Bradstreets garden poetry
(A)

reflected her disapproval of Americas rapid development

(B)

followed the pattern set by more innovative British poets

(C) espoused the Puritans belief in religious freedom

(D) promoted greater enthusiasm for gardening in America

(E)

file://E:\\b4.htm

focused on the vast American landscape rather than on individual gardens

2006-11-12

21/22

The Official SAT Online Course

ANSWERS

AND EXPLANATIONS

Explanation for Correct Answer E :


Choice (E) is correct. In lines 41-45, the author's discussion centers on Anne
Bradstreet's transformation of the traditional garden ode. Bradstreet changed the
focus of the traditional garden ode by "[tearing] down the conventional garden
wall." Removing the garden wall eliminates the separation between individual
gardens. This allows "the garden" in Bradstreet's work to stand for the "vast
American landscape."

Explanation for Incorrect Answer A :


Choice (A) is incorrect. The discussion of Bradstreet does not mention disapproval
of America's rapid development.

Explanation for Incorrect Answer B :


Choice (B) is incorrect. In this passage, it is Bradstreet who appears innovative
("she tore down the conventional garden wall") as she alters the traditional garden
ode.

Explanation for Incorrect Answer C :


Choice (C) is incorrect. This section of the passage does not discuss Puritan
religious beliefs.

re

Explanation for Incorrect Answer D :


Choice (D) is incorrect. Bradstreet does not "promote," or advance, the practice of
gardening in this passage. Further, the gardens and garden wall of Bradstreets
poetry are figurative, not literal.

te

24

eg
is

In line 56, the hierarchies most closely represent


(A)

a practical but undesirable consequence of urban life

(B)

a crucial method of determining territorial claims

(C) a misguided division of natures sacred space

nR

(D) an effort to protect pristine land from development

(E)

an unfortunate legacy of nineteenth-century America

ANSWERS

AND EXPLANATIONS

Explanation for Correct Answer C :


Choice (C) is correct. "Hierarchies" means a graded or ranked series. The passage
describes "set[ting] off a garden from the surrounding countryside," thus creating a
hierarchy or ranked series. The author clearly views the act of set[ting] off a
garden from the surrounding countryside" as both "sacrilegious" and as a misguided
division of "nature's sacred space."

Explanation for Incorrect Answer A :


Choice (A) is incorrect. Although setting off a garden from the area around it is
undesirable, this section of the passage does not present it as practical. In fact, the
author states that it "makes no sense at all."

Explanation for Incorrect Answer B :


Choice (B) is incorrect. The author does not mention any territorial claims. Instead;
emphasis is placed on the "sacrilegious" nature of "meddl[ing] with" the landscape.

Explanation for Incorrect Answer D :


Choice (D) is incorrect. Although landscape is referred to as "somehow sacred," it is
not described as "pristine," or unspoiled, and development is not mentioned.

Explanation for Incorrect Answer E :


Choice (E) is incorrect. The author clearly does not see fences as an American
legacy, but rather as a British one.

file://E:\\b4.htm

2006-11-12

22/22

The Official SAT Online Course

Back to Score Report

Privacy Policy

Copyright 2006 The College Board. All rights reserved.

Terms of Use

Contact Us

re

te

is

nR
eg

file://E:\\b4.htm

2006-11-12

1/11

The Official SAT Online Course

Help | Profile | My Organizer | My Bookmarks | Logout

Answers and Explanations

Back to Score Report

Test Sections

Section 1

View Answers and Explanations

Section 2

Online - Practice Test #2

Section 3

Section 4

then

If

Section 6

(A)

Section 7

(B)

Section 8

(C)

Section 9

(D)

Section 10

(E)

ANSWERS

AND EXPLANATIONS

re

Choice (D) is correct. Since

Explanation for Correct Answer D :

te

Explanation for Incorrect Answer A :


in the equation, the
Choice (A) is not correct. When 0.0001 is substituted for
which is not equal to
result is

is

nR
eg

Explanation for Incorrect Answer B :


is substituted for
Choice (B) is not correct. When
which is not equal to
is

in the equation, the result

Explanation for Incorrect Answer C :


is substituted for
Choice (C) is not correct. When

is

Explanation for Incorrect Answer E :


is substituted for
Choice (E) is not correct. When

If

is

in the equation, the

which is not equal to

result is

in the equation, the result

which is not equal to

less than

and

is

more than

what is the value of

when

(A)
(B)

(C)

(D)

(E)

ANSWERS

AND EXPLANATIONS

Explanation for Correct Answer A :


less than
is
Choice (A) is correct. If
When
and

file://E:\\b5.htm

and

is

more than
and

then

2006-11-12

2/11

The Official SAT Online Course

Explanation for Incorrect Answer B :


Choice (B) is not correct. If the value of
then
If
would be

then
were
and the value of

so the value of
not
would be

Explanation for Incorrect Answer C :


Choice (C) is not correct. If the value of
then
If
would be

then
were
and the value of

so the value of
not
would be

Explanation for Incorrect Answer D :


Choice (D) is not correct. If the value of

were

then

If

would be

Explanation for Incorrect Answer E :


Choice (E) is not correct. If the value of
then
If
would be

so the value of

then

not

and the value of

would be

then
were
and the value of

so the value of
not
would be

ed

st
er

(A)

nR

(B)

eg
i

people were asked


The pictograph above shows the results of a survey in which
to indicate which of four hot beverages they drink. How many more responses did
Coffee receive than Hot cider?

(C)

(D)

(E)

ANSWERS

AND EXPLANATIONS

Explanation for Correct Answer D :


Choice (D) is correct. In the pictograph, the responses for Coffee are shown by a
full cups, whereas the responses for Hot cider are shown by a row of only
row of

cups. Since each cup corresponds to

responses, while Hot cider received


responses than Hot cider.

responses, Coffee received

responses, so Coffee received

more

Explanation for Incorrect Answer A :


is the number of responses that Hot cider received,
Choice (A) is not correct.
but the question asks for the difference between the number of responses Coffee
received and the number of responses Hot cider received.

Explanation for Incorrect Answer B :


is the number of responses that Hot chocolate
Choice (B) is not correct.
received, but the question asks for the difference between the number of responses
Coffee received and the number of responses Hot cider received.

Explanation for Incorrect Answer C :

file://E:\\b5.htm

2006-11-12

3/11

The Official SAT Online Course

Choice (C) is not correct. There are four and one-half more full cups in the Coffee
responses.
row than the in Hot cider row and the half-cup represents

Explanation for Incorrect Answer E :


is the number of responses that Coffee received, but
Choice (E) is not correct.
the question asks for the difference between the number of responses Coffee
received and the number of responses Hot cider received.

could equal which of the

is a positive even integer, then

If

following?
(A)
(B)
(C)
(D)
(E)

ANSWERS

AND EXPLANATIONS

Explanation for Correct Answer C :

is positive and even,

Choice (C) is correct. Since

is a positive odd

is a positive even number, one more than

number, and

is the

re

only number among the options that is the product of consecutive integers of which
the smaller is odd. That is

te

Explanation for Incorrect Answer A :


Choice (A) is not correct. The factors

is

and

are consecutive positive

and
as the product of two positive integers
integers. The only ways to write
In each of the products, the integers
and
are as
where
are not consecutive.

eg

Explanation for Incorrect Answer B :


Choice (B) is not correct. The factors

and

are consecutive positive

nR

and
as the product of two positive integers
integers. The only ways to write
In the first two products, the
and
are as
where
which is not
integers are not consecutive. The last product corresponds to
an even integer.

Explanation for Incorrect Answer D :


Choice (D) is not correct. The factors

and

are consecutive positive

as the product of two positive integers


integers. The only ways to write
In each of the
and
are as
where
products, the integers are not consecutive.

Explanation for Incorrect Answer E :


Choice (E) is not correct. The factors

integers. The only ways to write


are as
where
integers are not consecutive.

and

and

are consecutive positive

and
as the product of two positive integers
In each of the products, the
and

file://E:\\b5.htm

2006-11-12

4/11

The Official SAT Online Course

represents shirts with pockets, circular region


In the figure above, circular region
represents shirts with
represents shirts with buttons, and circular region
collars. What is represented by the shaded region?
(A)

Shirts with pockets, buttons, and collars

(B)

Shirts with pockets and buttons, but without collars

(C) Shirts with pockets and buttons (some possibly with collars)

Shirts with buttons and collars (some possibly with pockets)

ANSWERS

te

(E)

re

(D) Shirts with pockets and collars (some possibly with buttons)

AND EXPLANATIONS

Explanation for Correct Answer C :

is

Choice (C) is correct. The shaded region covers all of the area where

and

overlap, so every shirt represented by this region must have both pockets and
so some of these shirts have collars.
buttons. Some of the shaded region is in

eg

nR

Explanation for Incorrect Answer A :


Choice (A) is not correct. Shirts with pockets, buttons, and collars would be

represented by only the small region in the center where all three regions

overlap. The shaded region includes but is larger than this small center
and
region.

Explanation for Incorrect Answer B :


Choice (B) is not correct. If none of these shirts have collars, then none of the
shaded region would be in

Explanation for Incorrect Answer D :


Choice (D) is not correct. These shirts would be represented by the region where
overlap.
and

Explanation for Incorrect Answer E :


Choice (E) is not correct. These shirts would be represented by the region where

and

overlap.

file://E:\\b5.htm

2006-11-12

5/11

The Official SAT Online Course

What is the value of

is

In the figure above, the slope of line

(A)
(B)
(C)
(D)
(E)

ANSWERS

AND EXPLANATIONS

Explanation for Correct Answer E :


Choice (E) is correct. The slope is the change in

This simplifies to

not

were

is
te

be

re

Explanation for Incorrect Answer A :


Choice (A) is not correct. If the value of

so

divided by the change in

then the slope of line

would

Explanation for Incorrect Answer B :

Choice (B) is not correct. If the value of

then the slope of line

would

were

then the slope of line

would

were

then the slope of line

would

not

eg

be

were

nR

Explanation for Incorrect Answer C :

Choice (C) is not correct. If the value of

not

be

Explanation for Incorrect Answer D :


Choice (D) is not correct. If the value of

be

not

and

If

where

which of the following must be equal to

(A)
(B)
(C)
(D)
(E)

ANSWERS

file://E:\\b5.htm

AND EXPLANATIONS
2006-11-12

6/11

The Official SAT Online Course

Explanation for Correct Answer A :

Choice (A) is correct. Multiplying both sides of the equation

and therefore

it follows that

Since

equation

gives the

by

Explanation for Incorrect Answer B :

by

Multiplying both sides of the equation

so

gives the equation

is equal to

However,

would also equal

would be equal to

then

were equal to

Choice (B) is not correct. If

Explanation for Incorrect Answer C :

and

Choice (C) is not correct. If

in this case.

Explanation for Incorrect Answer D :

but

and

Therefore,

in this case.

does not equal

However,

Therefore,

is
t

and

respectively, then

have values

and

Choice (D) is not correct. If

but

er
ed

does not equal

However,

and

respectively, then

and

have values

Explanation for Incorrect Answer E :

eg

Choice (E) is not correct. If

nR

respectively, then

does not equal

and

have values

However,

but

Therefore,

in this case.

U
8

and

and

In the figure above,

lies on

In terms of

which of the following must be

equivalent to

(A)

(B)

(C)

(D)

(E)

file://E:\\b5.htm

2006-11-12

7/11

The Official SAT Online Course

ANSWERS

AND EXPLANATIONS

Explanation for Correct Answer B :

is given by the expression

Choice (B) is correct. The measure of

The sum of the measures

This simplifies to

must be

Thus,

which simplifies to

to
equivalent to

is equivalent

Hence,

is

and

angle

of angle

Explanation for Incorrect Answer A :


were equivalent to
Choice (A) is not correct. If

then

which would lead to the false

would equal

statement

Explanation for Incorrect Answer C :


were equivalent to
Choice (C) is not correct. If

then

which would lead to the

would equal

which is false for the given figure because

statement

positive measure.

Therefore,

statement

then

which would lead to the

er
e

would equal

Explanation for Incorrect Answer D :


were equivalent to
Choice (D) is not correct. If

is an angle of

is equivalent to

which

only if

is not necessarily true for the given figure. The question asks for an expression that
must be equivalent to

st

Alternatively, a close examination of the figure (which is drawn to scale) shows that

which equals

Explanation for Incorrect Answer E :


were equivalent to
Choice (E) is not correct. If

then

which would lead to the

would equal

so

is less than

eg
i

the measure of

nR

which
only if
is equivalent to
Therefore,
statement
is not necessarily true for the given figure. The question asks for an expression that
must be equivalent to

In fact, a close examination of the figure (which is drawn to scale) shows that the

measure of

, which equals

is less than

so

cannot be

equivalent to

what is the value of

If

Your Response:

Correct Response(s): 5/2, 2.5

Explanation:

The correct answer is

so

10

file://E:\\b5.htm

or

. Since

the equation

This answer can also be expressed in decimal form as

also holds,

tablespoons
cups of flour and
loaves of bread requires
A recipe for making
loaves of
of baking powder. If the proportions in this recipe are to be used to make

2006-11-12

8/11

The Official SAT Online Course

bread, how many cups of flour will be needed? (Do not round your answer.)

Your Response:

Correct Response(s): 36/5, 7.2

Explanation:

or

The correct answer is

If the same proportions of the recipe are used to make

loaves of bread and

Therefore,

loaves of bread, then

of the

cups of flour are required for each

Alternatively,

so

cups of flour would be needed to make

loaves, so

loaves of bread.

This answer can also be expressed as

ed

11

er

st

eg
i

In the figure above,

and

intersect at

If

and

bisects

nR

what is the value of

Your Response:

Correct Response(s): 40

Explanation:

. Since

The correct answer is

which is

intersect at

is the same as the measure of

the measure of

12

and

which is

is half of

the measure of

Since

bisects

So

The length and width of a rectangle have integer values. If the area of the rectangle
what is one possible value for the perimeter of the rectangle?
is

Your Response:

Correct Response(s): 40 or 56 or 152

Explanation:

The area of the rectangle is


and
The possible correct answers are
and
is the width. Since
is the length and
where
so
must be integers, the only possible dimensions of this rectangle are
so the possible
The perimeter of the rectangle is
and

file://E:\\b5.htm

2006-11-12

9/11

The Official SAT Online Course

and

values of the perimeter are

13

A sequence is formed by repeating the


indefinitely. What is the sum of the first

numbers above in the same order


terms of the sequence?

Your Response:

Correct Response(s): 14

Explanation:

terms of the sequence repeat, the sum


. Since the first
The correct answer is
terms
terms of the sequence is equivalent to the sum of the first
of the first

So the sum is

of the sequence multiplied by

re

14

te

is

eg

people responded to the following question: How many weeks of


In a survey,
vacation did you take last year? Their responses to the question are summarized in
equals the number of vacation weeks indicated. How many
the chart above, where
weeks of vacation last year?
respondents took more than

nR

Your Response:

Correct Response(s): 2380

Explanation:

The correct answer is

respondents took

of the

. According to the graph,

of the

weeks or less of vacation. So,

of the

weeks of vacation. Therefore,

took more than

respondents took more than

respondents

weeks vacation last year.

15

for all values of

If

and

where

are constants, what is the value of

Your Response:

Correct Response(s): 103

Explanation:

file://E:\\b5.htm

2006-11-12

10/11

The Official SAT Online Course

The correct answer is

. Since the equation is true for all values of

must be true when

So, if

then it

and, if

so

16

re

Your Response:

Correct Response(s): 4

eg
is
te

Explanation:

The correct answer is

The length of the base is the distance from

So, the area of

is

Therefore,

nR

and

is the distance

is the height. The height of

-axis, which is

which is also

to

The total cost of a taxicab ride is the sum of

17

to the

where

. The formula for the area of a triangle is

is the length of the base and

from

What is the value of

is

-plane above, the area of

In the

(1) a basic fixed charge for using the taxicab, and

(2) an additional charge for each

If the total cost to ride

mile is

what is the total cost, in dollars, of a

of a mile that is traveled.

miles is

and the total cost to ride

-mile ride?

(Disregard the $ sign when gridding your answer. If, for example, your answer is
)
grid

Your Response:

Correct Response(s): 8.50, 17/2

Explanation:

The correct answer is

where

traveled, and

file://E:\\b5.htm

. The formula to find the total cost of a taxicab ride is

is the basic fixed charge,

is the charge for each

is the number of miles traveled. The cost to ride

of a mile

of a mile is

2006-11-12

11/11

The Official SAT Online Course

so

miles is

The cost to ride

so

. The solution to this system of equations is

Therefore, the total cost to ride

miles is

or in fraction form as

This answer can be gridded in decimal form as

18

Let the function


If

and

is the area of a semicircle with diameter


be defined so that
what is the value of

Your Response:

Correct Response(s): 10

Explanation:

is defined to be the area of a

semicircle with diameter

ed

. If the function

The correct answer is

So, the equation

then

er

can be written as

Solving for

yields

is
t

eg

nR

Copyright 2006 The College Board. All rights reserved.

Back to Score Report

Privacy Policy

Terms of Use

Contact Us

file://E:\\b5.htm

2006-11-12

1/21

The Official SAT Online Course

Help | Profile | My Organizer | My Bookmarks | Logout

Answers and Explanations

Back to Score Report

Test Sections

Section 1

View Answers and Explanations

Section 2

Online - Practice Test #2

Section 3

Section 4

A stranger, the students were surprised to see him enter the classroom carrying a
bowling ball.

Section 6

Section 7

Section 8

Section 9

(A)

A stranger, the students were surprised to see him enter the classroom
carrying a bowling ball.

(B)

A stranger carrying a bowling ball, the students were surprised to see him
entering the classroom.

(C) The students were surprised to see a stranger enter the classroom, and he
carried a bowling ball.

Section 10

(D) The students were surprised to see a stranger carrying a bowling ball
enter the classroom.

ANSWERS

AND EXPLANATIONS

The students, who were surprised to see a stranger enter the classroom
carrying a bowling ball.

re

(E)

is
te

Explanation for Correct Answer D :


Choice (D) is correct. It avoids the error of the original by placing the subject, "the
students," at the beginning and deleting the repetitive pronoun "him."

Explanation for Incorrect Answer A :


Choice (A) involves illogical word order. "A stranger," the object of the verb phrase
"were surprised to see," is placed before the subject, "the students"; it would be
more logical to place it just after the verb phrase and delete the repetitive pronoun
"him."

eg

nR

Explanation for Incorrect Answer B :


Choice (B) results in illogical word order. The phrase "A stranger carrying a bowling
ball" is placed at the beginning of the sentence; it would be more logical to replace
the pronoun "him" with that phrase.

Explanation for Incorrect Answer C :


Choice (C) involves improper coordination. It provides a coordinate clause, "and he
carried a bowling ball," instead of the modifying phrase that is needed for clarity
and conciseness.

Explanation for Incorrect Answer E :


Choice (E) results in a sentence fragment. The relative pronoun "who" creates a
subordinate clause ("were surprised . . . bowling ball"), leaving an incomplete main
clause.

Several of Frank Stellas paintings were inspired by the shapes of waves and whales,
titled after chapter headings from Moby-Dick.
(A)

paintings were inspired by the shapes of waves and whales, titled

(B)

paintings had their inspiration from the shapes of waves and whales with
titles

(C) paintings, inspired by the shapes of waves and whales, are titled

(D) paintings, which were inspired by the shapes of waves and whales and
which are titled

(E)

file://E:\\b6.htm

paintings, being inspired by the shapes of waves and whales, titled

2006-11-12

2/21

The Official SAT Online Course

ANSWERS

AND EXPLANATIONS

Explanation for Correct Answer C :


Choice (C) is correct. It avoids the error of the original by placing the verbal
phrase, "inspired by the shapes of waves and whales," immediately after the noun
it modifies, "paintings," and by making "several" the subject of the new verb "are
titled."

Explanation for Incorrect Answer A :


Choice (A) displays improper modification and word order. The verbal phrase
beginning with "titled" cannot logically modify the nouns immediately before it,
"waves and whales."

Explanation for Incorrect Answer B :


Choice (B) exhibits improper modification. The prepositional phrase "with titles"
cannot logically modify the noun immediately before it, "whales."

Explanation for Incorrect Answer D :


Choice (D) creates a sentence fragment. It contains verbs in dependent clauses
("were inspired " and "are titled") but no independent verb that can complete a
statement.

The mayor claimed that a majority of the property owners would have favored her
proposal if put to the vote.

is
te

re

Explanation for Incorrect Answer E :


Choice (E) produces a sentence fragment. Since it contains no verb (only the
verbals "being inspired" and "titled"), it does not make a complete statement.

(A)

would have favored her proposal if put

(B)

would have favored her proposal if it had been put

(C) favored her proposal if it would have been put

eg

(D) favored her proposal if put

favored her proposal if they were put

nR

(E)

ANSWERS

AND EXPLANATIONS

Explanation for Correct Answer B :


Choice (B) is correct. It avoids the error of the original by making clear that it is the
"proposal" that will be "put to the vote."

Explanation for Incorrect Answer A :


Choice (A) involves improper modification. The phrase "put to the vote" could refer
either to "the property owners" or to the "proposal."

Explanation for Incorrect Answer C :


Choice (C) involves an illogical sequence of verb tenses. The verb "favored"
indicates an event that did happen, but "would have been put" indicates that the
conditions for that event have not happened.

Explanation for Incorrect Answer D :


Choice (D) results in improper modification. The phrase "put to the vote" could
refer either to "the property owners" or to the "proposal."

Explanation for Incorrect Answer E :


Choice (E) involves improper modification. The plural pronoun "they" should be the
singular "it" to agree with the singular "proposal."

file://E:\\b6.htm

The psychologist states that most people want the same things: interesting and

2006-11-12

3/21

The Official SAT Online Course

meaningful work, respect, and to have them be loved for themselves alone.
(A)

interesting and meaningful work, respect, and to have them be loved for
themselves alone

(B)

to have interesting and meaningful work, respect, and loved for


themselves alone

(C) work that has interest and is meaningful, to have respect, and to be
beloved for themselves alone

(D) interesting and meaningful work, respect, and their own love

(E)

to have interesting and meaningful work, to be respected, and to be loved


for themselves alone

ANSWERS

AND EXPLANATIONS

Explanation for Correct Answer E :


Choice (E) is correct. It avoids the error of the original by providing three parallel
phrases, introduced by infinitives, to describe the things most people want.

Explanation for Incorrect Answer A :


Choice (A) involves a lack of parallelism. The expression "to have them be loved . .
. alone" is an infinitive phrase. It thus breaks the pattern of successive noun
phrases ("interesting and meaningful work" and "respect") established earlier in the
sentence.

er
e

Explanation for Incorrect Answer B :


Choice (B) results in a lack of parallelism. The three items in the list are an
infinitive phrase ("to have . . . work"), a noun ("respect"), and another infinitive
phrase ("loved . . . alone"), instead of the three grammatically similar phrases that
are needed.

is
t

Explanation for Incorrect Answer C :


Choice (C) is unsatisfactory because it results in a lack of parallelism. The
expression "work . . . meaningful" does not match the pattern established by the
pair of infinitive phrases that follow it.

eg

Explanation for Incorrect Answer D :


Choice (D) involves a problem with diction. The phrase "their own love" has a
meaning much different from "to be loved for themselves alone."

nR

By employing exotic harmonies and making unusual use of instruments, Mahler was a
pathfinder from romanticism to modern music.

(A)

Mahler was a pathfinder

(B)

a path was created by Mahler

(C) Mahler created a path

(D) Mahler was the creator of a path

(E)

was how Mahler created a path

ANSWERS

AND EXPLANATIONS

Explanation for Correct Answer C :


Choice (C) is correct. It avoids the error of the original by using the verb "created"
and the noun "path" to form the appropriate idiom.

Explanation for Incorrect Answer A :


Choice (A) results in an improper idiom. The verb "was" and the predicate noun
"pathfinder" precede the preposition "from," where it would be more idiomatic to
use the verb "created" and the direct object "a path."

Explanation for Incorrect Answer B :


Choice (B) creates improper modification. The introductory phrase, "By employing .
. . instruments," refers improperly to "a path" rather than to "Mahler."

file://E:\\b6.htm

2006-11-12

4/21

The Official SAT Online Course

Explanation for Incorrect Answer D :


Choice (D) results in wordiness. The phrase "was the creator of a path" could more
simply be expressed as "created a path."

Explanation for Incorrect Answer E :


Choice (E) involves improper modification. The introductory phrase "By employing .
. . instruments" is improperly placed next to the verb "was" rather than next to the
noun "Mahler," which it modifies.

In the past, many famous painters meticulously ground their own colors, an attention
to detail that is noteworthy.
(A)

an attention to detail that is noteworthy

(B)

inasmuch as they showed attention to detail, it is noteworthy

(C) this makes it noteworthy in showing their attention to detail

(D) an idea that is noteworthy in showing their attention to detail

(E)

which is noteworthy and it shows an attention to detail

ANSWERS

AND EXPLANATIONS

ed

Explanation for Correct Answer A :


Choice (A) is correct. It uses a concise noun phrase ("an attention to detail that is
noteworthy") to embed appropriate information within a single clause.

te
r

Explanation for Incorrect Answer B :


Choice (B) uses improper coordination. It links two complete thoughts ("In the
past . . . ground their own colors" and "inasmuch . . . it is noteworthy") with only a
comma.

eg
is

Explanation for Incorrect Answer C :


Choice (C) displays improper coordination. It uses only a comma to link two
complete thoughts ("In the past . . . ground their own colors" and "this makes it . .
. to detail").

nR

Explanation for Incorrect Answer D :


Choice (D) includes excess words. With a slight change in word order ("an attention
to detail that is noteworthy"), the words "idea" and "in showing their" become
unnecessary.

Explanation for Incorrect Answer E :


Choice (E) uses vague pronouns. The pronouns "which" and "it" may allude to the
act of grinding colors, but the sentence contains no noun to which they can refer.

By including pieces of cloth, newspaper, wallpaper, and other materials in his work,
Picassos innovation had an important influence on twentieth-century art.
(A)

Picassos innovation had an important influence on

(B)

this innovation of Picassos was important in its influence over

(C) Picassos important innovative influence was on

(D) Picasso was influential, with his innovation, over

(E)

the innovative Picasso was an important influence on

ANSWERS

AND EXPLANATIONS

Explanation for Correct Answer E :


Choice (E) is correct. It avoids the error of the original by providing an appropriate
subject, "the innovative Picasso," for the modifying phrase "By including . . . work."

file://E:\\b6.htm

2006-11-12

5/21

The Official SAT Online Course

Explanation for Incorrect Answer A :


Choice (A) involves improper modification. It improperly makes "Picasso's
innovation" the subject of the modifying phrase "By including . . . in his work."

Explanation for Incorrect Answer B :


Choice (B) is unsatisfactory because it results in improper modification. The phrase
"By including . . . in his work" cannot logically modify "this innovation."

Explanation for Incorrect Answer C :


Choice (C) results in an illogical sentence because of improper modification. The
phrase "By including . . . in his work" cannot logically modify "Picasso's . . .
influence."

Explanation for Incorrect Answer D :


Choice (D) exhibits awkward phrasing. It is strange to say that Picasso was
"influential, with his innovation, over twentieth-century art."

Once American films looked slick and commercial compared to European imports;
now, almost the reverse is true.
(A)

now, almost the reverse is true

(B)

now they are almost the reverse

(C) instead, there is almost a reversal now

(D) now it is almost the reverse that is true

ANSWERS

re

it has now been almost reversed

AND EXPLANATIONS

te

(E)

Explanation for Correct Answer A :


Choice (A) is correct. The adverb "now" appropriately introduces the comparison
completed by "almost the reverse is true."

is

nR
eg

Explanation for Incorrect Answer B :


Choice (B) results in improper use of a pronoun. The pronoun "they" is ambiguous,
since it can refer to either "American films" or "European imports."

Explanation for Incorrect Answer C :


Choice (C) results in a connective error. The use of the adverb "instead" improperly
suggests that the two clauses express alternatives rather than opposition.

Explanation for Incorrect Answer D :


Choice (D) involves wordiness. Neither the phrase "it is" nor the phrase "that is" is
necessary to the clause.

Explanation for Incorrect Answer E :


Choice (E) results in improper pronoun use. The singular pronoun "it" has no
singular antecedent in the sentence.

Although known primarily as a poet, the paintings of Lawrence Ferlinghetti have


recently been receiving public attention.
(A)

Although known primarily as a poet, the paintings of Lawrence Ferlinghetti

(B)

Although known primarily for his poetry, Lawrence Ferlinghettis


paintings

(C) Although his poetry is primarily what he is known for, Lawrence


Ferlinghettis paintings

(D) Although Lawrence Ferlinghetti is known primarily as a poet and his


paintings

(E)

file://E:\\b6.htm

Although Lawrence Ferlinghetti is known primarily for his poetry, his


paintings

2006-11-12

6/21

The Official SAT Online Course

ANSWERS

AND EXPLANATIONS

Explanation for Correct Answer E :


Choice (E) is correct. It avoids the error of the original by supplying the
introductory clause with a subject and verb (Lawrence Ferlinghetti is), thereby
making it clear that Ferlinghetti is known as a poet.

Explanation for Incorrect Answer A :


Choice (A) results in improper modification. The introductory adjective phrase
(Although . . . Ferlinghetti") refers improperly to the paintings, when it
should refer to Ferlinghetti.

Explanation for Incorrect Answer B :


Choice (B) creates improper modification. The opening adjective phrase
(Although . . . poetry) improperly modifies Ferlinghettis paintings rather
than Ferlinghetti.

Explanation for Incorrect Answer C :


Choice (C) involves ambiguous reference. The sentence does not contain a noun
antecedent for the pronoun his.

ed

Explanation for Incorrect Answer D :


Choice (D) results in improper coordination. The subordinate clause Although . . .
poetry is improperly joined to an independent clause his paintings . . .
attention with a coordinating conjunction rather than with a comma.

er

For many a brilliant architect, being free to innovate is more important than being
well paid.

is
t

10

(A)

being free to innovate is more important than

(B)

having freedom of innovation is more important than

(C) there is more importance in the freedom to innovate than

eg

(D) freedom to innovate has more importance than

to have the freedom to innovate is more important than

nR

(E)

ANSWERS

AND EXPLANATIONS

Explanation for Correct Answer A :


Choice (A) is correct. It uses parallel verbal phrases ("being free" and "being well
paid") to establish a clear comparison.

Explanation for Incorrect Answer B :


Choice (B) has a flaw in parallelism. The verbal phrase "having freedom" is not
parallel with the later verbal phrase "being well paid."

Explanation for Incorrect Answer C :


Choice (C) violates parallelism. The independent clause "there is more importance .
. . to innovate" is not parallel with the later verbal phrase "being well paid."

Explanation for Incorrect Answer D :


Choice (D) fails to maintain parallelism. The noun "freedom" is not parallel with the
later verbal "being."

Explanation for Incorrect Answer E :


Choice (E) displays a flaw in parallelism. The verbal phrase "to have the freedom" is
not parallel with the later verbal phrase "being well paid."

11

file://E:\\b6.htm

What was not achieved in last years county voter registration drive was more than
compensated for by this year, which registered over three thousand new voters.

2006-11-12

7/21

The Official SAT Online Course

(A)

by this year, which registered over three thousand new voters

(B)

by this year, having over three thousand new voters registered

(C) by this years drive, which registered over three thousand new voters

(D) when they registered three thousand new voters this year

(E)

this year, when they registered over three thousand new voters

ANSWERS

AND EXPLANATIONS

Explanation for Correct Answer C :


Choice (C) is correct. It avoids the error of the original by comparing two parallel
nouns, last years . . . drive and this years drive.

Explanation for Incorrect Answer A :


Choice (A) involves a lack of parallelism. A voter registration drive, last years .
. . drive, is not parallel with and cannot be compared to a year, this year.

Explanation for Incorrect Answer B :


Choice (B) creates improper modification. The adjective phrase having . . .
registered refers improperly to a year, this year, rather than to a registration
drive.

er
ed

Explanation for Incorrect Answer D :


Choice (D) results in improper use of a pronoun. The pronoun they is
ambiguous, since the sentence does not contain a noun to which it can refer.

Explanation for Incorrect Answer E :


Choice (E) creates improper pronoun use. The pronoun they is ambiguous
because it does not refer to any noun in the sentence.

is
t

12

eg

For the cyclist exploring Irelands western cliffs, every road leading out of the town

nR

of Clifden offers their own set of wonders. No error

ANSWERS

AND EXPLANATIONS

Corrected Sentence: For the cyclist exploring Irelands western cliffs, every road
leading out of the town of Clifden offers its own set of wonders.

Explanation for Correct Answer C :


The error in this sentence occurs at (C), where the number of the pronoun is
incorrect. The plural pronoun "their" does not agree with the singular noun to which
it refers, "road."

Explanation for Incorrect Answer A :


There is no error at (A). The preposition "for" is appropriate to introduce a phrase
identifying who might take the road.

Explanation for Incorrect Answer B :


There is no error at (B). The phrasal preposition "out of" appropriately links the
noun "road" with the later noun "town."

Explanation for Incorrect Answer D :


There is no error at (D). The adjective "own" correctly modifies the noun
immediately after it, "set."

Explanation for Incorrect Answer E : There is an error in the sentence.

file://E:\\b6.htm

2006-11-12

8/21

The Official SAT Online Course

13

The construction of a waterway linking the Atlantic and Pacific Oceans

was first proposed

such a project

in 1524,

become

ANSWERS

but not until

the Panama Canal opened in 1914 did

a reality No error
.

AND EXPLANATIONS

Corrected Sentence:

Explanation for Correct Answer E : There is no error in this sentence.

re

Explanation for Incorrect Answer A :


There is no error at (A). The singular verb phrase "was . . . proposed" agrees with
its singular subject, "construction," and the adjective "first" is appropriately placed.

Explanation for Incorrect Answer B :


There is no error at (B). The word "but" provides a link between the two clauses;
the phrase "not until" combines with the verb phrase "did . . . become" to form an
appropriate idiom.

te

Explanation for Incorrect Answer C :


There is no error at (C). The phrase "such a project" is appropriately used to refer
to the phrase "The construction . . . Oceans."

is

Explanation for Incorrect Answer D :


There is no error at (D). The noun phrase "a reality" is properly used to refer to the
phrase "such a project."

eg

nR

14

The charm of Loftings book lies in the humorous reversal of rolesthe animals

guide, assist, and generally they take care of the helpless humans. No error

ANSWERS

AND EXPLANATIONS

Corrected Sentence: The charm of Lofting's book lies in the humorous reversal of
roles the animals guide, assist, and generally take care of the helpless humans.

Explanation for Correct Answer D :


The error in this sentence occurs at (D), where there is a lack of parallelism. The
pronoun "they" introduces a clause, breaking the pattern of successive verbs that
precedes it.

Explanation for Incorrect Answer A :


There is no error at (A). The preposition "of" is properly used in combination with
the noun phrase "Lofting's book" to form a possessive.

Explanation for Incorrect Answer B :


There is no error at (B). The verb "lies" agrees with its singular subject "charm" and
combines with the preposition "in" to form an appropriate idiom.

file://E:\\b6.htm

2006-11-12

9/21

The Official SAT Online Course

Explanation for Incorrect Answer C :


There is no error at (C). The preposition "of" combines with the nouns "reversal"
and "roles" to form an appropriate idiom.

Explanation for Incorrect Answer E : There is an error in the sentence.

15

People were trained

industry, and so

ANSWERS

to perform

having

one tiny part of one process in one department of one

no sense

of

the process

as a whole No error
.

AND EXPLANATIONS

Corrected Sentence: People were trained to perform one tiny part of one process
in one department of one industry, and so they had no sense of the process as a
whole.

ed

Explanation for Correct Answer B :


The error in this sentence occurs at (B), where a failure to maintain parallelism
leaves the phrase after the comma incomplete. Since the verbal "having" has no
subject, it is not parallel with the earlier clause "People were trained."

er

is
t

Explanation for Incorrect Answer A :


There is no error at (A). The infinitive "to perform" correctly begins a phrase
describing how people were trained.

Explanation for Incorrect Answer C :


There is no error at (C). The preposition "of" functions properly to link its object,
the noun "process," with the earlier noun "sense."

eg

Explanation for Incorrect Answer D :


There is no error at (D). The prepositional phrase "as a whole" correctly modifies
the noun "process."

nR

Explanation for Incorrect Answer E : There is an error in the sentence.

U
16

The Stegosaurus, plant-eating dinosaurs with protective bony plates and tail spikes,

was once common in what is now Colorado. No error

ANSWERS

AND EXPLANATIONS

Corrected Sentence: The Stegosaurus, a plant-eating dinosaur with protective


bony plates and tail spikes, was once common in what is now Colorado.

Explanation for Correct Answer A :


The error in this sentence occurs at (A), where there is noun-noun disagreement
and subject-verb disagreement. The plural noun "dinosaurs" is an improper
appositive for the singular subject, "the Stegosaurus," and it does not agree with
the singular verb "was."

Explanation for Incorrect Answer B :

file://E:\\b6.htm

2006-11-12

10/21

The Official SAT Online Course

There is no error at (B). "Protective" is an appropriate adjective to modify the noun


"plates."

Explanation for Incorrect Answer C :


There is no error at (C). The adverb "once" functions properly to tell when
something happened, the adjective "common" is used correctly after the linking
verb "was" to describe "the Stegosaurus," and the preposition "in" is appropriate to
introduce a phrase telling where.

Explanation for Incorrect Answer D :


There is no error at (D). The noun clause "what is now" serves appropriately as the
object of the preposition "in."

Explanation for Incorrect Answer E : There is an error in the sentence.

17

Some plants use chemical signals that repel insects, and also, thesesignals help to

put neighboring plants on alert so they can mount their own defenses. No error

AND EXPLANATIONS

ANSWERS

re

Corrected Sentence: Some plants use chemical signals that repel insects, and
these signals help to put neighboring plants on alert so they can mount their own
defenses.

te

Explanation for Correct Answer B :


The error in this sentence occurs at (B), where the adverb "also," in addition to the
conjunction "and," results in wordiness.

is

eg

Explanation for Incorrect Answer A :


There is no error at (A). The relative pronoun "that" and the verb "repel" are
appropriately combined to begin a relative clause modifying "signals."

nR

Explanation for Incorrect Answer C :


There is no error at (C). The conjunction "so" is properly joined to the subject
"they" and the verb "can" to begin a clause. The plural pronoun "they" agrees with
the plural noun "plants," to which it refers.

Explanation for Incorrect Answer D :


There is no error at (D). The verb "mount" correctly precedes the noun phrase
"their own defenses" to form an appropriate idiom.

Explanation for Incorrect Answer E : There is an error in the sentence.

18

Innovative use of computers in the classroom allows students to undertake projects

that encourages them to be both analytical and intellectually adventurous. No error

ANSWERS

AND EXPLANATIONS

Corrected Sentence: Innovative use of computers in the classroom allows


students to undertake projects that encourage them to be both analytical and
intellectually adventurous.

Explanation for Correct Answer B :

file://E:\\b6.htm

2006-11-12

11/21

The Official SAT Online Course

The error in this sentence occurs at (B), where there is subject-verb disagreement.
The singular verb "encourages" does not agree with its plural subject, the relative
pronoun "that," which refers to the plural noun "projects."

Explanation for Incorrect Answer A :


There is no error at (A). The infinitive "to undertake" properly introduces the noun
phrase that operates as the direct object of the verb "allows."

Explanation for Incorrect Answer C :


There is no error at (C). The plural pronoun "them" agrees with the plural noun
"students," to which it refers.

Explanation for Incorrect Answer D :


There is no error at (D). The conjunction "both" combines with "and" to link the two
qualities that students are encouraged to have.

Explanation for Incorrect Answer E : There is an error in the sentence.

When one is researching the customs of a community, youmust learn aboutits

19

No error

re

history and observe its people going about their ordinary activities.

eg
is
te

ANSWERS

AND EXPLANATIONS

Corrected Sentence: When one is researching the customs of a community, one


must learn about its history and observe its people going about their ordinary
activities.

Explanation for Correct Answer B :


The error in this sentence occurs at (B), where there is an improper pronoun shift.
The person who is researching is also the person who must learn, and
therefore should be referred to by the pronoun one rather than by you.

nR

Explanation for Incorrect Answer A :


There is no error at (A). The singular verb is researching agrees with its
singular subject one.

Explanation for Incorrect Answer C :


There is no error at (C). The verb learn and the preposition about combine
to form an appropriate idiom.

Explanation for Incorrect Answer D :


There is no error at (D). The plural pronoun their agrees with its plural
antecedent people.

Explanation for Incorrect Answer E : There is an error in the sentence.

20

Working with consummate skill, Picasso sketched a portrait of the youthful but

experienced dancer who was posing for him. No error

file://E:\\b6.htm

2006-11-12

12/21

The Official SAT Online Course

ANSWERS

AND EXPLANATIONS

Corrected Sentence:

Explanation for Correct Answer E : There is no error in this sentence.

Explanation for Incorrect Answer A :


There is no error at (A). The participle "Working" and the preposition "with" are
appropriately used to introduce a modifying phrase.

Explanation for Incorrect Answer B :


There is no error at (B). The noun "portrait" and the preposition "of" combine to
form an appropriate idiom.

Explanation for Incorrect Answer C :


There is no error at (C). The word "but" is appropriately used to link two
contrasting adjectives describing the dancer.

Explanation for Incorrect Answer D :


There is no error at (D). The participle "posing" combines with the verb "was" to
form an appropriate verb form.

d
re

From 1566 until 1576 Santa Elena, now an excavation site in South Carolina, was the

te

21

capital of Spanish Florida; however, it has become an English settlement by

is

1735. No error

eg

AND EXPLANATIONS

nR

ANSWERS

Corrected Sentence: From 1566 until 1576, Santa Elena, now an excavation site
in South Carolina, was the capital of Spanish Florida; however, it had become an
English settlement by 1735.

Explanation for Correct Answer D :


The error in this sentence occurs at (D), where the present perfect verb phrase
"has become" is used instead of the pluperfect that is needed to describe an action
that had taken place "by 1735."

Explanation for Incorrect Answer A :


There is no error at (A). The words "From" and "until" are properly used to describe
the decade during which Santa Elena was the capital of Spanish Florida.

Explanation for Incorrect Answer B :


There is no error at (B). The compound noun "excavation site" is properly used in
the phrase enclosed by commas that describes what Santa Elena is now.

Explanation for Incorrect Answer C :


There is no error at (C). The conjunctive adverb "however" is properly used to
introduce a clause.

Explanation for Incorrect Answer E : There is an error in the sentence.

22

file://E:\\b6.htm

2006-11-12

13/21

The Official SAT Online Course

Despite

on
placed
the juvenile offender

the attorneys moving plea, the judge

probation for an

ANSWERS

indecisive

period.

No error

AND EXPLANATIONS

Corrected Sentence: Despite the attorney's moving plea, the judge placed the
juvenile offender on probation for an indefinite period.

Explanation for Correct Answer D :


The error in this sentence occurs at (D), where there is improper diction. The
adjective "indecisive" is used where "indefinite" is needed.

Explanation for Incorrect Answer A :


There is no error at (A). The preposition "Despite" is properly used to introduce a
prepositional phrase.

re

Explanation for Incorrect Answer B :


There is no error at (B). The verb "placed," which can be singular or plural, agrees
with its singular subject, "judge."

Explanation for Incorrect Answer C :


There is no error at (C). The preposition "on" combines with the noun "probation" to
form an appropriate prepositional phrase.

te

Explanation for Incorrect Answer E : There is an error in the sentence.

is

23

eg

Yearning for a truly representative art form of the Americas, the art world of the

nR

1920s looked hopefully to the three popular Mexican mural artists

of the day. No error

ANSWERS

AND EXPLANATIONS

Corrected Sentence:

Explanation for Correct Answer E : There is no error in this sentence.

Explanation for Incorrect Answer A :


There is no error at (A). The participle "yearning" correctly introduces a phrase
describing "the art world," and the preposition "for" functions properly within that
phrase to link the verbal "yearning" with the noun "form."

Explanation for Incorrect Answer B :


There is no error at (B). The adverb "truly" is used correctly to modify the adjective
"representative," and "representative" appropriately describes the noun "form."

Explanation for Incorrect Answer C :


There is no error at (C). The verb in past tense, "looked," correctly describes a
completed action, and the adverb "hopefully" (telling how) correctly modifies the
verb.

file://E:\\b6.htm

2006-11-12

14/21

The Official SAT Online Course

Explanation for Incorrect Answer D :


There is no error at (D). The prepositional phrase "of the day" serves appropriately
as an adjective describing the noun "artists."

24

There has always been a

opposing political views

ANSWERS

great deal of

about which

friction between

we are very vocal.

Joan and I because we


have

No error

AND EXPLANATIONS

Corrected Sentence: There has always been a great deal of friction between Joan
and me because we have opposing political views about which we are very vocal.

Explanation for Correct Answer B :


The error in this sentence occurs at (B), where there is an improper pronoun case.
The nominative case of the first-person pronoun, "I," is used where the objective
case, "me," is needed.

re

Explanation for Incorrect Answer A :


There is no error at (A). The phrase "a great deal of" is an appropriate idiom.

Explanation for Incorrect Answer C :


There is no error at (C). The subordinating conjunction "because" is appropriately
used to introduce a dependent adverbial clause; the nominative case of the
pronoun "we" is appropriately used, since it is the subject of the clause.

te

is

Explanation for Incorrect Answer D :


There is no error at (D). The preposition "about" is appropriately used to introduce
the relative pronoun "which," which refers appropriately to the plural noun "views."

eg

Explanation for Incorrect Answer E : There is an error in the sentence.

nR

25

When M. R. Harrington, an archaeologist from the Museum of the American Indian,

began to excavate the ruins he named the Pueblo Grande de Nevada, he unearthed

artifacts indicating a 500-year occupation by indigenous peoples. No error

ANSWERS

AND EXPLANATIONS

Corrected Sentence:

Explanation for Correct Answer E : There is no error in this sentence.

Explanation for Incorrect Answer A :


There is no error at (A). The subordinating conjunction "When" properly begins the
dependent adverbial clause "When . . . Nevada."

Explanation for Incorrect Answer B :


There is no error at (B). The verb "began" properly describes an action in the past

file://E:\\b6.htm

2006-11-12

15/21

The Official SAT Online Course

and joins with the infinitive phrase "to excavate" to form an appropriate idiom.

Explanation for Incorrect Answer C :


There is no error at (C). The singular pronoun "he," referring to "M.R. Harrington,"
agrees with the singular verb "named."

Explanation for Incorrect Answer D :


There is no error at (D). The participle "indicating" properly modifies the noun
"artifacts."

26

which is likely to
win a prize

The supervisor cited three workers, each of

for having suggested

ANSWERS

at

cost-effective changes

the factory.

No error

AND EXPLANATIONS

Corrected Sentence: The supervisor cited three workers, each of whom is likely to
win a prize for having suggested cost-effective changes at the factory.

ed

Explanation for Correct Answer A :


The error in this sentence occurs at (A), where there is an improper use of pronoun.
In the phrase "each of which," the pronoun "which" incorrectly refers to a person
(one of the "three workers") and should instead be "whom."

is
te
r

Explanation for Incorrect Answer B :


There is no error at (B). The singular verb "is" agrees with the singular subject
"each" and combines with the phrase "likely to" to form an appropriate idiom.

eg

Explanation for Incorrect Answer C :


There is no error at (C). The preposition "for" and the phrase "having suggested"
together form an appropriate idiom.

nR

Explanation for Incorrect Answer D :


There is no error at (D). The preposition "at" properly introduces the adjective
phrase modifying "changes."

Explanation for Incorrect Answer E : There is an error in the sentence.

27

Freedom of action and expression are at the foundation not only of our system of

government but also of our expectations concerning human relations at all levels of

society. No error

ANSWERS

AND EXPLANATIONS

Corrected Sentence: Freedom of action and expression is at the foundation not


only of our system of government but also of our expectations concerning human
relations at all levels of society.

Explanation for Correct Answer A :


The error in this sentence occurs at (A), where there is subject-verb disagreement.
The plural verb "are" does not agree with the singular subject "freedom."

file://E:\\b6.htm

2006-11-12

16/21

The Official SAT Online Course

Explanation for Incorrect Answer B :


There is no error at (B). The phrase "not only" is joined with the preposition "of" to
form an appropriate idiom that is paralleled later in the sentence with "but also of."

Explanation for Incorrect Answer C :


There is no error at (C). The noun "expectations" and the participle "concerning"
combine to form an appropriate idiom.

Explanation for Incorrect Answer D :


There is no error at (D). The preposition "at" and the adjective "all" together form
an appropriate idiom.

Explanation for Incorrect Answer E : There is an error in the sentence.

28

While both disaster rescue workers and news reporters may face physical danger,

the latter can usually control his or her exposure to risk, whereas rescue workers

er
e

ANSWERS

often cannot. No error

AND EXPLANATIONS

is
t

Corrected Sentence: While both disaster rescue workers and news reporters may
face physical danger, the latter can usually control their exposure to risk, whereas
rescue workers often cannot.

Explanation for Correct Answer C :


The error in this sentence occurs at (C), where there is noun-pronoun
disagreement. Because the pronouns "his" and "her" are both singular, neither
properly refers to the plural noun "the latter" ("news reporters").

eg

nR

Explanation for Incorrect Answer A :


There is no error at (A). The plural verb "may face" agrees with its plural compound
subjects "workers" and "reporters."

Explanation for Incorrect Answer B :


There is no error at (B). The phrase "the latter" properly refers to "news reporters."

Explanation for Incorrect Answer D :


There is no error at (D). The subordinating conjunction "whereas" properly begins
the dependent adverbial clause "whereas . . . cannot."

Explanation for Incorrect Answer E : There is an error in the sentence.

29

The refusal of the management to revise their policy on family leave caused an

uproar among employees. No error

ANSWERS

file://E:\\b6.htm

AND EXPLANATIONS

2006-11-12

17/21

The Official SAT Online Course

Corrected Sentence: The refusal of management to revise its policy on family


leave caused an uproar among employees.

Explanation for Correct Answer B :


The error in this sentence occurs at (B), where an improper pronoun is used. The
plural pronoun "their" refers incorrectly to the singular noun "management."

Explanation for Incorrect Answer A :


There is no error at (A). The infinitive phrase "to revise" operates as an nominative
adjective describing "refusal."

Explanation for Incorrect Answer C :


There is no error at (C). The noun "policy" and the preposition "on" combine to form
an appropriate idiom.

Explanation for Incorrect Answer D :


There is no error at (D). The verb "caused" idiomatically introduces the effect of the
management's action.

Explanation for Incorrect Answer E : There is an error in the sentence.

re

(1) Some of the worlds greatest scientists have been women, and most
people still tend to think of science as a mans game. (2) There are
probably many reasons that more men than women had fame as
scientists. (3) Unequal access to educational opportunities is certainly one.

te

(4) But sometimes the reason is plain old-fashioned dishonesty. (5) James
Watson, Francis Crick, and Maurice Wilkins were awarded the Nobel Prize for the
discovery by them of the double helix structure of the DNA molecule. (6) The
discovery is seen by most as one of the greatest contributions to the modern
history of biology.

is

nR
eg

(7) One of the most important pieces of evidence used by Watson and Crick to
figure out this structure was an x-ray diffraction photograph that had been
taken by a woman, Rosalind Franklin. (8) Scientists often build on the work of
other scientists, but they usually do so openly. (9) Franklins photograph was
secretly shown to Watson by her colleague Maurice Wilkins. (10) Who never
told her what he had done. (11) And then Watson, Crick, and Wilkins gave
Nobel Prize lectures that contained 98 references to the work of other scientists,
not citing a single one of Franklins papers. (12) Of them only Wilkins in his
speech making even a casual reference to her when he said she made some
very valuable contributions to the x-ray analysis.

U
30

In context, which of the following is the best change to make to sentence 1 ?

(A)

Insert As one can see at the beginning.

(B)

Insert In the field of genetics at the beginning.

(C) Insert of course after and.

(D) Insert yet after and.

(E)

Delete the quotation marks.

ANSWERS

AND EXPLANATIONS

Explanation for Correct Answer D :


Choice (D) is correct. The addition of the adverb yet establishes the necessary
contrast between clauses.

Explanation for Incorrect Answer A :


Choice (A) is unsatisfactory because the first sentence of the passage does not
require the addition of such a transitional phrase.

file://E:\\b6.htm

2006-11-12

18/21

The Official SAT Online Course

Explanation for Incorrect Answer B :


Choice (B) is unsatisfactory because it changes the meaning of the original
sentence, narrowing the role of women from all sciences to the field of genetics.

Explanation for Incorrect Answer C :


Choice (C) is unsatisfactory because it does not provide the necessary contrast
between clauses and because it adds an unnecessary phrase.

Explanation for Incorrect Answer E :


Choice (E) is unsatisfactory because the phrase a mans game is colloquial
speech, quoted by the writer.

31

What is the best way to deal with sentence 2?


(A)

Omit it.

(B)

Switch it with sentence 1.

(C) Insert Definitely at the beginning.

(D) Change many to lots of.

Change had to have achieved.

ANSWERS

AND EXPLANATIONS

ed

(E)

Explanation for Correct Answer E :


Choice (E) is correct. It replaces the simple past tense verb, had, with the
completed past tense, have achieved.

is
te
r

eg

Explanation for Incorrect Answer A :


Choice (A) is unsatisfactory. Sentence 2 begins the discussion of a series of reasons
(continued in sentences 3 and 4) that women have not achieved as much fame in
science as men have.

Explanation for Incorrect Answer B :


Choice (B) is unsatisfactory because sentence 1 presents a set of circumstances,
and sentences 2, 3, and 4 offer possible reasons for those circumstances.

nR

Explanation for Incorrect Answer C :


Choice (C) is unsatisfactory because Definitely contradicts the word
probably later in the sentence.

Explanation for Incorrect Answer D :


Choice (D) is unsatisfactory because it introduces a colloquial phrase more general
than the original adjective.

32

Which of the following sentences is best inserted after sentence 3?

(A)

They think of science as a field in which men have been traditionally


encouraged to participate.

(B)

The failure of the educational system to nurture young girls interest in


science is certainly another.

(C) Some of the best-known names in science are those of men such as
Galileo and Einstein.

(D) The girls in my school are not given the same opportunities to study
scientific subjects as the boys are.

(E)

Yet Rosalind Franklin was a woman who made a number of extremely


significant contributions to modern science.

ANSWERS

AND EXPLANATIONS

Explanation for Correct Answer B :


Choice (B) is correct. The sentence extends the discussion of educational
opportunities in sentence 2 and provides an additional reason for mens greater

file://E:\\b6.htm

2006-11-12

19/21

The Official SAT Online Course

fame in science.

Explanation for Incorrect Answer A :


Choice (A) is unsatisfactory because the pronoun They is ambiguous and
because the sentence returns to the concerns of sentence 1 rather than building on
sentence 3.

Explanation for Incorrect Answer C :


Choice (C) is unsatisfactory because it makes an illogical leap, moving from a
mention of access to educational opportunities in sentence 3 to a sudden
presentation of the names of great scientists.

Explanation for Incorrect Answer D :


Choice (D) is unsatisfactory because it moves to a discussion of present conditions
in school instead of providing background information about conditions hindering
female scientists from achieving fame.

Explanation for Incorrect Answer E :


Choice (E) is unsatisfactory because it moves illogically to defend the contributions
of Rosalind Franklin, though the writer has not yet introduced her.

In context, which of the following is the best way to express the underlined portion of
sentence 5 (reproduced below) ?

ed

33

James Watson, Francis Crick, and Maurice Wilkins were awarded the Nobel Prize for
the discovery by them of the double helix structure of the DNA molecule.

te
r

(A)

(As it is now)

(B)

They were awarded the Nobel Prize for the discovery

eg
is

(C) Watson, Crick, and Wilkins were awarded the Nobel Prize for the discovery
by them

(D) James Watson, Francis Crick, and Maurice Wilkins were awarded the Nobel
Prize for their discovery

(E)

But the Nobel Prize was awarded to Watson, Crick, and Wilkins for the
discovery

nR

ANSWERS

AND EXPLANATIONS

Explanation for Correct Answer D :


Choice (D) is correct. It replaces the wordy phrase "the discovery by them" with the
more concise "their discovery."

Explanation for Incorrect Answer A :


Choice (A) is unsatisfactory because the phrase "by them" is repetitive and
unnecessary.

Explanation for Incorrect Answer B :


Choice (B) is unsatisfactory because the three scientists have not been named up to
this point in the essay; hence, there is no one to whom "They" could logically refer.

Explanation for Incorrect Answer C :


Choice (C) is unsatisfactory. Since Watson, Crick, and Wilkins have not been named
previously, their first names should be given; also, the phrase "by them" is
repetitive.

Explanation for Incorrect Answer E :


Choice (E) is unsatisfactory because the word "But" indicates that what follows will
contrast with what came before. Instead, sentence 5 introduces an example
illustrating the idea expressed in sentence 4.

34

file://E:\\b6.htm

In context, which of the following is the best version of sentences 9 and 10

2006-11-12

20/21

The Official SAT Online Course

(reproduced below) ?

Franklins photograph was secretly shown to Watson by her colleague Maurice


Wilkins. Who never told her what he had done.
(A)

(As it is now)

(B)

Similarly, her colleague Maurice Wilkins showed Franklins photograph to


Watson, and he never told her about it.

(C) Unfortunately, she did not know that her colleague Maurice Wilkins had
secretly shown this photograph to Watson.

(D) In fact, Franklins colleague Maurice Wilkins never told her that he had
shown Watson the photograph.

(E)

In this case, Franklins colleague Maurice Wilkins secretly showed Watson


her photograph without telling her.

ANSWERS

AND EXPLANATIONS

Explanation for Correct Answer E :


Choice (E) is correct. The phrase "In this case" alerts the reader that what follows
may be an exception to the general rule that has just been described in sentence 8.

re

Explanation for Incorrect Answer A :


Choice (A) is unsatisfactory because sentence 10 is a fragment: it is grammatically
incomplete because it is a subordinate clause.

te

Explanation for Incorrect Answer B :


Choice (B) is unsatisfactory because the word "Similarly" indicates that what follows
will agree with what has just been said in sentence 8; in fact, the writer is
describing an exception to the rule stated in sentence 8.

Explanation for Incorrect Answer C :


Choice (C) is unsatisfactory because it illogically places the emphasis on Franklin's
misfortune rather than on Wilkins' dishonesty, which is the overall subject of the
passage.

is

eg

Explanation for Incorrect Answer D :


Choice (D) is unsatisfactory because the sequence of events is confusing. It
describes Wilkins' failure to tell Franklin before mentioning that he showed her
photograph to Watson, but it would be more logical to describe showing the
photograph first.

nR

U
35

Which of the following is the best way to express the underlined portion of sentence
12 (reprinted below) ?

Of them only Wilkins in his speech making even a casual reference to her when he
said she made some very valuable contributions to the x-ray analysis.

(A)

(As it is now)

(B)

Having made his speech, only Wilkins had made even

(C) Of the three, only Wilkins made even

(D) Only his speech contained even

(E)

In the same manner, Wilkins made

ANSWERS

AND EXPLANATIONS

Explanation for Correct Answer C :


Choice (C) is correct. The phrase "Of the three" provides a concise link to the
previous sentence; also, the sentence has an appropriate main verb, "made."

Explanation for Incorrect Answer A :

file://E:\\b6.htm

2006-11-12

21/21

The Official SAT Online Course


Choice (A) is unsatisfactory because it is a fragment: the sentence has no main
verb. Also, the pronoun "them" is ambiguous; it is not clear to which of the many
plural nouns in the previous sentence it refers.

Explanation for Incorrect Answer B :


Choice (B) is unsatisfactory because the phrase "Having made his speech"
unnecessarily repeats information from sentence 11; also, the pluperfect tense of
the verb, "had made," would be more logical if it were changed to the past tense,
"made."

Explanation for Incorrect Answer D :


Choice (D) is unsatisfactory because it is not clear whether the pronoun "his" refers
to Watson, Crick, or Wilkins.

Explanation for Incorrect Answer E :


Choice (E) is unsatisfactory because the phrase "In the same manner" is illogical;
the sentence is describing something in Wilkins's speech that was different from the
other two speeches.

Privacy Policy

Terms of Use

Contact Us

er
e

Copyright 2006 The College Board. All rights reserved.

Back to Score Report

st

eg
i

nR

file://E:\\b6.htm

2006-11-12

1/21

The Official SAT Online Course

Help | Profile | My Organizer | My Bookmarks | Logout

Answers and Explanations

Back to Score Report

Test Sections

Section 1

View Answers and Explanations

Section 2

Online - Practice Test #2

Section 3

Section 4

New data measuring the ------- of land beneath the oceans permit accurate
generalizations about the topography of the seafloor.

Section 6

Section 7

Section 8

(A)

models

(B)

contours

Section 9

(C) remnants

Section 10

(D) population

(E)

pigments

AND EXPLANATIONS

ANSWERS

re

Explanation for Correct Answer B :


Choice (B) is correct. "Contours" are the outlines or shapes of curving or irregular
surfaces. If one were to insert this term into the text, the sentence would read
"New data measuring the contours of land beneath the oceans permit accurate
generalizations about the topography of the seafloor." The missing term has a
meaning similar to "topography," or the surface features of a place or region. The
"contours of land" are the outlines or shape of the land's surface.

te

eg
is

Explanation for Incorrect Answer A :


Choice (A) is incorrect. "Models" are miniature representations of something. If one
were to insert this term into the text, the sentence would read "New data
measuring the models of land beneath the oceans permit accurate generalizations
about the topography of the seafloor." The actual land beneath the oceans, not
miniature representations of the land, must be measured in order to make accurate
generalizations about the seafloor.

nR

Explanation for Incorrect Answer C :


Choice (C) is incorrect. "Remnants" are remains or things left over. If one were to
insert this term into the text, the sentence would read "New data measuring the
remnants of land beneath the oceans permit accurate generalizations about the
topography of the seafloor." In this context, it does not make sense to speak of
remains of land on the ocean floor.

Explanation for Incorrect Answer D :


Choice (D) is incorrect. A "population" is the total number of people or organisms in
a given area. If one were to insert this term into the text, the sentence would read
"New data measuring the populations of land beneath the oceans permit accurate
generalizations about the topography of the seafloor." The land beneath the oceans
may have a "population" of organisms, but knowing the number of these organisms
would not assist in making generalizations about the seafloor's "topography," or
surface features.

Explanation for Incorrect Answer E :


Choice (E) is incorrect. "Pigments" are substances used for coloring. If one were to
insert this term into the text, the sentence would read "New data measuring the
pigments of land beneath the oceans permit accurate generalizations about the
topography of the seafloor." Colors are not among the surface features typically
referred to by the term "topography."

file://E:\\b7.htm

Excessive secrecy tends to ------- excessive curiosity and thus serves to ------- the
very impulses against which it guards.

2006-11-12

2/21

The Official SAT Online Course

(A)

inhibit . . protect

(B)

disguise . . supplant

(C) satisfy . . limit

(D) compel . . deride

(E)

invite . . provoke

ANSWERS

AND EXPLANATIONS

Explanation for Correct Answer E :


Choice (E) is correct. "To invite" means to encourage. "To provoke" means to give
rise to. If one were to insert these terms into the text, the sentence would read
"Excessive secrecy tends to invite excessive curiosity and thus serves to provoke
the very impulses against which it guards." This sentence makes sense because
excessive secrecy encourages excessive curiosity. Secrecy thus gives rise to "the
very impulses" that the overly secretive seek to avoid--namely, people's desires to
find out what is going on.

re

Explanation for Incorrect Answer A :


Choice (A) is incorrect. "To inhibit" means to prevent. "To protect" means to guard.
If one were to insert these terms into the text, the sentence would read "Excessive
secrecy tends to inhibit excessive curiosity and thus serves to protect the very
impulses against which it guards." Excessive secrecy does not prevent excessive
curiosity. In fact, it does exactly the opposite.

Explanation for Incorrect Answer B :


Choice (B) is incorrect. "To disguise" means to conceal an identity. "To supplant"
means to oust and take the place of. If one were to insert these terms into the text,
the sentence would read "Excessive secrecy tends to disguise excessive curiosity
and thus serves to supplant the very impulses against which it guards." Excessive
secrecy does not conceal the identity of excessive curiosity. Neither does excessive
secrecy oust and take the place of curious interest.

te

eg
is

Explanation for Incorrect Answer C :


Choice (C) is incorrect. "To satisfy" means to meet expectations. "To limit" means
to restrict. If one were to insert these terms into the text, the sentence would read
"Excessive secrecy tends to satisfy excessive curiosity and thus serves to limit the
very impulses against which it guards." Excessive secrecy does not meet the
expectations of excessive curiosity, nor does it restrict curious interest.

nR

Explanation for Incorrect Answer D :


Choice (D) is incorrect. "To compel" means to force. "To deride" means to mock. If
one were to insert these terms into the text, the sentence would read "Excessive
secrecy tends to compel excessive curiosity and thus serves to deride the very
impulses against which it guards." Excessive secrecy may encourage excessive
curiosity, but it does not force it.

In frigid regions a layer of permafrost under the soil surface prevents water from
sinking deep into the soil, and so the water ------- the land, helping to create bog and
------- conditions.
(A)

freezes . . tropical

(B)

parches . . marsh

(C) inundates . . desert

(D) aerates . . jungle

(E)

floods . . swamp

ANSWERS

AND EXPLANATIONS

Explanation for Correct Answer E :


Choice (E) is correct. "Floods" means covers with water. A "swamp" is a watersoaked patch of land. If one were to insert these terms into the text, the sentence
would read "In frigid regions a layer of permafrost under the soil surface prevents
water from sinking deep into the soil, and so the water floods the land, helping to

file://E:\\b7.htm

2006-11-12

3/21

The Official SAT Online Course

create bog and swamp conditions." The first missing term describes what water
does when it cannot sink into soil, and the second missing term describes a
consequence of that situation. A piece of land "floods" when water cannot sink into
it, and this creates watery areas, or "swamps."

Explanation for Incorrect Answer A :


Choice (A) is incorrect. "Freezes" means turns to ice. "Tropical" refers to a hot
region or hot conditions. If one were to insert these terms into the text, the
sentence would read "In frigid regions a layer of permafrost under the soil surface
prevents water from sinking deep into the soil, and so the water freezes the land,
helping to create bog and tropical conditions." It is possible that water might turn to
ice atop the cold region's permafrost, but this would not cause hot, "tropical"
conditions.

Explanation for Incorrect Answer B :


Choice (B) is incorrect. "Parches" means dries out. A "marsh" is a patch of watery
ground. If one were to insert these terms into the text, the sentence would read "In
frigid regions a layer of permafrost under the soil surface prevents water from
sinking deep into the soil, and so the water parches the land, helping to create bog
and marsh conditions." A piece of land gets very wet when water cannot sink into it.
This land could not be called "parched." Furthermore, dried out land could not be
considered a "marsh," which is a patch of watery ground.

ed

Explanation for Incorrect Answer C :


Choice (C) is incorrect. "Inundates" means covers with water. A "desert" is an
extremely dry region. If one were to insert these terms into the text, the sentence
would read "In frigid regions a layer of permafrost under the soil surface prevents
water from sinking deep into the soil, and so the water inundates the land, helping
to create bog and desert conditions." A piece of land gets very wet when water
cannot sink through it. This is the opposite of "desert conditions."

er

is
t

Explanation for Incorrect Answer D :


Choice (D) is incorrect. "Aerates" means adds air to. A "jungle" is an overgrown
forest. If one were to insert these terms into the text, the sentence would read "In
frigid regions a layer of permafrost under the soil surface prevents water from
sinking deep into the soil, and so the water aerates the land, helping to create bog
and jungle conditions." A piece of land gets very wet when water cannot sink
through it. Water does not "aerate" land, or expose land to air. Furthermore, an
overgrown forest would probably not occur in "frigid," or cold, climates.

eg

nR

Although the bystanders account of the car accident at first seemed -------, the
police officer was surprised, on further investigation, to find that it was -------.

(A)

dubious . . erroneous

(B)

incongruous . . inconsistent

(C) implausible . . correct

(D) logical . . pertinent

(E)

probable . . coherent

ANSWERS

AND EXPLANATIONS

Explanation for Correct Answer C :


Choice (C) is correct. "Implausible" means hard to believe. "Correct" means
accurate. If one were to insert these words into the text, the sentence would read
"Although the bystander's account of the car accident at first seemed implausible,
the police officer was surprised, on further investigation, to find that it was correct."
The word "although" signals that the first missing term will contrast with the second
missing term. The police officer would be surprised if a story that at first seemed
hard to believe later proved to be correct.

Explanation for Incorrect Answer A :


Choice (A) is incorrect. "Dubious" means doubtful. "Erroneous" means full of error.
If one were to insert these words into the text, the sentence would read "Although
the bystander's account of the car accident at first seemed dubious, the police

file://E:\\b7.htm

2006-11-12

4/21

The Official SAT Online Course

officer was surprised, on further investigation, to find that it was erroneous." The
officer would hardly have been surprised had a doubtful account turned out to be
full of errors. The sentence is illogical because the contrast signaled by the word
"although" does not occur.

Explanation for Incorrect Answer B :


Choice (B) is incorrect. "Incongruous" means illogical. "Inconsistent" means
contradictory. If one were to insert these words into the text, the sentence would
read "Although the bystander's account of the car accident at first seemed
incongruous, the police officer was surprised, on further investigation, to find that it
was inconsistent." The police officer would not be surprised if a story that at first
seemed illogical later proved to be contradictory. The contrast signaled by the word
"although" does not occur, and, as a result, the sentence does not make sense.

Explanation for Incorrect Answer D :


Choice (D) is incorrect. "Logical" means rational or making sense. "Pertinent" means
relevant. If one were to insert these words into the text, the sentence would read
"Although the bystander's account of the car accident at first seemed logical, the
police officer was surprised, on further investigation, to find that it was pertinent."
The sentence does not make sense because there is no connection between a story
being rational and a story being relevant.

Explanation for Incorrect Answer E :


Choice (E) is incorrect. "Probable" means likely. "Coherent" means logical. If one
were to insert these words into the text, the sentence would read "Although the
bystander's account of the car accident at first seemed probable, the police officer
was surprised, on further investigation, to find that it was coherent." The police
officer would not be surprised if a story that at first seemed likely later proved to be
logical. The sentence is illogical because the contrast signaled by the word
"although" does not occur.

ed

is
te
r

The legislation facing Congress was so ------- that it threatened to shatter the
governing bodys fragile bipartisanship.
(A)

divisive

(B)

transparent

(C) concordant

eg

(D) repetitive

rhetorical

nR

(E)

ANSWERS

AND EXPLANATIONS

Explanation for Correct Answer A :


Choice (A) is correct. "Divisive" means creating division. If one were to insert this
term into the text, the sentence would read "The legislation facing Congress was so
divisive that it threatened to shatter the governing body's fragile bipartisanship."
The missing term describes the nature of a piece of legislation that threatened to
break up a relationship between two political parties. Therefore, by definition,
"divisive legislation" will tend to divide people and disrupt bipartisan political action.

Explanation for Incorrect Answer B :


Choice (B) is incorrect. "Transparent" means easily understood. If one were to
insert this term into the text, the sentence would read "The legislation facing
Congress was so transparent that it threatened to shatter the governing body's
fragile bipartisanship." Legislation that can be easily understood does not
necessarily threaten a relationship between political parties.

Explanation for Incorrect Answer C :


Choice (C) is incorrect. "Concordant" means harmonious or agreeing. If one were to
insert this term into the text, the sentence would read "The legislation facing
Congress was so concordant that it threatened to shatter the governing body's
fragile bipartisanship." Legislation that is harmonious would not suggest a threat to
a relationship between political parties.

Explanation for Incorrect Answer D :


Choice (D) is incorrect. "Repetitive" means repeating unnecessarily. If one were to
insert this term into the text, the sentence would read "The legislation facing
Congress was so repetitive that it threatened to shatter the governing body's fragile

file://E:\\b7.htm

2006-11-12

5/21

The Official SAT Online Course

bipartisanship." "Repetitive" legislation would not necessarily affect the governing


body's bipartisanship.

Explanation for Incorrect Answer E :


Choice (E) is incorrect. "Rhetorical" refers to saying or asking something for
persuasive effect. If one were to insert this term into the text, the sentence would
read "The legislation facing Congress was so rhetorical that it threatened to shatter
the governing body's fragile bipartisanship." Legislation that states something for
effect would not necessarily suggest a threat to a relationship between political
parties.

In All Gods Children Need Traveling Shoes, author Maya Angelou uses -------, brief
descriptive sketches, to provide ------- view of Ghana that clearly details the land and
its people.
(A)

missives. . an illusory

(B)

themes. . a thorough

(C) vignettes. . a vivid

(D) treatises. . an authentic

(E)

abstracts. . an ambiguous

AND EXPLANATIONS

ANSWERS

re

Explanation for Correct Answer C :


Choice (C) is correct. "Vignettes" are short, descriptive literary sketches. "Vivid"
means lively or intensely clear. If one were to insert these terms into the text, the
sentence would read "In All God's Children Need Traveling Shoes, author Maya
Angelou uses vignettes, brief descriptive sketches, to provide a vivid view of Ghana
that clearly details the land and its people." This sentence makes sense because
"vignettes" are brief descriptive sketches, and "vivid" sketches clearly describe a
subject.

te

eg
is

Explanation for Incorrect Answer A :


Choice (A) is incorrect. "Missives" are letters. "Illusory" means deceptive. If one
were to insert these terms into the text, the sentence would read "In All God's
Children Need Traveling Shoes, author Maya Angelou uses missives, brief
descriptive sketches, to provide an illusory view of Ghana that clearly details the
land and its people." A letter is not a brief descriptive sketch. Also, a deceptive view
of Ghana would not clearly describe the land and its people.

nR

Explanation for Incorrect Answer B :


Choice (B) is incorrect. "Themes" are main ideas. "Thorough" means complete. If
one were to insert these terms into the text, the sentence would read "In All God's
Children Need Traveling Shoes, author Maya Angelou uses themes, brief descriptive
sketches, to provide a thorough view of Ghana that clearly details the land and its
people." Since all books have main ideas, one does not need to specify that author
Maya Angelou uses them.

Explanation for Incorrect Answer D :


Choice (D) is incorrect. "Treatises" are typically lengthy, complex, and often
analytical pieces of writing. "Authentic" means real or factually correct. If one were
to insert these terms into the text, the sentence would read "In All God's Children
Need Traveling Shoes, author Maya Angelou uses treatises, brief descriptive
sketches, to provide an authentic view of Ghana that clearly details the land and its
people." A "treatise" is not a brief sketch.

Explanation for Incorrect Answer E :


Choice (E) is incorrect. "Abstracts" are summaries of main points. "Ambiguous"
means uncertain or indistinct. If one were to insert these terms into the text, the
sentence would read "In All God's Children Need Traveling Shoes, author Maya
Angelou uses abstracts, brief descriptive sketches, to provide an ambiguous view of
Ghana that clearly details the land and its people." An "ambiguous" view of Ghana
would not offer clear details about the country's land and people.

file://E:\\b7.htm

Because an older horse is more ------- than a younger one, it is safer for a novice

2006-11-12

6/21

The Official SAT Online Course

rider.
(A)

frolicsome

(B)

cantankerous

(C) gargantuan

(D) tractable

(E)

precipitate

ANSWERS

AND EXPLANATIONS

Explanation for Correct Answer D :


Choice (D) is correct. "Tractable" means easily handled or controlled. If one were to
insert this term into the text, the sentence would read "Because an older horse is
more tractable than a younger one, it is safer for a novice rider." The missing term
refers to the quality of a horse that will make it safe for inexperienced riders. Most
likely, an older horse is easier to handle or control than a younger horse and is
therefore a good choice for a "novice," or inexperienced, rider.

re

Explanation for Incorrect Answer A :


Choice (A) is incorrect. "Frolicsome" means playful. If one were to insert this term
into the text, the sentence would read "Because an older horse is more frolicsome
than a younger one, it is safer for a novice rider." An older horse is usually less, not
more, playful than a younger horse. In addition, the energy of a playful horse might
be unsafe for a "novice," or inexperienced, rider.

Explanation for Incorrect Answer B :


Choice (B) is incorrect. "Cantankerous" means difficult to deal with. If one were to
insert this term into the text, the sentence would read "Because an older horse is
more cantankerous than a younger one, it is safer for a novice rider." A horse that
is difficult to deal with might be dangerous for a "novice," or inexperienced, rider.

te

eg
is

Explanation for Incorrect Answer C :


Choice (C) is incorrect. "Gargantuan" means extremely large. If one were to insert
this term into the text, the sentence would read "Because an older horse is more
gargantuan than a younger one, it is safer for a novice rider." This would not make
sense because the size of a horse is not always correlated with its age.
Furthermore, an extremely large horse would not necessarily be safer than a
smaller horse.

nR

Explanation for Incorrect Answer E :


Choice (E) is incorrect. "Precipitate" means being hasty and impulsive. If one were
to insert this term into the text, the sentence would read "Because an older horse is
more precipitate than a younger one, it is safer for a novice rider." A horse that is
impulsive would be extremely unsafe for a "novice," or inexperienced, rider.

The librarys collection is a ------- of Asian American historical documents, including


rare materials about race relations.
(A)

summary

(B)

fabrication

(C) consensus

(D) trove

(E)

replication

ANSWERS

AND EXPLANATIONS

Explanation for Correct Answer D :


Choice (D) is correct. A "trove" is a valuable collection. If one were to insert this
term into the text, the sentence would read "The library's collection is a trove of
Asian American historical documents, including rare materials about race relations."
The missing term is characterized by the phrase that follows the comma. A library
collection containing rare materials is a valuable collection.

file://E:\\b7.htm

2006-11-12

7/21

The Official SAT Online Course

Explanation for Incorrect Answer A :


Choice (A) is incorrect. A "summary" is a review of the main points of something. If
one were to insert this term into the text, the sentence would read "The library's
collection is a summary of Asian American historical documents, including rare
materials about race relations." A library collection, which consists of many diverse
documents, is not a "summary."

Explanation for Incorrect Answer B :


Choice (B) is incorrect. A "fabrication" is something made up. If one were to insert
this term into the text, the sentence would read "The library's collection is a
fabrication of Asian American historical documents, including rare materials about
race relations." A library collection includes real historical documents and thus is
not a "fabrication."

Explanation for Incorrect Answer C :


Choice (C) is incorrect. "Consensus" mean general agreement. If one were to insert
this term into the text, the sentence would read "The library's collection is a
consensus of Asian American historical documents, including rare materials about
race relations." The term "consensus" does not make sense in this context because
a library collection cannot be described as a "general agreement."

Explanation for Incorrect Answer E :


Choice (E) is incorrect. A "replication" is a copy. If one were to insert this term into
the text, the sentence would read "The library's collection is a replication of Asian
American historical documents, including rare materials about race relations."
These historical documents would be originals, not copies. A "replication" is a copy
of an original, not a collection of rare historical documents.

re

There you are, driving along the freeway,


nobody in
sight, just you, daydreaming, when RIGHT
BEHIND YOU
theres a blast of sound, as of some eighteenwheeler out
of control, howling in rage . . . . WHERE IS IT?
Line
Without
thinking, you see all directions at once: the
5
empty road
ahead and behind, the lack of vehicles on all
sides. . . .
Slowly, you come to understand that the sound
is not
from the road but from a highballing freight
train travelling
parallel to the freeway.
How long to go from full alert to anticlimax?
10
Less than a
minute. It was a chemical reaction, pure and
simple, and it
crippled time in your mind.

te

is

eg

nR

The capital letters in lines 2 and 4 primarily serve to evoke a sense of


(A)

anger

(B)

alarm

(C) impatience

(D) disbelief

(E)

file://E:\\b7.htm

embarrassment

2006-11-12

8/21

The Official SAT Online Course

ANSWERS

AND EXPLANATIONS

Explanation for Correct Answer B :


Choice (B) is correct. The capitalized words are intended to suggest the surprising
suddenness of the noise and convey the driver's sense of urgency in finding where
the noise is coming from. The driver is clearly startled and "alarmed" by the blast of
sound.

Explanation for Incorrect Answer A :


Choice (A) is incorrect. The passage does not mention the driver showing anger.
Rather the capitalized letters indicate the driver's alarm, or concern, over the blast
of sound.

Explanation for Incorrect Answer C :


Choice (C) is incorrect. The capitalized words relate to the driver's experience of a
loud, unknown sound. The driver is probably impatient to find where the sound is
coming from, but "impatience" is not a strong enough feeling to be expressed in
capital letters.

Explanation for Incorrect Answer D :


Choice (D) is incorrect. There is no question about the existence of the blast of
sound, so disbelief cannot apply.

re

Explanation for Incorrect Answer E :


Choice (E) is incorrect. The driver is not embarrassed by the sound, but startled
and worried.

is
te

10

If the passage were to continue, the next paragraph would most likely discuss

(A)

other qualities of freight trains that often cause anxiety in humans

(B)

the chemical reaction that occurs when drivers repeatedly honk their horns
in traffic

nR
eg

(C) the destination of the driver and the reason for the drivers haste

(D) the nature of the chemical reaction that occurs when people become
scared

(E)

important differences between the effects of truck horns and train whistles
on humans

ANSWERS

AND EXPLANATIONS

Explanation for Correct Answer D :


Choice (D) is correct. The last sentence of the passage is "it was a chemical
reaction, pure and simple, and it crippled time in your mind" (lines 11-12). The
logical next step for the passage would be to provide more detail about this
chemical reaction.

Explanation for Incorrect Answer A :


Choice (A) is incorrect. As is suggested by the last two sentences, the subject of the
passage is the nature of the driver's reaction. It is not trains or other loud vehicles.

Explanation for Incorrect Answer B :


Choice (B) is incorrect. The passage does not discuss the repeated honking of
horns.

Explanation for Incorrect Answer C :


Choice (C) is incorrect. Where the driver was going is irrelevant, and the passage
does not indicate that the driver was in a hurry.

Explanation for Incorrect Answer E :


Choice (E) is incorrect. Since the "chemical reaction" does not make distinctions
between types of noises, it is unlikely that the passage would go on to discuss
those distinctions.

file://E:\\b7.htm

2006-11-12

9/21

The Official SAT Online Course

When Dr. Mae C. Jemison blasted into orbit


aboard the
space shuttle Endeavour on September 12,
1992, she also
blasted into history as the first woman of color
to go into
space. A chemical engineer, scientist,
Line
physician, teacher,
and astronaut, Jemison has been undaunted by
5
a lack of
role models or by roadblocks to women and
minority
people. I had to learn very early not to limit
myself
due to others limited imagination, says Dr.
Jemison.
An advocate for science and technology,
Jemison maintains, we need to change the image of who
10
does science.
Thats important not only for folks who want
to go into
science, but for the folks who fund science.

re

te

is

The primary purpose of the passage is to


(A)

note obstacles facing women in science

(B)

explain how Jemison fosters interest in science

nR
eg

11

(C) discuss how Jemison rose to fame

(D) provide a sketch of Jemison and her goals

(E)

describe Jemisons introduction to science

ANSWERS

AND EXPLANATIONS

Explanation for Correct Answer D :


Choice (D) is correct. The passage's first sentence explains Jemison's primary
achievement: she "blasted into history as the first woman of color to go into
space" (lines 3-4). The remainder of the passage sums up Jemison's background
and her goals for science.

Explanation for Incorrect Answer A :


Choice (A) is incorrect. This passage describes the achievements and goals of a
particular individual who was undaunted by the obstacles before her. The primary
purpose of the passage is not to note the "obstacles facing women in science."

Explanation for Incorrect Answer B :


Choice (B) is incorrect. Although the passage does indicate that Jemison is an
advocate for science and technology, this is not the passage's central focus. In
addition, the passage does not explain how she "fosters," or encourages, interest in
science.

Explanation for Incorrect Answer C :


Choice (C) is incorrect. The idea of fame is not discussed in this passage.

Explanation for Incorrect Answer E :


Choice (E) is incorrect. The passage does not mention how Jemison first became

file://E:\\b7.htm

2006-11-12

10/21

The Official SAT Online Course

involved in science.

12

In lines 9-12, Jemison is represented as


(A)

a political ideologue

(B)

an arbitrator of disputes

(C) a mentor to women

(D) a charismatic visionary

(E)

a champion of reform

ANSWERS

AND EXPLANATIONS

Explanation for Correct Answer E :


Choice (E) is correct. The passage calls Jemison an "advocate for science and
technology" and quotes her as saying, "we need to change the image of who does
science" (lines 9-10). These lines indicate that Jemison would like to "reform," or
change, common perceptions about who can or should participate in science.

re

Explanation for Incorrect Answer A :


Choice (A) is incorrect. Jemison is concerned not with politics but with science.

Explanation for Incorrect Answer B :


Choice (B) is incorrect. Jemison shows no interest in "arbitrating," or refereeing, an
argument. Rather, she wants to expand the public's understanding of science and
technology, and change common perceptions people have of scientists.

is
te

Explanation for Incorrect Answer C :


Choice (C) is incorrect. Jemison would no doubt make a fine mentor, but the focus
of this passage is on her and her ideas about science.

Explanation for Incorrect Answer D :


Choice (D) is incorrect. Although Jemison clearly has many gifts, she is not
portrayed as having unusual personal magnetism or as being an idealist.

eg

nR

Mark Twain is the pseudonym of Samuel Clemens (1835-1910), who is best known for
his stories about life along the Mississippi River. He also lectured and read from his
writings in many countries around the world. The following passages are adapted from
two essays about Clemens published while Clemens was still alive.

Passage 1

While Mark Twain has declared that humor

is a
subject which has never had much interest
for him, it is
as a humorist that the world persists in
regarding him. It is
certain that Mark Twain is the greatest genius
Line
evolved by
natural selection out of the ranks of American
5
journalism.
Crude, rudimentary, and often coarse as much
of his writing
was, it bore upon it the fresh stamp of
contemporary

actuality. American humor, neither


file://E:\\b7.htm

2006-11-12

11/21

The Official SAT Online Course

10

15

unfathomably absurd
like the Irish, nor sharp and sensible and full of
the realities
of life like the Scottish, is simply the humor of
imagination.
It consists in piling towers on towers and
mountains on
mountains; of heaping a joke up to the stars
and extending
it to the end of the world.
Humor, it must be remembered, is a
function of
nationality. The same joke, as related by an
American,
a Scotsman, or an Irishman, carries with it a
distinctive
ethnic flavor and individuality of approach.
Indeed, it is
open to question whether most humor does not
require
some specialized knowledge of some particular
locality.
The secret of Mark Twains worldwide
popularity as a
humorist is not to be attributed to any tricks of
style, to
any breadth of knowledge, or even to any depth
of intellectuality. His humor has international range
because it is
constructed out of a deep comprehension of
human nature
and a profound sympathy for human
relationships and
human failings; thus, it successfully surmounts
the difficulties of translation into alien tongues. Above
all, he has
sympathized with and admired the citizens of
every nation,
seeking beneath the surface veneer the
universal traits of
that nations humanity. It is a matter of fact
that he has
made far more damaging admissions
concerning America
than concerning any other nation. My secret,
if there is a
secret, Twain has said, is to create humor
independent of
local conditions. Through studying humanity as
exhibited

te

20

re

is

eg

30

file://E:\\b7.htm

25

nR

2006-11-12

12/21

The Official SAT Online Course

35

in the people and localities I best knew and


understood, I
have sought to winnow out the encumbrance of
the local.
Humor, like morality, has its eternal verities.

Passage 2

40

Humor as a solid quality and a lucrative


trade is of
modern invention. The great men who dared to
laugh in
an earlier age than ours laughed in moderation
and with
a wise purpose. Aristophanes, Shakespeare,
and Chaucer
are the true humorists of our world. They did
not jest out
of season. Their humor is precious on account
of its parsimony. They do not at every turn slap their
readers on the
back and assure them that there is nothing
congruous in
the visible world. Of the irreverence that turns
whatever
is beautiful or noble into a stupid jest they
knew nothing.
They kept their humor in its proper place; they
used it for a
wise purpose; they did not degrade it to catch
an easy round
of applause; and, fortunately for them, they are
today refused
the august title of humorist, which sits so aptly
upon the
shoulders of Mark Twain.
The essence of humor is that it should be
unexpected.
The modern humorist is never unexpected. He
beats the
drum from the moment at which he appears
upon the stage.
Mark Twain brings whatever time has honored
down to
the level of a Yankee drummer. He finds every
custom
ridiculous that does not conform with the
standard of the
United States. He holds his sides when he
thinks of the
old masters. Nor does he understand that there
are certain

45

re

te

is

eg

nR

50

55

60

file://E:\\b7.htm

2006-11-12

13/21

The Official SAT Online Course

manifestations of genius which should be


sacred even for
the jester. In other words, Mark Twain the
humorist is a
bull in the china shop of ideas. When, as in A
Connecticut
Yankee in King Arthurs Court, he gave full
rein to his
fancy, he achieved such a masterpiece of
65
vulgarity as the
world has never seen. His book gives you the
same sort
of impression which you might receive from a
beautiful
picture over which a poisonous slug had
crawled. The hint
of magnificence is there, pitilessly deformed
and defaced.
And it is the more pitiful because he has a
70
talent which
stands in need of no folly for its embellishment.
Had he
never cut a joke, had he refrained always from
grinning at
grave and beautiful things, how brilliant a fame
would have
been his!

ed

te
r

is

eg

Which statement best describes how the authors of the two passages differ in their
views of Twains humor?

The author of Passage 1 criticizes its offensive style, whereas the author of
Passage 2 deplores its American bias.

nR

13

(A)

(B)

(C) The author of Passage 1 concludes that it is trivial, whereas the author of
Passage 2 concludes that it is harmless.

The author of Passage 1 views it as mediocre, whereas the author of


Passage 2 views it as intolerable.

(D) The author of Passage 1 praises its universality, whereas the author of
Passage 2 disparages its lack of discrimination.

(E)

The author of Passage 1 admires its vigor, whereas the author of Passage
2 considers it understated.

ANSWERS

AND EXPLANATIONS

Explanation for Correct Answer D :


Choice (D) is correct. The author of Passage 1 claims that Twain respects citizens
from every nation and describes "the universal traits of ... that nation's humanity."
The author of Passage 2 accuses Twain of a lack of discrimination, or good
judgment and taste, saying that Twain "brings whatever time has honored down to
the level of a Yankee drummer."

Explanation for Incorrect Answer A :


Choice (A) is incorrect. Although the author of Passage 2 does state that Twain
thinks the customs of other countries are ridiculous, there is no indication that the
author of Passage 1 finds Twain's humor offensive.

Explanation for Incorrect Answer B :


Choice (B) is incorrect. Although the author of Passage 2 finds Twain's humor highly
distasteful, the author of Passage 1, far from viewing Twain's humor as mediocre,

file://E:\\b7.htm

2006-11-12

14/21

The Official SAT Online Course

calls Twain "the greatest genius" that emerged out of American journalism.

Explanation for Incorrect Answer C :


Choice (C) is incorrect. The author of Passage 1 calls Twain a "genius" and mentions
his "worldwide popularity," so clearly this author would not describe Twain's humor
as trivial. The author of Passage 2 calls Twain "a bull in the china shop of ideas,"
suggesting that Twain is far from harmless.

Explanation for Incorrect Answer E :


Choice (E) is incorrect. The author of Passage 1 does appear to admire the vigor, or
liveliness, of Twain's humor. However, the author of Passage 2 contrasts Twain's
humor with the "moderation" of writers like Aristophanes, Shakespeare, and
Chaucer, suggesting that Twain's humor is the opposite of understated, or
restrained.

14

In line 23, Passage 1, range most nearly means


(A)

scope

(B)

distance

(C) variation

(D) ranking

value

ANSWERS

is
te
re
d

(E)

AND EXPLANATIONS

Explanation for Correct Answer A :


Choice (A) is correct. The term "range" is used to express the vast amount, or
scope, of people throughout the world who appreciate Twain's humor.

Explanation for Incorrect Answer B :


Choice (B) is incorrect. "Distance" gives the sense of how far two points are from
each other. The use of the term "range" here is not to describe the distance
between Twain and the people his humor touches, but to indicate the vast amount
of different people who understand his jokes.

eg

nR

Explanation for Incorrect Answer C :


Choice (C) is incorrect. Although it could make sense to say that Twain's humor
"has international variation," the end of the sentence suggests that the humor has
a universal quality that does not depend on variations among people.

Explanation for Incorrect Answer D :


Choice (D) is incorrect. The reason given for the humor's "international range" does
not suggest that the humor has been ranked, or judged in relation to others.

Explanation for Incorrect Answer E :


Choice (E) is incorrect. Although the author does suggest that Twain's humor could
have international value, the term "range" refers to vast amount of people who
value Twain's humor, not the value itself.

15

In lines 23-27 (His humor . . . tongues), the author of Passage 1 attributes


Twains international popularity to his
(A)

knowledge of comedic style

(B)

intellectual breadth

(C) understanding of people

(D) mastery of foreign languages

(E)

reputation for appealing to ethnic humor

ANSWERS

AND EXPLANATIONS

Explanation for Correct Answer C :

file://E:\\b7.htm

2006-11-12

15/21

The Official SAT Online Course

Choice (C) is correct. The passage states that Twain's humor is based on "a deep
comprehension of human nature," or a profound understanding of what it means to
be human.

Explanation for Incorrect Answer A :


Choice (A) is incorrect. According to the sentence in lines 20-23, Twain's popularity,
or widespread public acclaim, is not the result of a tricky or adept use of literary
style.

Explanation for Incorrect Answer B :


Choice (B) is incorrect. In lines 20 to 21, the author denies that Twain's popularity,
or widespread public acclaim, results from "any breadth of knowledge, or . . . depth
of intellectuality."

Explanation for Incorrect Answer D :


Choice (D) is incorrect. In lines 26-27, the author claims that Twain's humor can be
translated easily into other languages, not that Twain himself was familiar with
those languages.

Explanation for Incorrect Answer E :


Choice (E) is incorrect. The author argues that Twain's humor is not ethnic or local,
but universal.

re

Which of the following most nearly captures the meaning of winnow . . . local
(line 36, Passage 1) ?

(A)

Intermix local details with universal truths

(B)

Take out that which prevents one from recognizing the universal

st
e

16

(C) Use the universal as a way to place the specific in context

(D) Devise new ways to express old truths

Reap the benefits of local customs so as to understand the universal

eg
i

(E)

ANSWERS

AND EXPLANATIONS

nR

Explanation for Correct Answer B :


Choice (B) is correct. Twain is saying that in order to "create humor independent of
local conditions," or to create a more universal humor, he removes the obstacles to
universal humor that local ideas present.

Explanation for Incorrect Answer A :


Choice (A) is incorrect. To "winnow out" means to get rid of something, not to mix
it with something else.

Explanation for Incorrect Answer C :


Choice (C) is incorrect. Instead of placing the specific, or local, in context, according
to the sentence, Twain seeks to eliminate the local altogether.

Explanation for Incorrect Answer D :


Choice (D) is incorrect. Twain is speaking of removing obstacles, not expressing
truth.

Explanation for Incorrect Answer E :


Choice (E) is incorrect. By the expression "winnow out," Twain means that he
ignores local customs when they hinder understanding universal truths, not that he
reaps the benefits of the local customs.

17

Which of the following, if true, would most seriously undermine the main argument
presented in Passage 1?
(A)

file://E:\\b7.htm

Humor is intrinsically connected to a sense of morality.

2006-11-12

16/21

The Official SAT Online Course

(B)

Certain cultural differences are so powerful that it is impossible for them


to be transcended.

(C) Humor is a function of imagination coupled with exaggeration.

(D) In order to be successful as a humorist, one must have specialized


knowledge of local customs.

(E)

Humor is based on perpetual truths.

ANSWERS

AND EXPLANATIONS

Explanation for Correct Answer B :


Choice (B) is correct. If some cultural differences could not be transcended, or risen
above, then the author's argument that looking beyond cultural differences is the
basis of Twain's humor would be seriously undermined.

Explanation for Incorrect Answer A :


Choice (A) is incorrect. The discovery of a link between humor and a sense of
morality would not necessarily undermine the argument that Twain's humor is
based on universal rather than local characteristics.

re

Explanation for Incorrect Answer C :


Choice (C) is incorrect. The combination of imagination and humor is precisely the
way the author defines American humor. If this were proven true, the argument
would be strengthened, not undermined.

Explanation for Incorrect Answer D :


Choice (D) is incorrect. The passage quotes Twain as saying that his experience and
knowledge of particular localities has enabled him to find universal humor, so the
idea that a humorist must have such knowledge would not undermine the author's
argument.

te

Explanation for Incorrect Answer E :


Choice (E) is incorrect. The author quotes Twain as agreeing that "[h]umor, like
morality, has its eternal verities," or perpetual truths. If this were proven true, the
argument would be strengthened, not undermined.

is

eg

nR

18

Twains style of humor is criticized in Passage 2 mainly because the author believes
that Twain

(A)

lacks the subtlety of the literary masters

(B)

ignores local customs

(C) confuses the standards of different nations

(D) attempts too hard to appear refined

(E)

is less amusing as a public speaker than as a writer

ANSWERS

AND EXPLANATIONS

Explanation for Correct Answer A :


Choice (A) is correct. Throughout all of Passage 2, the author argues that, unlike
"the true humorists," Aristophanes, Shakespeare, and Chaucer, Twain exhibits no
sense of restraint or subtlety in his humor. According to the author, because Twain
continually jokes at everything, keeping nothing sacred, his humor lacks the subtle
unexpected quality of the masters. The literary masters "laughed in moderation and
with a wise purpose," while Twain cannot refrain from "grinning at grave and
beautiful things."

Explanation for Incorrect Answer B :


Choice (B) is incorrect. The author of Passage 2, in fact, criticizes Twain for not
ignoring local customs. According to the author, Twain mocks any custom that does
not conform to the culture of the United States.

Explanation for Incorrect Answer C :

file://E:\\b7.htm

2006-11-12

17/21

The Official SAT Online Course

Choice (C) is incorrect. The author of Passage 2 accuses Twain of mocking all
nations other than the United States. The author does not claim that Twain confuses
the standards, rather that he inappropriately thinks that the standards of all nations
should conform to those of the United States.

Explanation for Incorrect Answer D :


Choice (D) is incorrect. Far from suggesting that Twain tries to seem refined,
Passage 2 implies that Twain's humor "turns whatever is beautiful or noble into a
stupid jest."

Explanation for Incorrect Answer E :


Choice (E) is incorrect. There is no indication in the passage that the author
distinguishes between Twain's performance as a public speaker and his performance
as a writer.

19

Which of the following terms would the author of Passage 2 most likely use to
describe Twain?
(A)

Aristocrat

(B)

Reformer

(C) Apologist

(D) Visionary

ANSWERS

AND EXPLANATIONS

Chauvinist

er
e

(E)

Explanation for Correct Answer E :


Choice (E) is correct. The statement in Passage 2 that Twain "finds every custom
ridiculous that does not conform with the standard of the United States" suggests
that Twain is a chauvinist, one who shows an extreme preference for a particular
group or place.

is
t

eg

Explanation for Incorrect Answer A :


Choice (A) is incorrect. The negative view of Twain's humor, for example, calling
one of Twain's works "a masterpiece of vulgarity," suggests that the author of
Passage 2 would not describe Twain as having the refined manners of an aristocrat.

nR

Explanation for Incorrect Answer B :


Choice (B) is incorrect. There is no evidence in the passage that the author would
describe Twain as a reformer, one who wishes to correct whatever is defective,
corrupt, or evil.

Explanation for Incorrect Answer C :


Choice (C) is not correct. The passage suggests that rather that being an apologist,
one who defends someone or something, Twain uses humor to attack and
undermine "whatever is beautiful or noble."

Explanation for Incorrect Answer D :


Choice (D) is incorrect. There is nothing in the passage that indicates whether the
author would describe Twain as a dreamer or a visionary, one whose ideas are
idealistic or impractical.

20

In the last sentence of Passage 2 (lines 71-74), the author indicates that Mark Twain
(A)

would have enjoyed fame despite his vulgarity and crudeness

(B)

would have been a better writer if he had not attempted humor

(C) would have enjoyed a brilliant career if he had perfected his comedic
technique

(D) was an amateur and a dilettante whose interest in humor was superficial

(E)

file://E:\\b7.htm

was destined for failure as a result of his insensitivity to his audience

2006-11-12

18/21

The Official SAT Online Course

ANSWERS

AND EXPLANATIONS

Explanation for Correct Answer B :


Choice (B) is correct. According to the last sentence in the passage, if Twain had
"never cut a joke," that is, if Twain had not tried to be humorous, he would have
achieved a "brilliant" fame as a writer.

Explanation for Incorrect Answer A :


Choice (A) is incorrect. Although the author of Passage 2 criticizes Twain for his
vulgarity earlier in the passage, the subject of the last sentence in the passage is
the effect the author believes that Twain's attempts at humor had on his work.

Explanation for Incorrect Answer C :


Choice (C) is incorrect. The last sentence in the passage indicates that the author
believes that Twain would have had to avoid comedy altogether in order to have
"brilliant" fame.

Explanation for Incorrect Answer D :


Choice (D) is incorrect. There is no indication in the sentence that the author saw
Twain as an amateur or a dilettante, one who has a superficial interest in the arts.

ed

Explanation for Incorrect Answer E :


Choice (E) is incorrect. There is no indication that the author of Passage 2 thinks of
Twain as likely to be a failure; nor is there any suggestion that Twain's own
audience thinks of him as insensitive.

er

Which of the following, if true, would best support the main argument presented in
Passage 2 ?

is
t

21

(A)

Literary greatness can sometimes be the subject of offensive satire.

(B)

Certain subjects are not easily satirized.

eg

(C) Intelligent men and women appreciate a good joke at their own expense
every now and then.

(D) Humorists are likely to be well received when they undertake to ridicule
respected writers of the past.

nR
(E)

Contemporary humorists are never as insightful as the great comic writers


of the past.

ANSWERS

AND EXPLANATIONS

Explanation for Correct Answer B :


Choice (B) is correct. The idea that certain subjects are not easily satirized, or held
up to ridicule, supports the argument that certain subjects should not be ridiculed.
As the author of Passage 2 argues, "there are certain manifestations of genius
which should be sacred even for the jester" but that modern humor, like that of
Twain, treats every subject with ridicule.

Explanation for Incorrect Answer A :


Choice (A) is incorrect. The idea that any sort of greatness can be subjected to
offensive satire is inconsistent with the argument that certain subjects should not
be ridiculed.

Explanation for Incorrect Answer C :


Choice (C) is incorrect. Whether intelligent people appreciate certain kinds of humor
is irrelevant to the argument that certain subjects should not be ridiculed.

Explanation for Incorrect Answer D :


Choice (D) is incorrect. The reception humorists are likely to receive is irrelevant to
the argument that certain subjects should not be ridiculed.

Explanation for Incorrect Answer E :


Choice (E) is incorrect. Although the author of Passage 2 does suggest that
contemporary humorists are less insightful than past writers, this idea is irrelevant

file://E:\\b7.htm

2006-11-12

19/21

The Official SAT Online Course

to the argument that certain subjects should not be ridiculed.

22

How would the author of Passage 1 most likely respond to the assertion in Passage 2
that Twain ridicules everything that does not conform with the standard of the
United States (lines 58-59) ?
(A)

Twain reserves his harshest criticism for America.

(B)

Twain is a gifted American journalist.

(C) Twain achieved international success because he is American.

(D) Twain attempts to create a uniquely American style of humor.

(E)

Twain is captivated by American culture.

ANSWERS

AND EXPLANATIONS

Explanation for Correct Answer A :


Choice (A) is correct. The author of Passage 1 claims that, in addition to showing
respect for others by having "sympathized with and admired the citizens of every
nation," Twain "has made far more damaging admissions concerning America than
concerning any other nation." It is clear that Twain's sympathy and respect for
other nations, combined with his willingness to criticize America, is evidence that he
reserves, or keeps, his harshest criticism for America.

re

eg
is
te

Explanation for Incorrect Answer B :


Choice (B) is incorrect. The author of Passage 1 does imply that Twain is a great
journalist, but this statement is not a response to the statement about Twain's use
of ridicule.

Explanation for Incorrect Answer C :


Choice (C) is incorrect. There is no indication in Passage 1 that the author believes
that Twain's popularity was due to his being American.

Explanation for Incorrect Answer D :


Choice (D) is incorrect. Unlike the author of Passage 2, the author of Passage 1
suggests that Twain's humor is not uniquely American but is based on "a deep
sympathy for human relationships and human failings."

nR

Explanation for Incorrect Answer E :


Choice (E) is incorrect. The idea that Twain is fascinated by American culture is not
suggested in Passage 1.

U
23

Which of the following pairs best represents each authors attitude toward the term
humorist, as used in Passage 1 (line 3) and Passage 2 (line 62) ?
Passage 1 / Passage 2
(A)

Amusement / Anger

(B)

Envy / Mockery

(C) Pride / Embarrassment

(D) Approval / Smugness

(E)

Respect / Disdain

ANSWERS

AND EXPLANATIONS

Explanation for Correct Answer E :


Choice (E) is correct. In Passage 1, the term "humorist" is applied to Twain by
others though Twain himself claims to be unconcerned with humor. In Passage 2,
the author portrays Twain the humorist as "a bull in the china shop of ideas,"
indicating a scornful view of Twain's humor. The respect felt by the author of
Passage 1 is reflected in the description of Twain as a "genius." The disdain, or
contempt, of the author of Passage 2 can be inferred from the negative remarks
made in comparing Twain with the earlier literay masters.

file://E:\\b7.htm

2006-11-12

20/21

The Official SAT Online Course

Explanation for Incorrect Answer A :


Choice (A) is incorrect. There is no evidence in Passage 1 that the author is amused
by the term. Although the author of Passage 2 is critical of Twain as a humorist,
"anger" would be too strong a word to describe the author's tone.

Explanation for Incorrect Answer B :


Choice (B) is incorrect. Although the tone of the discussion in Passage 2 could be
seen as mocking, there is no indication that the author of Passage 1 is envious of
Twain or other humorists.

Explanation for Incorrect Answer C :


Choice (C) is incorrect. There is no indication that the author of Passage 1 feels
pride with regard to the term, or that the author of Passage 2 feels embarrassment.

Explanation for Incorrect Answer D :


Choice (D) is incorrect. Though the author of Passage 1 seems to express approval
of Twain as a humorist, the author of Passage 2 expresses contempt for modern
humorists rather than the self-satisfaction that smugness suggests.

How would the author of Passage 2 most likely respond to the claim made by the
author of Passage 1 that Twain admired the citizens of every nation, seeking
beneath the surface veneer the universal traits of that nations humanity (lines
28-30)?

te
re
d

24

(A)

Twains humor is so unexpected that he is able to see startling aspects


about the behavior of people all over the world.

(B)

Twain shares with the humorists of the past the tendency to attack
foolishness whenever he sees it.

(C) Twain is limited by his inability to see things from anything other than an
American perspective.

(D) Twain always uses his humor for a wise purpose and uses humor to
emphasize the importance of people respecting each other.

is

The effect of Twains probing beneath the surface is limited by the fact
that he is not nearly as funny as many people claim he is.

eg

(E)

ANSWERS

AND EXPLANATIONS

nR

Explanation for Correct Answer C :


Choice (C) is correct. The author of Passage 2 claims that Twain "finds every
custom ridiculous that does not conform with the standard of the United States."
This suggests that Twain's ability to understand other people and their customs is
limited by his American perspective, or point of view.

Explanation for Incorrect Answer A :


Choice (A) is incorrect. The author of Passage 2 claims that "The modern humorist
[such as Twain] is never unexpected."

Explanation for Incorrect Answer B :


Choice (B) is incorrect. The author of Passage 2 suggests that Twain does not share
any characteristics with great humorists of the past.

Explanation for Incorrect Answer D :


Choice (D) is incorrect. The author of Passage 2 argues that Twain's humor has no
wisdom at all, and that it is largely characterized by irreverence, or disrespect.

Explanation for Incorrect Answer E :


Choice (E) is incorrect. The author of Passage 2 does not claim that Twain is not
funny, but rather that he never stops trying to be funny.

Back to Score Report

file://E:\\b7.htm

2006-11-12

21/21

The Official SAT Online Course

Privacy Policy

Copyright 2006 The College Board. All rights reserved.

Terms of Use

Contact Us

re

te

eg
is

nR

file://E:\\b7.htm

2006-11-12

1/13

The Official SAT Online Course

Help | Profile | My Organizer | My Bookmarks | Logout

Answers and Explanations

Back to Score Report

Test Sections

Section 1

View Answers and Explanations

Section 2

Online - Practice Test #2

Section 3

Section 4

Section 6

Section 7

words at City Newspaper. What is the least number


per
Classified ads cost
-word classified ad to reduce the
of words that must be deleted from the text of a
or less?
cost to

Section 8

(A)

Section 9

(B)

Section 10

(C)
(D)

AND EXPLANATIONS

Explanation for Correct Answer B :

re

ANSWERS

Choice (B) is correct. Since the ads cost

Therefore, at least

cost exactly

(E)

words,

from the text.

words need to be deleted

Explanation for Incorrect Answer A :


words from the text leaves
Choice (A) is not correct. Deleting

eg

words long costs exactly

an ad that is

words would

is
te

per

an ad that is

words. Since

words long

nR

costs more than

Explanation for Incorrect Answer C :


words from the text would reduce the cost to
Choice (C) is not correct. Deleting

or less, but

is the least number of words that need to deleted from the

or less.

text to reduce the cost to

Explanation for Incorrect Answer D :


words from the text would reduce the cost
Choice (D) is not correct. Deleting

is the least number of words that need to deleted from

or less, but

to

the text to reduce the cost to

or less.

Explanation for Incorrect Answer E :


words from the text would reduce the cost to
Choice (E) is not correct. Deleting

or less, but

is the least number of words that need to deleted from the

text to reduce the cost to

or less.

then

If
(A)
(B)
(C)

(D)

(E)

file://E:\\b8.htm

2006-11-12

2/13

The Official SAT Online Course

ANSWERS

AND EXPLANATIONS

Explanation for Correct Answer A :


then
Choice (A) is correct. If
and multiplying by
from both sides gives

Subtracting

gives

Explanation for Incorrect Answer B :


then substituting
Choice (B) is not correct. If
gives
which is a false statement.
simplifies to

Explanation for Incorrect Answer C :


then substituting
Choice (C) is not correct. If
gives
which is a false statement.
to

Explanation for Incorrect Answer D :


then substituting
Choice (D) is not correct. If
gives
which is a false statement.
to

in the equation
which

for

for

in the equation
which simplifies

for

in the equation
which simplifies

for

in the equation
which simplifies

re

Explanation for Incorrect Answer E :


then substituting
Choice (E) is not correct. If
gives
which is a false statement.
to

eg
is
te

nR

In the figure above, what is the value of

(A)

(B)

(C)

(D)

(E)

ANSWERS

AND EXPLANATIONS

Explanation for Correct Answer B :


Choice (B) is correct. For each of the right triangles, two of the three angle
measures are given. The fact that the degree measures of the angles of a triangle
and
means
add up to
Therefore,

Explanation for Incorrect Answer A :


Choice (A) is not correct. The sum of the three angles of the large triangle is
were
However, if
which should add up to
would equal only
then

Explanation for Incorrect Answer C :


Choice (C) is not correct. The sum of the three angles of the large triangle is
were
However, if
which should add up to
would equal
then the sum

file://E:\\b8.htm

2006-11-12

3/13

The Official SAT Online Course

Explanation for Incorrect Answer D :


Choice (D) is not correct. The sum of the three angles of the large triangle is
were
However, if
which should add up to
would equal
then the sum

Explanation for Incorrect Answer E :


Choice (E) is not correct. The sum of the three angles of the large triangle is
were
However, if
which should add up to
would equal
then the sum

which of the following is greater than

If
(A)

(B)

(C)

(D)

ANSWERS

re

(E)

AND EXPLANATIONS

Explanation for Correct Answer E :

te

and

are both positive, then

is positive. Therefore,

eg
is

Choice (E) is correct. If

is greater than

nR

Explanation for Incorrect Answer A :


are positive and
and
Choice (A) is not correct. Since

sides of the inequality by

so

gives

Explanation for Incorrect Answer B :


are positive and
and
Choice (B) is not correct. Since

sides of the inequality by

the inequality by

So,

Explanation for Incorrect Answer C :


are positive and
and
Choice (C) is not correct. Since

by

gives

dividing both

. On the other hand, dividing both sides of

gives

gives

sides of the inequality by

dividing both

while dividing both sides of the inequality

gives

So,

dividing both

Since

and

it follows that

Explanation for Incorrect Answer D :

Choice (D) is not correct. Since

file://E:\\b8.htm

and

are positive,

2006-11-12

4/13

The Official SAT Online Course

what is the value of

If
(A)
(B)
(C)
(D)
(E)

ANSWERS

AND EXPLANATIONS

Explanation for Correct Answer B :

so it follows that

Choice (B) is correct.

Explanation for Incorrect Answer A :

Choice (A) is not correct.

Explanation for Incorrect Answer C :

Explanation for Incorrect Answer D :

er

Choice (D) is not correct.

ed

Choice (C) is not correct.

Explanation for Incorrect Answer E :

st

Choice (E) is not correct.

eg
i

nR

(not shown) is another line in the plane, what is the


If
In the figure above,
can intersect these four lines?
least number of points at which

(A)

None

(B)

One

(C) Two

(D) Three

(E)

Four

ANSWERS

AND EXPLANATIONS

Explanation for Correct Answer C :


is parallel to lines
Choice (C) is correct. If line

intersect either

since lines

same), line

and

or

but

and

then

must then intersect both lines

intersect only at line

will intersect the lines

and

(and lines

and

will not

and

Moreover,

are not the

at two distinct points. If line

is

and
it must intersect each of the lines
and
not parallel to lines
can intersect these
Therefore, the least possible number of points at which line
four lines is two.

file://E:\\b8.htm

2006-11-12

5/13

The Official SAT Online Course

Explanation for Incorrect Answer A :


Choice (A) is not correct. The only way line

lines is if

could avoid intersecting any of these

were parallel to all four of them. However, only two of the four lines are

to be parallel to all four of the

parallel to each other, so it is not possible for


lines.

Explanation for Incorrect Answer B :


Choice (B) is not correct. The only way line

could avoid intersecting any three of

were parallel to three of them. However, only two of the four

these lines is if

to be parallel to three of

lines are parallel to each other, so it is not possible for


the lines.

Explanation for Incorrect Answer D :


can intersect the other
Choice (D) is not correct. While it is possible that line
lines at three points, two is also a possible number of intersections, and the
question asks for the least number.

Explanation for Incorrect Answer E :


can intersect the other lines
Choice (E) is not correct. While it is possible that line
at four points, two is also a possible number of intersections, and the question asks
for the least number.

which of the following must be true?

If

er
e

(A)
(B)

is
t

(C)
(D)

(E)

eg

ANSWERS

AND EXPLANATIONS

nR

Explanation for Correct Answer C :


This means
and
is between
then
Choice (C) is correct. If
This can be written using
is less than
and
that the distance between

or,

absolute value notation as

Explanation for Incorrect Answer A :

Choice (A) is not correct. If

then

but

which is

not less than

Explanation for Incorrect Answer B :

Choice (B) is not correct. If

then

but

which is not

greater than

Explanation for Incorrect Answer D :

Choice (D) is not correct. If

then

but

which is

not greater than

Explanation for Incorrect Answer E :

Choice (E) is not correct. If

then

but

which is not less than

file://E:\\b8.htm

2006-11-12

6/13

The Official SAT Online Course

for selected values of

The table above gives values of the linear function


of the following defines

Which

(A)
(B)

(C)
(D)

(E)

ANSWERS

AND EXPLANATIONS

Explanation for Correct Answer D :


Choice (D) is correct. For each value of

in the table. In the table,

equals the value of

formula

in the table, the value given by the

is always

ed

less than

is
t

Choice (A) is not correct. When

er

Explanation for Incorrect Answer A :

Explanation for Incorrect Answer B :

Choice (B) is not correct. When

eg

Explanation for Incorrect Answer C :

Choice (C) is not correct. When

nR

Explanation for Incorrect Answer E :

Choice (E) is not correct. When

U
9

and

is equal to the median of


and
The average (arithmetic mean) of
which of the following must be equal to
If
(A)
(B)
(C)
(D)
(E)

ANSWERS

AND EXPLANATIONS

Explanation for Correct Answer A :


the median of
Choice (A) is correct. Since
is equal to the median of
and
the average of

This simplifies to

file://E:\\b8.htm

Since
is
and
it follows that
and

and further to

and

2006-11-12

7/13

The Official SAT Online Course

finally to

Explanation for Incorrect Answer B :


Choice (B) is not correct. Since

then

that

This is impossible because

Explanation for Incorrect Answer C :


Choice (C) is not correct. Since

were also

This would imply

This is impossible because

that

Explanation for Incorrect Answer D :


Choice (D) is not correct. Since

would have to be equal to

re

then

equal to

This is impossible because

is
te

that

Explanation for Incorrect Answer E :


Choice (E) is not correct. Since

then

was equal to

impossible because

were also

This would imply

If

so

and

were also

This would imply that

would have to be equal to

eg

equal to

If

and the average and median of the

are equal, it follows that

and

three numbers

and the average and median of the

are equal, it follows that

and

three numbers

would be equal. This is

nR

10

If

would have to be equal to

then

equal to

and the average and median of the

are equal, it follows that

and

were also

This would imply

would have to be equal to

equal to

three numbers

If

are equal, it follows that

and

three numbers

and the average and median of the

exposures on a certain roll of film. The cost of the roll of film is


There are
cents per exposure. What is the cost, in
dollars, and the cost to develop the film is
dollars, to purchase and develop the roll of film?

(A)

(B)
(C)
(D)
(E)

ANSWERS

AND EXPLANATIONS

Explanation for Correct Answer A :


Choice (A) is correct. The cost to purchase and develop the roll of film is the sum of
dollars, and the cost to develop the film,
the cost to purchase the roll, which is
multiplied by the cost to develop each
which is the number of exposures

exposure of

file://E:\\b8.htm

dollars. Thus, the total cost, in dollars, is

2006-11-12

8/13

The Official SAT Online Course

Explanation for Incorrect Answer B :


Choice (B) is not correct. This expression interchanges the cost of the roll of film
and the cost per exposure to develop the film.

Explanation for Incorrect Answer C :


Choice (C) is not correct. This expression represents the product, not the sum, of
the cost of the roll of film and the cost to develop the film.

Explanation for Incorrect Answer D :


Choice (D) is not correct. This expression gives the cost, in cents, to purchase and
develop the roll of film.

Explanation for Incorrect Answer E :


and
Choice (E) is not correct. In this expression the roles and the units of
might convert from dollars to
have been interchanged. The multiplication by
cents, but the question asks for the cost in dollars.

11

A circular piece of cardboard is cut in half along a diameter. If the diameter is


inches, what is the perimeter, in inches, of one of the semicircular pieces?
(A)

(B)

(C)

re
d

(D)
(E)

ANSWERS

AND EXPLANATIONS

te

Explanation for Correct Answer B :


Choice (B) is correct. The perimeter of each of the semicircular pieces is made up of
two parts: the diameter of the original circle and half of the circumference of the
inches, the circumference of the original
original circle. Since the diameter is
inches. This means that the perimeter, in inches, of one of the
circle was

is

eg

semicircular pieces is

which simplifies to

nR

Explanation for Incorrect Answer A :


Choice (A) is not correct. The perimeter of a semicircular piece is made up of two
parts: the diameter of the original circle and half of the circumference of the
is not the full
is half the perimeter of the circle,
original circle. Though
measure of the diameter.

Explanation for Incorrect Answer C :


Choice (C) is not correct. The perimeter of a semicircular piece is made up of two
parts: the diameter of the original circle and half of the circumference of the
is only half of the
is the full circumference the circle while
original circle.
measure of the diameter.

Explanation for Incorrect Answer D :


Choice (D) is not correct. The perimeter of a semicircular piece is made up of two
parts: the diameter of the original circle and half of the circumference of the
is the full circumference of the
is the diameter,
original circle. Though
circle, instead of half of it.

Explanation for Incorrect Answer E :


Choice (E) is not correct. The question asks for the perimeter of only one of the
semicircular pieces, not the total perimeter for both.

12

file://E:\\b8.htm

2006-11-12

9/13

The Official SAT Online Course

The figure above shows the remaining pieces of a paper chain from which a section
has been removed. The first link of the original chain was red and the colors of the

yellow

links formed the repeating pattern of red

green

white

from left to right. Which of the following could be the number of links in

and blue

the section that was removed?


(A)
(B)
(C)
(D)
(E)

ANSWERS

AND EXPLANATIONS

Explanation for Correct Answer D :


Choice (D) is correct. The first piece of the chain ends with G, and for the chain to
get to Y from G, it needs to go through three colors W, B, and R. However, while
is a possibility for the number of missing links, it is not an answer choice. Since

the chain is set up so that the coloring system repeats itself every

number that is exactly

more than a multiple of

because

ed

answer choice that has this property is

links, any

is a possibility. The only

eg
is
te
r

Explanation for Incorrect Answer A :


Choice (A) is not correct. From the explanation for the correct answer, the number
does not
The number
more than a multiple of
of missing links must be

have that property, because

Explanation for Incorrect Answer B :


Choice (B) is not correct. From the explanation for the correct answer, the number
does not
The number
more than a multiple of
of missing links must be

have that property, because

nR

Explanation for Incorrect Answer C :


Choice (C) is not correct. From the explanation for the correct answer, the number
does not
The number
more than a multiple of
of missing links must be

have that property, because

Explanation for Incorrect Answer E :


Choice (E) is not correct. From the explanation for the correct answer, the number
does not
The number
more than a multiple of
of missing links must be

have that property, because

13

In the figure above, the length of

quadrilateral

is

is

and the length of each side of

What is the area of quadrilateral

(A)
(B)
(C)

file://E:\\b8.htm

2006-11-12

10/13

The Official SAT Online Course

(D)
(E)

ANSWERS

AND EXPLANATIONS

Explanation for Correct Answer D :


Choice (D) is correct. The area of quadrilateral

is equal to the sum of the

and the area of right triangle

area of quadrilateral

Quadrilateral

is a rhombus, and therefore a parallelogram, because all four sides are


is the
where
equal. The area of a parallelogram is given by the formula

is

so the area of

the base is

Right triangle

using the Pythagorean theorem to find the length of

has length

of

gives

Therefore,

is then

The area of

or

has

as its base. Since the hypotenuse,

as its height and the side

and

or

the height is

is the height. For

length of the base and

is

the total area of quadrilateral

Explanation for Incorrect Answer A :


Choice (A) is not correct. From the explanation for the correct answer, the length of
is not a rectangle
However, the figure
is
and the height
is

would require another piece congruent to

re

and height

(a rectangle of length

must be less than

), so the area of

Choice (B) is not correct. For the rhombus

is

so the area of

the base is

would have to be

then the area of

is
te

Explanation for Incorrect Answer B :

nR
eg

the height is

or

and

were

If the total area for

instead of

Explanation for Incorrect Answer C :

Choice (C) is not correct. For the rhombus

the base is

so the area of

then the area of

is

would have to be

the height is

or

and

were

If the total area for

instead of

Explanation for Incorrect Answer E :

Choice (E) is not correct. For the rhombus

the base is

so the area of

then the area of

is

would have to be

the height is

If the total area for

or

and

were

instead of

14

A list consists of all possible three-letter arrangements formed by using the letters
If no letter
and one of the remaining letters is
above such that the first letter is
is used more than once in an arrangement in the list and one three-letter
arrangement is randomly selected from the list, what is the probability that the
arrangement selected will be

(A)

(B)

(C)

file://E:\\b8.htm

2006-11-12

11/13

The Official SAT Online Course

(D)
(E)

ANSWERS

AND EXPLANATIONS

Explanation for Correct Answer D :


and
Choice (D) is correct. Since the three-letter arrangement must start with
there are 10 possible arrangements :
must contain the letter
So, the probability
and

being randomly selected is

of

Explanation for Incorrect Answer A :


Choice (A) is not correct. If there were only

three-letter arrangements meeting

from the list would be

probability of randomly selecting

re

Explanation for Incorrect Answer B :


Choice (B) is not correct. If there were only

three-letter arrangements meeting

was one of those

st
e

the conditions of the problem, and if

probability of randomly selecting

from the list would be

eg
i

Explanation for Incorrect Answer C :


Choice (C) is not correct. If there were only

nR

probability of randomly selecting

not

from the list would be

then the

However, there

arrangements meeting the conditions of the problem.

Explanation for Incorrect Answer E :


Choice (E) is not correct. If there were

However, there

three-letter arrangements meeting

was one of those

the conditions of the problem, and if

are

then the

arrangements meeting the conditions of the problem.

not

are

However, there

arrangements meeting the conditions of the problem.

not

are

then the

was one of those

the conditions of the problem, and if

conditions of the problem, and if

of randomly selecting

three-letter arrangements meeting the

was one of those

from the list would be

can be used more than once, there are only


the conditions of the problem.

not

then the probability

However, since no letter

arrangements meeting

15

The pyramid above has a square base and four congruent triangular faces. The height
inches long. What is the
inches, and each edge of the base is
of the pyramid is
in inches?
length of

file://E:\\b8.htm

2006-11-12

12/13

The Official SAT Online Course

(A)

(approximately

(B)

(approximately

(C)

(approximately

(D)

(approximately

(E)

(approximately

ANSWERS

AND EXPLANATIONS

Explanation for Correct Answer B :


Choice (B) is correct. Since the base of the pyramid is square, its two diagonals
intersect at the center of the square. The sides of the square have a length of

so

Therefore, the distance

the length of each diagonal of the square base is

Since the

from each vertex of the base to the center of the square base is

triangular faces of the pyramid are identical, the point at which they meet is

is the hypotenuse of a

directly above the center of the square base. Segment

to the center of the

the vertical line connecting

right triangle, composed of

The height of the pyramid


base, and the segment from the center of the base to
By the Pythagorean
so the vertical side of the triangle will have a length of
is

ed

theorem,

te
r

Explanation for Incorrect Answer A :


Choice (A) is not correct. See the explanation for the correct answer (B).

Explanation for Incorrect Answer C :


Choice (C) is not correct. See the explanation for the correct answer (B).

eg
is

Explanation for Incorrect Answer D :


Choice (D) is not correct. See the explanation for the correct answer (B).

Explanation for Incorrect Answer E :


Choice (E) is not correct. See the explanation for the correct answer (B).

nR

16

For all

let the function

Which of the following

be defined as

is equal to
(A)

(B)

(C)

(D)

(E)

ANSWERS

AND EXPLANATIONS

Explanation for Correct Answer B :

then

Choice (B) is correct. If

Because

we have

Explanation for Incorrect Answer A :

Choice (A) is not correct.

file://E:\\b8.htm

This is not equal to

2006-11-12

13/13

The Official SAT Online Course

Explanation for Incorrect Answer C :

This is not equal to

Choice (C) is not correct.

Explanation for Incorrect Answer D :

This is not equal to

Choice (D) is not correct.

Explanation for Incorrect Answer E :

This is not equal to

Choice (E) is not correct.

Back to Score Report

Privacy Policy

Terms of Use

Contact Us

ed

Copyright 2006 The College Board. All rights reserved.

st
er

eg
i

nR

file://E:\\b8.htm

2006-11-12

1/16

The Official SAT Online Course

Help | Profile | My Organizer | My Bookmarks | Logout

Answers and Explanations

Back to Score Report

Test Sections

Section 1

View Answers and Explanations

Section 2

Online - Practice Test #2

Section 3

Section 4

Seeking to ------- what people view and read by determining what art and literature
should be available, censorship laws directly ------- free expression.

Section 6

Section 7

Section 8

(A)

govern. . liberate

(B)

juxtapose. . prescribe

Section 9

(C) defer. . nullify

Section 10

(D) control. . prohibit

(E)

balance. . promote

AND EXPLANATIONS

ANSWERS

er
e

Explanation for Correct Answer D :


Choice (D) is correct. "Control" means to regulate. "Prohibit" means to prevent. If
one were to insert these terms into the text, the sentence would read "Seeking to
control what people view and read by determining what art and literature should be
available, censorship laws directly prohibit free expression." By regulating what
people read and view, censorship laws do in fact prevent free expression.

is
t

Explanation for Incorrect Answer A :


Choice (A) is incorrect. "Govern" means to control. "Liberate" means to free. If one
were to insert these terms into the text, the sentence would read "Seeking to
govern what people view and read by determining what art and literature should be
available, censorship laws directly liberate free expression." Although censorship
laws do seek to control what people read and view, these laws do not allow people
to express themselves freely. In fact, they do the opposite.

eg

nR

Explanation for Incorrect Answer B :


Choice (B) is incorrect. "Juxtapose" means to place side by side. "Prescribe" means
to dictate. If one were to insert these terms into the text, the sentence would read
"Seeking to juxtapose what people view and read by determining what art and
literature should be available, censorship laws directly prescribe free expression."
Censorship laws do not seek to "juxtapose" two things. Censorship laws also do not
"prescribe" free expression. In fact, these laws do just the opposite.

Explanation for Incorrect Answer C :


Choice (C) is incorrect. "Defer" means to put off or postpone. "Nullify" means to
negate. If one were to insert these terms into the text, the sentence would read
"Seeking to defer what people view and read by determining what art and literature
should be available, censorship laws directly nullify free expression." Censorship
laws do negate free expression and thus prevent people from viewing and reading
certain things. However, censorship laws do not put off or postpone people's
viewing and reading; they block it altogether.

Explanation for Incorrect Answer E :


Choice (E) is incorrect. "Balance" means to make equal. "Promote" means to
advocate. If one were to insert these terms into the text, the sentence would read
"Seeking to balance what people view and read by determining what art and
literature should be available, censorship laws directly promote free expression."
Censorship laws do not "balance" the type of art and literature that people view and
read. Also, these laws do not advocate free expression. Instead, they discourage
and prevent it.

file://E:\\b9.htm

Myra laughed exuberantly and embraced her friends repeatedly, so ------- was she

2006-11-12

2/16

The Official SAT Online Course

about having been selected.


(A)

ambivalent

(B)

quizzical

(C) euphoric

(D) jaded

(E)

exacting

ANSWERS

AND EXPLANATIONS

Explanation for Correct Answer C :


Choice (C) is correct. "Euphoric" means full of joy. If one were to insert this term
into the text, the sentence would read "Myra laughed exuberantly and embraced
her friends repeatedly, so euphoric was she about having been selected." The
missing term describes Myras mood as evidenced by her actions, which include
laughing and hugging her friends. It makes sense to describe Myras energetic
displays of happiness as "euphoric."

ed

Explanation for Incorrect Answer A :


Choice (A) is incorrect. "Ambivalent" means having contradictory feelings. If one
were to insert this term into the text, the sentence would read "Myra laughed
exuberantly and embraced her friends repeatedly, so ambivalent was she about
having been selected." The missing term describes Myras mood as evidenced by
her actions, which include laughing and hugging her friends. Such actions reflect
pure joy, not contradictory feelings.

er

Explanation for Incorrect Answer B :


Choice (B) is incorrect. "Quizzical" means disbelieving or questioning. If one were to
insert this term into the text, the sentence would read "Myra laughed exuberantly
and embraced her friends repeatedly, so quizzical was she about having been
selected." Myras energetic displays of happiness are not a reflection of
questioning. Nothing in the sentence indicates that Myra is questioning anything.

is
t

Explanation for Incorrect Answer D :


Choice (D) is incorrect. "Jaded" means worn out or wearied. If one were to insert
this term into the text, the sentence would read "Myra laughed exuberantly and
embraced her friends repeatedly, so jaded was she about having been selected."
Given her energetic display of emotion, it does not make sense to describe Myra as
"jaded," or weary.

eg

nR

Explanation for Incorrect Answer E :


Choice (E) is incorrect. "Exacting" means severely demanding. If one were to insert
this term into the text, the sentence would read "Myra laughed exuberantly and
embraced her friends repeatedly, so exacting was she about having been selected."
The missing term describes Myras mood as evidenced by her actions, which
include laughing "exuberantly," or with high spirits, and hugging her friends. It does
not make sense to describe Myra as severely demanding.

It would be a waste of time for any reviewer to bother ------- a book whose utter
worthlessness is ------- to even the least discerning reader.
(A)

enjoying. . doubtful

(B)

mocking . . figurative

(C) assessing . . welcome

(D) condemning. . obvious

(E)

ignoring . . obnoxious

ANSWERS

AND EXPLANATIONS

Explanation for Correct Answer D :


Choice (D) is correct. "Condemning" means expressing strong disapproval.
"Obvious" means evident or easily understood. If one were to insert these terms
into the text, the sentence would read "It would be a waste of time for any reviewer
to bother condemning a book whose utter worthlessness is obvious to even the

file://E:\\b9.htm

2006-11-12

3/16

The Official SAT Online Course

least discerning reader." This sentence makes sense: a reviewer should not bother
to disapprove of a book whose "worthlessness," or poor quality, is evident to even
the least "discerning," or perceptive, reader.

Explanation for Incorrect Answer A :


Choice (A) is incorrect. "Enjoying" means having fun. "Doubtful" means uncertain. If
one were to insert these terms into the text, the sentence would read "It would be a
waste of time for any reviewer to bother enjoying a book whose utter worthlessness
is doubtful to even the least discerning reader." It makes little sense to advise a
book reviewer not to bother having fun with a book whose quality is uncertain. It is
the reviewer's job to judge the worth or quality of the book in question.

Explanation for Incorrect Answer B :


Choice (B) is incorrect. "Mocking" means making fun of. "Figurative" means
metaphorical or not literal. If one were to insert these terms into the text, the
sentence would read "It would be a waste of time for any reviewer to bother
mocking a book whose utter worthlessness is figurative to even the least discerning
reader." While a reviewer might make fun of a clearly worthless book, it does not
make sense to say that a book's worthlessness is "figurative."

Explanation for Incorrect Answer C :


Choice (C) is incorrect. "Assessing" means evaluating. "Welcome" means giving
pleasure. If one were to insert these terms into the text, the sentence would read
"It would be a waste of time for any reviewer to bother assessing a book whose
utter worthlessness is welcome to even the least discerning reader." A reader would
be unlikely to consider the worthlessness of any book "welcome."

ed

Explanation for Incorrect Answer E :


Choice (E) is incorrect. "Ignoring" means refusing to notice. "Obnoxious" means
offensive or unpleasant. If one were to insert these terms into the text, the
sentence would read "It would be a waste of time for any reviewer to bother
ignoring a book whose utter worthlessness is obnoxious to even the least discerning
reader." This sentence is illogical: "ignoring" something could never be a waste of
time.

te
r

eg
is

The new policy has been called a quiet revolution because, though introduced without
-------, it is already producing ------- changes.

nR

(A)

warning . . specious

(B)

fanfare . . momentous

(C) concealment . . transient

(D) hesitation . . ostensible

(E)

debate . . negligible

ANSWERS

AND EXPLANATIONS

Explanation for Correct Answer B :


Choice (B) is correct. "Fanfare" means showy display. "Momentous" means of the
highest importance. If one were to insert these terms into the text, the sentence
would read "The new policy has been called a quiet revolution because, though
introduced without fanfare, it is already producing momentous changes." The
correct answer builds on the idea that the policy is a "quiet revolution"-- something
that, though understated, has a great effect. A policy that lacks showy display but
produces important changes could definitely be called a quiet revolution.

Explanation for Incorrect Answer A :


Choice (A) is incorrect. A "warning" is an alert. "Specious" means having a false
appearance. If one were to insert these terms into the text, the sentence would
read "The new policy has been called a quiet revolution because, though introduced
without warning, it is already producing specious changes." A policy that is
introduced without a prior alert may or may not be called a quiet revolution.
However, "specious changes," or apparent changes that are actually not changes at
all, would not be part of a "quiet revolution."

file://E:\\b9.htm

2006-11-12

4/16

The Official SAT Online Course

Explanation for Incorrect Answer C :


Choice (C) is incorrect. "Concealment" means secrecy. "Transient" means remaining
in place for only a brief time. If one were to insert these terms into the text, the
sentence would read "The new policy has been called a quiet revolution because,
though introduced without concealment, it is already producing transient changes."
The correct answer builds on the idea that the policy is a quiet revolution, or
something that, though understated or restrained, has a great effect. A policy that
is introduced without secrecy would probably not be called quiet. Moreover, it does
not make sense to refer to changes as "transient."

Explanation for Incorrect Answer D :


Choice (D) is incorrect. "Hesitation" means a pause due to uncertainty. "Ostensible"
means seeming to be real. If one were to insert these terms into the text, the
sentence would read "The new policy has been called a quiet revolution because,
though introduced without hesitation, it is already producing ostensible changes."
While a revolutionary policy might be introduced without "hesitation," one would not
describe the changes produced by such a policy as only seeming to be real.

Explanation for Incorrect Answer E :


Choice (E) is incorrect. A "debate" is a discussion between two opposing sides.
"Negligible" means extremely unimportant. If one were to insert these terms into
the text, the sentence would read "The new policy has been called a quiet revolution
because, though introduced without debate, it is already producing negligible
changes." The changes produced by a policy called a quiet revolution would not be
unimportant.

ed

When two chemical compounds are combined, a ------- effect can be achieved: the
resulting combination can be more potent than either of the individual compounds
alone.
(A)

synergistic

(B)

naturalistic

(C) competitive

(D) retroactive

neutralizing

eg
i

(E)

st
er

ANSWERS

AND EXPLANATIONS

nR

Explanation for Correct Answer A :


Choice (A) is correct. "Synergistic" refers to two or more things working together to
create something powerful. If one were to insert this term into the text, the
sentence would read "When two chemical compounds are combined, a synergistic
effect can be achieved: the resulting combination can be more potent than either of
the individual compounds alone." The missing term describes a combination that is
"more potent," or stronger, than either of its "compounds," or parts, is alone. A
mixture whose combination is more powerful than its individual parts is
"synergistic" by definition.

Explanation for Incorrect Answer B :


Choice (B) is incorrect. "Naturalistic" refers to imitating nature. If one were to insert
this term into the text, the sentence would read "When two chemical compounds
are combined, a naturalistic effect can be achieved: the resulting combination can
be more potent than either of the individual compounds alone." "Naturalistic" refers
to something that is like or imitates nature, not something that is stronger than its
individual parts.

Explanation for Incorrect Answer C :


Choice (C) is incorrect. "Competitive" refers to the enjoyment of contests or rivalry.
If one were to insert this term into the text, the sentence would read "When two
chemical compounds are combined, a competitive effect can be achieved: the
resulting combination can be more potent than either of the individual compounds
alone." A mixture whose combination is more powerful than its individual parts has
nothing to do with being "competitive."

Explanation for Incorrect Answer D :


Choice (D) is incorrect. "Retroactive" means applying to a prior period. If one were
to insert this term into the text, the sentence would read "When two chemical
compounds are combined, a retroactive effect can be achieved: the resulting

file://E:\\b9.htm

2006-11-12

5/16

The Official SAT Online Course

combination can be more potent than either of the individual compounds alone."
The fact that a combination is more powerful than its individual parts has nothing to
do with it being "retroactive," or referring to a prior period.

Explanation for Incorrect Answer E :


Choice (E) is incorrect. "Neutralizing" means counteracting or making ineffective. If
one were to insert this term into the text, the sentence would read "When two
chemical compounds are combined, a neutralizing effect can be achieved: the
resulting combination can be more potent than either of the individual compounds
alone." "Neutralizing" the compounds would make them less potent rather than
more potent.

Artists who are described as ------- are the first to experiment with new forms or
concepts.
(A)

aesthetic

(B)

partisan

(C) decorous

(D) cerebral

(E)

avant-garde

AND EXPLANATIONS

ANSWERS

er
e

Explanation for Correct Answer E :


Choice (E) is correct. "Avant-garde" means innovative or cutting-edge. If one were
to insert this term into the text, the sentence would read "Artists who are described
as avant-garde are the first to experiment with new forms or concepts." The
missing term is defined by the final phrase "the first to experiment with new forms
or concepts." This is precisely the definition of an "avant-garde" person.

st

eg
i

Explanation for Incorrect Answer A :


Choice (A) is incorrect. "Aesthetic" means artistic. If one were to insert this term
into the text, the sentence would read "Artists who are described as aesthetic are
the first to experiment with new forms or concepts." An artistic person would not
necessarily be "the first to experiment with new forms or concepts."

nR

Explanation for Incorrect Answer B :


Choice (B) is incorrect. "Partisan" means supporting a certain group. If one were to
insert this term into the text, the sentence would read "Artists who are described as
partisan are the first to experiment with new forms or concepts." A "partisan"
person would not necessarily be "the first to experiment with new forms or
concepts."

Explanation for Incorrect Answer C :


Choice (C) is incorrect. "Decorous" means showing good manners. If one were to
insert this term into the text, the sentence would read "Artists who are described as
decorous are the first to experiment with new forms or concepts." A well-mannered
person would not necessarily be "the first to experiment with new forms or
concepts."

Explanation for Incorrect Answer D :


Choice (D) is incorrect. "Cerebral" means intellectual. If one were to insert this term
into the text, the sentence would read "Artists who are described as cerebral are
the first to experiment with new forms or concepts." Although artists who
experiment with new forms and concepts can be cerebral, they do not have to be
cerebral.

The following passage was adapted from a book about the Great Plains, a plateau
extending westward from the prairies of central North America to the eastern base of
the Rocky Mountains.

Before the railroads were built, the way


west followed

file://E:\\b9.htm

2006-11-12

6/16

The Official SAT Online Course

the rivers: west along the Platte into Wyoming,


over South
Pass, up the Snake River into the Oregon
Territory; or up
the Missouri through the Dakotas and into
Line
Montana, then
west along the Yellowstone. It was the easiest
5
but not the
most accurate way to see the country. The
country looked
better or worse from the prospect of the river; I
cant say
which, not having gone that way. But the
country looked
different, certainly, not at all like the Great
Plains. There
are many reasons why it could not have
10
seemed the same.
A river is an edge, for one thing, and not
simply in the
cartographers sense that it divides one piece
of ground
from another. It is a biological edge. There
worlds collide,
strangers meet, and much business, although
not of the
monetary kind, is transacted. Edges in the
15
natural world
are like cities in human cultures. They are the
cosmopolitan
placespopulous, noisy, gaudy, rich, exciting
where one
expects the unexpected and the extraordinary
is ordinary.
They are altogether unlike the provinces, where
the surprises
lie not in discovering what is odd or new but in
20
appreciating,
at last, what is routine and everyday, a larger
accomplishment than one might imagine. The rivers of the
earliest
westward travelers passed through the
provinces, but these
rivers revealed a world that was not, in itself,
characteristic
25 of those regions.
Rivers carry water, for instance, but the
region of the
Great Plains is by its nature aridnot so arid as
the deserts,
although for a long time the Great Plains were

re

te

is

eg

nR

file://E:\\b9.htm

2006-11-12

7/16

The Official SAT Online Course

30

35

regarded
as a desert, but arid enough to inhibit the
growth of trees,
except along rivers. You could no more know
the Great
Plains by canoeing up the Missouri than you
could see
the Sonoran Desert by rafting down the
Colorado. River
travelers poled or steamed up the channels by
day and
fished for supper by twilight. The Blackfeet, the
lords of
the Great Plains and the prairies most serious
students,
would no sooner have dined on catfish than we
would on
a dish of fricasseed sewer rat. The mucuscovered creatures
of the muddy river bottoms, the Blackfeet
thought, were
simply not the best the plains had to offer; far
from being
palatable, catfish were repulsive, disgusting.
The rivers, moreover, seek the level, that is,
the low
ground, but the plains are the province of the
big sky. The
rivers are always running away to the sea, but
the plains
are always rising toward the mountains. They
are contrary
forces working in contrary directions. The rivers
dig in; the
plains surmount. A river closes in, rounds the
bend, runs
between banks, hides shallows and snags,
tumbles over
rapids, skirts islands, is forever calling attention
to itself,
like a trail, which a river inevitably becomes.
The plains,
on the other hand, open out, unfold, beg the
long and trackless view. The river draws a line; the plains
reveal a space.
It is like the difference between an
interstate expressway
and a county road. An interstate is broad and
swift and, in
its own way, keeps to the level. You can drive
on an inter-

ed

er

st

40

eg
i

50

file://E:\\b9.htm

45

nR

2006-11-12

8/16

The Official SAT Online Course

60

65

state across the most endlessly enchanting of


countrysides
and encounter only an unrelieved monotony.
The expressway exists in its own world, an unwalled tunnel,
and moves
at its own urgent pace. It has a rhythm and a
rigidly regular
time quite distinct from the landscape it
crosses. It would
not serve its purposes in any other way.
The county road, on the other hand, moves
in and
through the landscape and exists as one more
feature of
it. Where there is a tree and a sun, the road
falls under
shadow; where there is a stream, the road
follows down
one bank, across the water, and up the other
bank; where
a tall hill intervenes, the road goes around
rather than
through it and the traveler enjoys the sensation
of having
moved among hills; at the village, the road
unhesitatingly
takes it in, and your own pace slows to
accommodate the
taking in, rather than swooping to the right or
left around
the settlement at a curve calculated for highspeed safety.
A stray cow might cross in front of you and you
will be
obliged to stop to let it pass, and so you will
chance to
hear the song of the meadowlark on the fence
post. The
hay wagon ahead cannot be gotten around; you
are forced
to reduce your own speed to the local standard,
and so you
see the marsh hawk circling above a pothole.
The best way to go west, therefore, is the
slowest way
possible and across country rather than along
the rivers,
avoiding both the old watery rivers and the new
ones of
asphalt and cast concrete.

te
re
d

55

is

eg

nR

70

75

80

file://E:\\b9.htm

The authors primary purpose in the passage is to

2006-11-12

9/16

The Official SAT Online Course

(A)

introduce the narrative figure of the traveler

(B)

convey the excitement felt by the earliest explorers

(C) encourage an appreciation of the Great Plains

(D) establish the vanished beauty of western rivers

(E)

confirm the mysterious nature of the Great Plains

ANSWERS

AND EXPLANATIONS

Explanation for Correct Answer C :


Choice (C) is correct. The author's central thesis that one must travel across the
land in order to know the richness of the prairie is meant to encourage an
increased appreciation or understanding of the Great Plains. As the author states,
"the best way to go west, therefore, is the slowest way possible" (lines 78-79).

Explanation for Incorrect Answer A :


Choice (A) is incorrect. The author's main purpose is to encourage travelers to
explore and better understand the prairie. The author is not writing a "narrative."

ed

Explanation for Incorrect Answer B :


Choice (B) is incorrect. Although the author mentions the region's earliest travelers,
relating the excitement of their exploration is not the main point of the passage.

Explanation for Incorrect Answer D :


Choice (D) is incorrect. In this passage, the author does not appear to be interested
in whether or not the western rivers are beautiful, or whether they are less
beautiful than they used to be.

is
te
r

Explanation for Incorrect Answer E :


Choice (E) is incorrect. The author's purpose is not to make the Great Plains seem
mysterious, but rather to encourage readers to know and understand the prairie on
its own terms.

eg

For the author, a biological edge (line 13) represents a

nR

(A)

place where communities mingle

(B)

barrier that separates different groups

(C) contrast to an urbanized environment

(D) change over time in the geography of the land

(E)

clash between incompatible ways of life

ANSWERS

AND EXPLANATIONS

Explanation for Correct Answer A :


Choice (A) is correct. The passage describes a biological edge as a place where
worlds collide, strangers meet (lines 13-14). A biological edge is a "place
where communities mingle," or mix.

Explanation for Incorrect Answer B :


Choice (B) is incorrect. The opposite is actually true: in line 13, a biological edge is
a place where worlds collide, not a structure or limit that keeps different
groups apart.

Explanation for Incorrect Answer C :


Choice (C) is incorrect. The author actually claims that a biological edge is similar to
an urban environment, or city: "edges in the natural world are like cities in human
cultures" (lines 15-16). They are places were different groups are thrown together
with often unexpected results.

Explanation for Incorrect Answer D :

file://E:\\b9.htm

2006-11-12

10/16

The Official SAT Online Course

Choice (D) is incorrect. A biological edge does not represent a change over time,
but a situation in which different kinds and systems of life interact.

Explanation for Incorrect Answer E :


Choice (E) is incorrect. According to the passage, the ways of life that exist at a
biological edge are not incompatible. On the contrary, such an edge is a productive
place of exchange among different groups.

In line 20, appreciating most nearly means


(A)

preserving from harm

(B)

increasing in value

(C) understanding better

(D) praising

(E)

thanking

ANSWERS

AND EXPLANATIONS

Explanation for Correct Answer C :


Choice (C) is correct. The passage explains that edges "are altogether unlike the
provinces, where the surprises lie not in discovering what is odd or new but in
appreciating, at last, what is routine and everyday" (lines 19-21). Since surprises
come about when one understands the routine aspects of life better, "appreciating"
most nearly means "understanding better" in this context.

er
e

is
t

Explanation for Incorrect Answer A :


Choice (A) is incorrect. Nothing in the passage indicates that the provinces are in
danger of being harmed or that they require preservation.

Explanation for Incorrect Answer B :


Choice (B) is incorrect. While "appreciating" can mean increasing in value over
time, it does not make sense to say that the surprises of the provinces lie in
increasing the value of the routine and everyday. Rather, "appreciating" means
understanding the value that is already there.

eg

nR

Explanation for Incorrect Answer D :


Choice (D) is incorrect. In this context, "appreciating" means understanding the
meaning or importance of something. It does not make sense to say that the
surprises lie in "praising" the routine and everyday.

Explanation for Incorrect Answer E :


Choice (E) is incorrect. Although one may give thanks for something that one
appreciates, it does not make sense to say that the surprises of the provinces lie in
"thanking" what is routine and everyday.

10

In lines 26-32 (Rivers carry . . . Colorado), the author implies that viewing the
plains from the prospect of a river might lead one to conclude that the plains are
(A)

less arid than they actually are

(B)

less populous than they actually are

(C) rising more rapidly toward the mountains than they actually do

(D) not fertile enough to sustain the growth of trees

(E)

not as arid as the desert

ANSWERS

AND EXPLANATIONS

Explanation for Correct Answer A :


Choice (A) is correct. The passage states that the Great Plains are "arid enough to
inhibit the growth of trees, except along rivers" (lines 29-30). It makes sense to
say that if one only saw the Great Plains from a river, where trees grow, one might
get the mistaken impression that the plains are not arid.

file://E:\\b9.htm

2006-11-12

11/16

The Official SAT Online Course

Explanation for Incorrect Answer B :


Choice (B) is incorrect. Nothing in the passage indicates that the perspective of river
travel would lead one to think of the plains as less "populous," or crowded, than
they actually are.

Explanation for Incorrect Answer C :


Choice (C) is incorrect. The passage indicates that "the rivers are always running
away to the sea, but the plains are always rising toward the mountains" (lines 4244). The passage implies that river travel would lead one to miss the fact that the
plains are rising. It does not suggest that one would see the plains to be "rising
more rapidly toward the mountains than they actually do."

Explanation for Incorrect Answer D :


Choice (D) is incorrect. The opposite is most likely true. The passage states that the
Great Plains are "arid enough to inhibit the growth of trees, except along
rivers" (lines 29-30). From the riverbanks, where trees grow, one would wrongly
conclude that the plains are "fertile enough to sustain the growth of trees."

Explanation for Incorrect Answer E :


Choice (E) is incorrect. The point of lines 26-32 is that viewing the plains from the
river gives one a mistaken impression. The plains are in fact "not as arid as the
desert," so a traveler having this impression would be correct. Lines 26-32 focus
instead on travelers' mistakes: "you could no more know the Great Plains by
canoeing up the Missouri than you could see the Sonoran Desert by rafting down
the Colorado."

The author mentions the Blackfeet (lines 34-40) primarily because

te

11

re
d

(A)

they appreciated the plains

(B)

they were experts in using the resources of the rivers

is

(C) they cared about the ecology of the plains

(D) river travelers learned a lot from them

eg

(E)

local people were in awe of them

AND EXPLANATIONS

nR

ANSWERS

Explanation for Correct Answer A :


Choice (A) is correct. The author holds up the Blackfeet as the prairies most
serious students as an example of people who knew and appreciated the plains,
and who had little interest in the very different life on the river. The fact that the
Blackfeet refused to eat catfish from the rivers is offered as support for the author's
claim that life on the rivers is very different from life on the open prairie.

Explanation for Incorrect Answer B :


Choice (B) is incorrect. The passage actually states the opposite: the Blackfeet did
not use the resources of the rivers, even refusing to eat the catfish that swam
there. Instead, they focused their energies on making use of the land.

Explanation for Incorrect Answer C :


Choice (C) is incorrect. While the Blackfeet probably cared about the "ecology" of
the plains, that does not explain why the author mentions them. They are used to
support the passages broader claims about the very different worlds one
encounters on the river and on land.

Explanation for Incorrect Answer D :


Choice (D) is incorrect. Nothing in the passage indicates that river travelers learned
from the Blackfeet, who stayed on the land and knew little of the waters.

Explanation for Incorrect Answer E :


Choice (E) is incorrect. The passage does not discuss the relationship between the
Blackfeet and any other local people.

file://E:\\b9.htm

2006-11-12

12/16

The Official SAT Online Course

12

The author indicates that the view offered by the interstate is similar to that offered
by the river in that both
(A)

are teeming with unexpected surprises

(B)

are reflective of environmental changes

(C) are subjected to minor obstructions

(D) give the traveler an unobstructed look at nature

(E)

give an erroneous impression of the countryside

ANSWERS

AND EXPLANATIONS

Explanation for Correct Answer E :


Choice (E) is correct. According to the passage, both the interstate and the river
give an "erroneous," or incorrect, impression of the land. As the author explains,
"you can drive on an interstate across the most endlessly enchanting of
countrysides and encounter only an unrelieved monotony" (lines 54-56). Likewise,
"the river draws a line; the plains reveal a space" (line 51). Interstates and rivers
both separate the traveler from the true nature of the places they cross.

re
d

Explanation for Incorrect Answer A :


Choice (A) is incorrect. The author does not describe the view from the interstate as
teeming with, or full of, the unexpected, but rather as dull and monotonous.

Explanation for Incorrect Answer B :


Choice (B) is incorrect. The author describes the river as "calling attention to
itself" (line 48) and the interstate as existing "in its own world" (line 57). In other
words, both the interstate and the river are cut off from the land, and are thus not
reflective of changes in the environment.

is
te

Explanation for Incorrect Answer C :


Choice (C) is incorrect. It is not the river and highway that are likely to involve
obstructions: the interstate, for example, is decribed as "broad and swift" (line 53).

eg

Explanation for Incorrect Answer D :


Choice (D) is incorrect. The view from the river and interstate is not
"unobstructed," or free. On the contrary, the passage argues that the only way to
freely and immediately experience the plains is to travel across the land itself, via
county roads.

13

nR

In context, the word tunnel (line 57) most directly emphasizes the expressways
(A)

function as a means of escape from provincial dullness

(B)

role as a connection between two points

(C) independence from its surroundings

(D) intrusion into the landscape

(E)

destruction of wildlife

ANSWERS

AND EXPLANATIONS

Explanation for Correct Answer C :


Choice (C) is correct. The passage describes the expressway as "exist[ing] in its
own world, an unwalled tunnel, and mov[ing] at its own pace." This suggests that
the expressway is independent of its surroundings.

Explanation for Incorrect Answer A :


Choice (A) is incorrect. Nothing in the passage emphasizes the expressway as a
means of escape from dullness; on the contrary, the view from the expressway is
described as being one of "unrelieved monotony" (line 56).

file://E:\\b9.htm

2006-11-12

13/16

The Official SAT Online Course

Explanation for Incorrect Answer B :


Choice (B) is incorrect. The passage does not emphasize a tunnel as something that
connects points, but as something that is isolated from the landscape in which it
exists.

Explanation for Incorrect Answer D :


Choice (D) is incorrect. The opposite is actually true. The author describes the
expressway as remaining distinct and separate from the countryside through which
it passes.

Explanation for Incorrect Answer E :


Choice (E) is incorrect. The authors description of the expressway as an "unwalled
tunnel" does not suggest any connection between the expressway and the
destruction of wildlife.

14

The description of the county road (lines 61-77) suggests that it is most like the
Great Plains in the way that it
(A)

encourages a feeling of satisfaction with ones lot in life

(B)

fosters a sense of the world around one

(C) creates a desire to cultivate stillness and silence

(D) evokes a sense of power and mastery over the land

ANSWERS

AND EXPLANATIONS

imparts an intuitive awareness of the complexity of all life

re

(E)

te

Explanation for Correct Answer B :


Choice (B) is correct. Like the Great Plains, the county road allows the traveler to
interact with different elements of the country. The county road "moves in and
through the landscape" (lines 61-62). It makes sense to say that the experience of
driving on this road "fosters," or encourages, a sense of the world around one.

is

eg

Explanation for Incorrect Answer A :


Choice (A) is incorrect. While the author celebrates the Great Plains and the county
road for the pleasures they provide to the traveler, the passage does not argue that
either "encourages a feeling of satisfaction with ones lot in life."

nR

Explanation for Incorrect Answer C :


Choice (C) is incorrect. The passage does not mention stillness or silence. Rather,
the county road encourages the traveler to immerse himself or herself in the local
rhythms of life.

Explanation for Incorrect Answer D :


Choice (D) is incorrect. Neither the county road nor the Great Plains "evokes," or
causes one to feel, a sense of power and mastery over the land. On the contrary,
both demand that the traveler adjust to suit the landscape, changing his or her
speed and route as necessary.

Explanation for Incorrect Answer E :


Choice (E) is incorrect. The author does not discuss "an intuitive awareness of the
complexity of all life" in this passage.

15

Both the stray cow (line 72) and the hay wagon (line 75) are presented as
(A)

features of a rural landscape that are depicted sentimentally in American


art

(B)

objects that present unexpected dangers to the unwary

(C) memories of the authors childhood that are typical of the Great Plains

(D) obstacles that force people to pay attention to the beauty of their
surroundings

(E)

file://E:\\b9.htm

symbols of an agrarian ideal that attracted settlers to the west

2006-11-12

14/16

The Official SAT Online Course

ANSWERS

AND EXPLANATIONS

Explanation for Correct Answer D :


Choice (D) is correct. The author presents both the cow and the wagon as examples
of "obstacles," or things that prevent passage and force travelers to slow or stop.
As the passages describes it, "A stray cow might cross in front of you and you will
be obliged to stop to let it pass, and so you will chance to hear the song of the
meadowlark" (lines 72-74). The key thing about these obstacles is that they give
travelers the opportunity to notice the beauty that surrounds them.

Explanation for Incorrect Answer A :


Choice (A) is incorrect. The passage does not discuss the depiction of rural
landscapes in American art.

Explanation for Incorrect Answer B :


Choice (B) is incorrect. The cow and wagon are not described as dangers. Rather,
they are presented as obstacles encountered on county roads that force one to stop
and enjoy the environment.

Explanation for Incorrect Answer C :


Choice (C) is incorrect. The passage makes no mention of the authors childhood
or memories.

ed

Explanation for Incorrect Answer E :


Choice (E) is incorrect. While the cow and wagon may be symbols of the
countryside, the passage is not concerned with an "agrarian ideal," or dream of a
life of farming, that may have attracted settlers.

is
t

The opinion expressed in lines 78-81 (The best . . . concrete) assumes that the
westward traveler

(A)

is familiar with the Great Plains

(B)

dislikes cities

eg

16

er

(C) appreciates the slow pace of river travel

nR

(D) is interested in studying different species in nature

(E)

wants to savor the countryside

ANSWERS

AND EXPLANATIONS

Explanation for Correct Answer E :


Choice (E) is correct. The passages states that "the best way to go west, therefore,
is the slowest way possible and across country rather than along the rivers" (lines
78-79). This advice assumes that a traveler actually wants to spend time in the
countryside.

Explanation for Incorrect Answer A :


Choice (A) is incorrect. The authors opinion that it is best to travel slowly across
the Great Plains assumes that one is unfamiliar with the area and interested in
learning about it.

Explanation for Incorrect Answer B :


Choice (B) is incorrect. Nothing in the passage suggests that enjoyment of the
countryside is necessarily related to a dislike of cities.

Explanation for Incorrect Answer C :


Choice (C) is incorrect. The passage actually urges the traveler to avoid "both the
old watery rivers and the new ones of asphalt and cast concrete" (lines 80-81).

Explanation for Incorrect Answer D :


Choice (D) is incorrect. The author encourages westward travelers to go slowly
through the prairies so that they may enjoy and appreciate the countryside.
Nothing in the passage mentions the study of different species of life, however.

file://E:\\b9.htm

2006-11-12

15/16

The Official SAT Online Course

17

How is the way referred to in line 78 best?


(A)

It is safe and easy.

(B)

It provides the most insight.

(C) It is the most convenient.

(D) It has the most historical significance.

(E)

It illustrates the authors fund of knowledge.

ANSWERS

AND EXPLANATIONS

Explanation for Correct Answer B :


Choice (B) is correct. In the final sentence of the passage, the author states, "the
best way to go west, therefore, is the slowest way possible." The author suggests
that the slowest way across the Great Plains is the best way because it provides the
greatest understanding of the land.

ed

Explanation for Incorrect Answer A :


Choice (A) is incorrect. The passage does not describe county roads as "safe and
easy," but rather as slow trails that require the traveler to adjust his or her
schedule to the shape and rhythm of the landscape.

er

Explanation for Incorrect Answer C :


Choice (C) is incorrect. The opposite is actually true: far from being convenient, the
land route is described as slow and roundabout, requiring that the traveler adapt to
the plains and to obstacles on the road.

is
t

Explanation for Incorrect Answer D :


Choice (D) is incorrect. Given that the earliest explorers traveled west by river, it is
likely that the water routes, rather than the overland routes, have the most
historical significance. Further, there is no discussion of the historical significance of
the Great Plains.

eg

Explanation for Incorrect Answer E :


Choice (E) is incorrect. Nothing in the passage indicates that traveling overland is
best because it illustrates the authors "fund," or store, of knowledge. On the
contrary, the prairie adds to and expands the author's fund of knowledge.

nR

U
18

Which of the following best expresses the authors central point about river travel?
(A)

It was often hazardous.

(B)

It caused disputes between travelers and the residents of the Northwest


Territory.

(C) It was an inefficient way to travel.

(D) It offered little opportunity to learn about the landscape of the Great
Plains.

(E)

It offered the safest route to the West Coast.

ANSWERS

AND EXPLANATIONS

Explanation for Correct Answer D :


Choice (D) is correct. According to the passage, river travelers did not see much of
the Great Plains. Cut off from the land, the world they experienced looked "not at
all like the Great Plains" (line 9). The author emphasizes that river travel was "not
the most accurate way to see the country" (lines 5-6) because river travelers saw
little of the Great Plains' celebrated landscapes.

Explanation for Incorrect Answer A :


Choice (A) is incorrect. The passage does not discuss whether or not river travel

file://E:\\b9.htm

2006-11-12

16/16

The Official SAT Online Course

was dangerous.

Explanation for Incorrect Answer B :


Choice (B) is incorrect. The passage does not include any information on disputes,
or fights, between travelers on the river and residents of the Northwest Territory.

Explanation for Incorrect Answer C :


Choice (C) is incorrect. The author does not desribe river travel as "inefficient," or
something that wastes time. On the contrary, the passage states that traveling by
the river was "the easiest but not the most accurate way to see the country" (lines
5-6).

Explanation for Incorrect Answer E :


Choice (E) is incorrect. The passage does not address which westward route offered
the safest passage to the coast. Rather, it contrasts what one learns about the
prairie from river travel with what one learns about the prairie by traveling
overland.

19

The passage as a whole most fully answers which question?


(A)

Which western river provides the best access to the mountains?

(B)

Which ecosystems of the Great Plains are most exciting for visitors?

(C) What do the rivers of the West teach us about our past?

(E)

te
re
d

(D) What route west provides most information about the Great Plains?

How can one identify the best scenery in the Great Plains?

ANSWERS

AND EXPLANATIONS

Explanation for Correct Answer D :


Choice (D) is correct. In the passage, the author compares the information
travelers gather about the country on different western routes, ultimately
concluding that only by going on slow county roads can one really learn about the
Great Plains. As the last sentence of the passage points out, "the best way to go
west, therefore, is the slowest way possible and across country rather than along
the rivers" (lines 78-79).

is

eg

nR

Explanation for Incorrect Answer A :


Choice (A) is incorrect. The passage mentions several river routes but does not
single any out as being the best route to the mountains.

Explanation for Incorrect Answer B :


Choice (B) is incorrect. Although the author encourages travelers to take the time
to learn about and appreciate the prairie, the passage does not discuss which Great
Plains ecosystems visitors find most exciting.

Explanation for Incorrect Answer C :


Choice (C) is incorrect. Although the passage mentions the experience of earlier
explorers on the western rivers, it does not make broad claims about what these
rivers can teach us about the past.

Explanation for Incorrect Answer E :


Choice (E) is incorrect. While the passage includes descriptions of the countryside,
it does so to compare different ways of traveling across the prairie, not to "identify
the best scenery." The author states that one gets an inaccurate impression of the
prairie by traveling along a river, but the passage does not claim that river scenery
is better or worse than other scenery.

Back to Score Report

Copyright 2006 The College Board. All rights reserved.

file://E:\\b9.htm

Privacy Policy

Terms of Use

Contact Us

2006-11-12

1/9

The Official SAT Online Course

Help | Profile | My Organizer | My Bookmarks | Logout

Answers and Explanations

Back to Score Report

Test Sections

Section 1

View Answers and Explanations

Section 2

Online - Practice Test #2

Section 3

Section 4

The amount of garbage produced in the United States could be reduced by recycling
trash, minimizing packaging, and developing new technology for incinerators and
landfills.

Section 6

Section 7

Section 8

Section 9

(A)

and developing new technology

(B)

and if they develop new technology

(C) and also by developing new technology

Section 10

(D) and new technology being developed

and if there was new technology

ANSWERS

AND EXPLANATIONS

(E)

re

Explanation for Correct Answer A :


Choice (A) is correct. It uses three parallel verbal phrases ("recycling trash,"
"minimizing packaging," and "developing new technology") to identify three actions
that might reduce the amount of garbage.

st
e

eg
i

Explanation for Incorrect Answer B :


Choice (B) fails to maintain parallelism. The clause "they develop" (used to state
the third action in a series) is not parallel with the verbals "recycling" and
"minimizing" (used to name the two earlier actions).

nR

Explanation for Incorrect Answer C :


Choice (C) displays wordiness. Since the conjunction "and" already implies that
additional information will follow, the word "also" is not needed; since the
preposition "by" is not repeated before the second verbal in the series
("minimizing"), it is not needed before the third ("developing").

Explanation for Incorrect Answer D :


Choice (D) has a flaw in parallelism. Although two verbals in the series ("recycling"
and "minimizing") are in active voice, the third verbal ("being developed") shifts
awkwardly to passive voice.

Explanation for Incorrect Answer E :


Choice (E) violates parallelism. The clause "there was" is not parallel with the
earlier verbals "recycling" and "minimizing."

Tickets are available at the box office they can be picked up one hour before the
performance.
(A)

at the box office they can be picked up one hour before the performance

(B)

at the box office; they can be picked up one hour before the performance

(C) one hour before the performance, they can be picked up at the box office

(D) and that can be picked up at the box office one hour before the
performance

(E)

at the box office, one hour before the performance is when they can be
picked up

ANSWERS

file://E:\\b10.htm

AND EXPLANATIONS

2006-11-12

2/9

The Official SAT Online Course

Explanation for Correct Answer B :


Choice (B) is correct. It avoids the error of the original by using a semicolon to join
two complete thoughts.

Explanation for Incorrect Answer A :


Choice (A) displays improper coordination. Two complete thoughts ("Tickets . . . at
the box office" and "they can . . . before the performance") are fused together
without any punctuation to indicate their boundaries.

Explanation for Incorrect Answer C :


Choice (C) uses improper coordination. Two complete thoughts ("Tickets . . . before
the performance" and "they can . . . at the box office") are linked with only a
comma.

Explanation for Incorrect Answer D :


Choice (D) uses an inappropriate pronoun. The singular pronoun "that" does not
agree with the plural noun "tickets."

Explanation for Incorrect Answer E :


Choice (E) exhibits improper coordination. It uses only a comma to link two
complete thoughts ("Tickets . . . at the box office" and "one hour before the
performance . . . picked up").

re

Medical research now emphasizes that the best treatment for many diseases is the
bodys own defenses being stimulated.

(A)

the best treatment for many diseases is the bodys own defenses being
stimulated

(B)

the best treatment for many diseases is stimulation of the bodys own
defenses

is
te

(C) there should be stimulation of the bodys own defenses for the best
treatment for many diseases

(D) there should be a stimulation of the bodys own defenses to achieve the
best treatment for many diseases

nR
eg
(E)

the bodys own defenses should have stimulation for the best treatment
for many diseases

ANSWERS

AND EXPLANATIONS

Explanation for Correct Answer B :


Choice (B) is correct. It avoids the error of the original by linking the subject of the
clause,"treatment," with the predicate noun, "stimulation."

Explanation for Incorrect Answer A :


Choice (A) involves logical disagreement. The resulting sentence links two nouns
("treatment" and "defenses") of different logical types, illogically suggesting that
the bodys defenses are a medical treatment rather than the object of such a
treatment.

Explanation for Incorrect Answer C :


Choice (C) results in wordiness. The phrase "there should be" is unnecessary.

Explanation for Incorrect Answer D :


Choice (D) results in wordiness and creates an improper idiom. The phrase "there
should be" is unnecessary. The phrase "a stimulation" improperly suggests a
discrete event rather than an ongoing effort.

Explanation for Incorrect Answer E :


Choice (E) involves an improper idiom. The phrase "should have stimulation"
improperly suggests that stimulation is a form of treatment for the defenses rather
than for the body.

file://E:\\b10.htm

Some political scientists are convinced that the major conflict toward democracy and

2006-11-12

3/9

The Official SAT Online Course

communism is moral.
(A)

toward democracy and communism is moral

(B)

is a moral one between democracy and communism

(C) between democracy, compared to communism, is a moral one

(D) is a moral one for democracy, compared to communism

(E)

between democracy and communism is a moral one

ANSWERS

AND EXPLANATIONS

Explanation for Correct Answer E :


Choice (E) is correct. It avoids the error of the original by replacing the preposition
toward with the idiomatic preposition between.

Explanation for Incorrect Answer A :


Choice (A) results in an improper idiom. The noun conflict is followed by the
preposition toward, rather than by the preposition between.

ed

Explanation for Incorrect Answer B :


Choice (B) creates an illogical sentence. The sentence identifies the major
conflict as a moral one rather than as one between democracy and
communism.

Explanation for Incorrect Answer C :


Choice (C) involves an improper idiom. The phrase between democracy is
followed by the phrase compared to communism rather than by and
communism.

er

is
t

Explanation for Incorrect Answer D :


Choice (D) results in an illogical sentence. The conflict that political scientists have
identified exists between democracy and communism, not for democracy,
compared to communism.

eg

nR

Until just recently many students would take lengthy trips during spring break rather
than temporary jobs like now.
(A)

break rather than temporary jobs like now

(B)

break instead of temporary jobs like now

(C) break; now they take temporary jobs

(D) break; instead, students are taking temporary jobs

(E)

break, but now it is temporary jobs

ANSWERS

AND EXPLANATIONS

Explanation for Correct Answer C :


Choice (C) is correct. It avoids the error of the original by providing a clause, "now
they take temporary jobs," whose structure parallels that of the first clause. It thus
clarifies the comparison between what students used to do and what they do now.

Explanation for Incorrect Answer A :


Choice (A) involves an illogical comparison. The word "like" indicates that
"temporary jobs" are illogically being compared to "now."

Explanation for Incorrect Answer B :


Choice (B) results in an illogical comparison. By the use of the word "like," it
illogically compares a noun, "jobs," to an adverb, "now."

Explanation for Incorrect Answer D :


Choice (D) is unsatisfactory because of vague wording. It provides the adverb
"instead" to introduce the second clause ("students are . . . jobs") instead of the

file://E:\\b10.htm

2006-11-12

4/9

The Official SAT Online Course

adverb "now" that is needed, thus obscuring the comparison between what students
used to do and what they do now.

Explanation for Incorrect Answer E :


Choice (E) involves the use of a vague pronoun. There is nothing in the sentence to
which the pronoun "it" can logically refer.

To meet the colleges requirement of service to her community, 40 hours of her time
to Habitat for Humanity were volunteered by Laurie.

(A)

40 hours of her time to Habitat for Humanity were volunteered by Laurie

(B)

Laurie volunteered 40 hours of her time to Habitat for Humanity

(C) 40 hours of her time to Habitat for Humanity was what Laurie volunteered

(D) through 40 hours of volunteering by Laurie to Habitat for Humanity

(E)

Laurie, to Habitat for Humanity, was volunteering 40 hours of her time

ANSWERS

AND EXPLANATIONS

Explanation for Correct Answer B :


Choice (B) is correct. It avoids the error of the original by placing the introductory
phrase "To meet . . . community" directly next to "Laurie," the noun it modifies.

ed

st
er

Explanation for Incorrect Answer A :


Choice (A) involves improper modification. The introductory phrase, "To meet . . .
community," improperly modifies "40 hours" rather than "Laurie."

Explanation for Incorrect Answer C :


Choice (C) creates improper modification. The opening phrase, "To meet . . .
community," improperly modifies "40 hours" rather than "Laurie."

eg
i

Explanation for Incorrect Answer D :


Choice (D) results in an incomplete thought. The introductory phrase "To meet . . .
community" is followed by another phrase, "through . . . Humanity," rather than by
an independent clause.

nR

Explanation for Incorrect Answer E :


Choice (E) creates improper modification. The adverb phrase "to Habitat for
Humanity" modifies "Laurie" rather than the verb "was volunteering."

U
7

Reviews of books and films have generally been exempt from the standards of libel
that are applied to news stories, but they are now questioning the distinction between
reporting and criticism.
(A)

they are now questioning the distinction between reporting and criticism

(B)

questions are now being asked by them as to the distinction between


reporting and criticism

(C) the distinction between reporting and criticism is now being questioned

(D) the question they are asking now is whether there is a distinction between
reporting and criticism

(E)

the distinction they make between reporting and criticism is now


questioned

ANSWERS

AND EXPLANATIONS

Explanation for Correct Answer C :


Choice (C) is correct. It avoids the error of the original by providing a passive verb
phrase ("is . . . being questioned") and by removing the vague pronoun "they."

Explanation for Incorrect Answer A :

file://E:\\b10.htm

2006-11-12

5/9

The Official SAT Online Course

Choice (A) involves the use of a vague pronoun. There is nothing in the sentence to
which the pronoun "they" can logically refer.

Explanation for Incorrect Answer B :


Choice (B) includes the use of a vague pronoun. The pronoun "they" cannot logically
refer to any of the plural nouns in the sentence.

Explanation for Incorrect Answer D :


Choice (D) results in an awkward sentence, combining unnecessary emphasis ("the
question they are asking now") with the use of a vague pronoun, "they," which
cannot logically refer to anything in the sentence.

Explanation for Incorrect Answer E :


Choice (E) creates a confusing sentence. The pronoun "they" apparently refers to
"reviews," but it is odd to say that reviews, which are criticism, could make a
distinction between reporting and criticism.

A writer who well understood the plight of the underprivileged, many acclaim Richard
Wright as the novelist of the downtrodden.

(A)

many acclaim Richard Wright as

(B)

many have acclaimed Richard Wright as

(C) Richard Wright is being acclaimed by many as

(E)

re
d

(D) Richard Wright has been widely acclaimed as

widely acclaimed is Richard Wright as

AND EXPLANATIONS

is
te

ANSWERS

Explanation for Correct Answer D :


Choice (D) is correct. It avoids the error of the original by following the introductory
adjective clause (A writer who . . . underprivileged ) with Richard Wright,
the noun it modifies.

eg

Explanation for Incorrect Answer A :


Choice (A) results in improper modification. The introductory noun clause A
writer . . . underprivileged improperly modifies many when it should modify
Richard Wright.

nR

Explanation for Incorrect Answer B :


Choice (B) creates improper modification. The noun clause A writer . . .
underprivileged improperly modifies the pronoun many rather than the
proper noun Richard Wright.

Explanation for Incorrect Answer C :


Choice (C) results in an improper sequence of verb tenses. The verb
understood (past tense) is improperly followed by the verb is being
acclaimed (present progressive tense).

Explanation for Incorrect Answer E :


Choice (E) involves improper modification. The introductory adjective clause (A
writer . . . underprivileged) is improperly followed by an adjective phrase,
widely acclaimed, rather than by Richard Wright, the noun it modifies.

Having no additional funds to spend, the meeting of the budget committee was
promptly concluded.
(A)

the meeting of the budget committee was promptly concluded

(B)

the meeting of the budget committee concluded promptly

(C) the conclusion of the budget committee meeting was prompt

(D) the budget committee promptly concluded its meeting

(E)

file://E:\\b10.htm

the budget committees meeting was promptly concluded

2006-11-12

6/9

The Official SAT Online Course

ANSWERS

AND EXPLANATIONS

Explanation for Correct Answer D :


Choice (D) is correct. It avoids the error of the original by providing an appropriate
subject, "the budget committee," for the modifying phrase "Having . . . spend."

Explanation for Incorrect Answer A :


Choice (A) involves improper modification. It improperly makes "the meeting" the
subject of the modifying phrase "Having . . . spend."

Explanation for Incorrect Answer B :


Choice (B) results in improper modification. The phrase "Having no additional funds
to spend" cannot logically modify "the meeting of the budget committee."

Explanation for Incorrect Answer C :


Choice (C) is unsatisfactory because it involves improper modification. The phrase
"Having . . . spend" cannot logically modify "the conclusion of the budget
committee meeting."

Explanation for Incorrect Answer E :


Choice (E) results in an illogical sentence because it involves improper modification.
It illogically makes "the budget committee's meeting" the subject of the modifying
phrase "Having . . . spend."

te
re
d

In the early songs of the Beatles, one hears plaintive Blues-inspired melodies that
would seem to be more a product of rural southern America than an English industrial
city.

(A)

a product of rural southern America than an English industrial city

(B)

a product from rural southern America than that of an English industrial


city

is

10

(C) produced in rural southern America than by an English industrial city

nR
eg

(D) a product out of rural southern America than from an English industrial
city

(E)

a product of rural southern America than of an English industrial city

ANSWERS

AND EXPLANATIONS

Explanation for Correct Answer E :


Choice (E) is correct. It avoids the error of the original by providing a phrase, "of an
English industrial city," that is grammatically parallel to the phrase "of rural
southern America," thus making clear that two locations are being compared.

Explanation for Incorrect Answer A :


Choice (A) involves an illogical comparison. A product is being illogically compared
to a city.

Explanation for Incorrect Answer B :


Choice (B) is wordy; the pronoun "that" is not necessary. Also, there is an improper
idiom: the phrase "a product from . . . America" is used where it would be more
idiomatic to say "a product of . . . America."

Explanation for Incorrect Answer C :


Choice (C) results in the use of an improper idiom. The phrase "more produced in .
. . America" is used where it would be more idiomatic to say "more a product of . . .
America."

Explanation for Incorrect Answer D :


Choice (D) involves the use of an improper idiom. The phrase "a product out of . . .
America" is used where it would be more idiomatic to say "a product of . . .
America."

file://E:\\b10.htm

2006-11-12

7/9

The Official SAT Online Course

In 1968 air pollution from automobile exhaust, particularly like that in the Los
Angeles environs, became of an increasingly more urgent concern to
environmentalists.

11

(A)

like that in the Los Angeles environs, became of an increasingly more


urgent

(B)

of the sort found in the Los Angeles area, became an increasingly urgent

(C) such as you have in the area of Los Angeles, became of increasingly more
urgent

(D) like the kind in and around Los Angeles and its environs, became an
increasingly urgent

(E)

the kind similar to the Los Angeles area, became an increasingly more
urgent

ANSWERS

AND EXPLANATIONS

Explanation for Correct Answer B :


Choice (B) is correct. It avoids the error of the original by changing the overly fancy
word "environs" to the simpler "area" and by changing the phrase "became of an
increasingly more urgent concern" to the more idiomatic "became an increasingly
urgent concern."

re

Explanation for Incorrect Answer A :


Choice (A) results in an awkward and wordy sentence. The word "environs" is
unnecessarily fancy; also, the phrase "became of an increasingly more urgent
concern" is used where it would be more idiomatic to say "became an increasingly
urgent concern."

is
te

Explanation for Incorrect Answer C :


Choice (C) involves an illogical sequence of tenses. The present tense of the verb in
the phrase "such as you have" does not make sense in a sentence describing
something that happened in 1968.

Explanation for Incorrect Answer D :


Choice (D) involves wordiness. The word "environs" means the area around
something, so it is repetitive to use both it and the phrase "in and around."

eg

nR

Explanation for Incorrect Answer E :


Choice (E) involves an illogical comparison. Air pollution is illogically being
compared to a geographical area.

U
12

Although long considered a vestigial organ that has no function in humans, the
appendix, some scientists believe, may have a significant role as part of the bodys
immune system.
(A)

the appendix, some scientists believe, may have

(B)

the appendix, which, some scientists believe, may have

(C) the appendix, in the belief of some scientists, is possibly having

(D) the appendix is believed by some scientists as perhaps having

(E)

some scientists believe the appendix may have

ANSWERS

AND EXPLANATIONS

Explanation for Correct Answer A :


Choice (A) is correct. It provides an appropriate subject, "the appendix," for the
modifying phrase, "Although long considered . . . in humans."

Explanation for Incorrect Answer B :


Choice (B) results in a sentence fragment. The sentence is grammatically
incomplete because it has no main verb.

file://E:\\b10.htm

2006-11-12

8/9

The Official SAT Online Course

Explanation for Incorrect Answer C :


Choice (C) is unsatisfactory because it involves the use of improper idioms. The
phrase "in the belief of" is used where it would be more idiomatic to say "in the view
of," and the verb phrase "is possibly having" is used where it would be more
idiomatic to say "may have."

Explanation for Incorrect Answer D :


Choice (D) involves the use of an improper idiom. It provides the phrase "is
believed . . . as perhaps having" where it would be more idiomatic to use a phrase
such as "is believed . . . to have."

Explanation for Incorrect Answer E :


Choice (E) involves improper modification. It improperly makes "some scientists"
the subject of the modifying phrase "Although long considered . . . in humans."

Failing to anticipate the sharp downturn in the economy, millions of dollars were lost
through the investment funds managers slowness to act.

13

(A)

millions of dollars were lost through the investment funds managers


slowness to act

(B)

millions of dollars were lost due to the slowness of the investment funds
managers to act

(C) the investment funds managers were slow to act, millions of dollars
were lost as a result

the investment fund lost millions of dollars because its managers acted too
slowly

ANSWERS

te
r

(E)

ed

(D) the investment funds managers were slow to act and thus lost millions
of dollars

AND EXPLANATIONS

Explanation for Correct Answer D :


Choice (D) is correct. It avoids the error of the original by providing an appropriate
subject, "managers," for the modifying phrase "Failing to . . . the economy."

eg
is

Explanation for Incorrect Answer A :


Choice (A) results in improper modification. It improperly makes the noun phrase
"millions of dollars" the subject of the modifying phrase "Failing to anticipate . . .
the economy."

nR

Explanation for Incorrect Answer B :


Choice (B) involves improper modification. The noun phrase "millions of dollars"
cannot logically be modified by the phrase "Failing to anticipate . . . the economy."

Explanation for Incorrect Answer C :


Choice (C) results in improper coordination. The clause beginning with the word
"millions" is linked to the previous clause by only a comma.

Explanation for Incorrect Answer E :


Choice (E) results in an illogical sentence because it involves improper modification.
It illogically makes the noun phrase "the investment fund" the subject of the
modifying phrase "Failing to anticipate . . . the economy."

14

Most often defenders of art have justified its existence with their pointing out a
function that nothing but art itself could perform.
(A)

with their pointing out a function that nothing but art itself could perform

(B)

by them pointing out a function that only art could perform

(C) through the pointing out of a function performed by art alone

(D) by pointing to some function that art alone can perform

(E)

through their having pointed to a function performed by art itself

ANSWERS

file://E:\\b10.htm

AND EXPLANATIONS

2006-11-12

9/9

The Official SAT Online Course

Explanation for Correct Answer D :


Choice (D) is correct. It avoids the error of the original by using the proper
preposition, "by," and by eliminating the unnecessary pronoun "their."

Explanation for Incorrect Answer A :


Choice (A) results in an improper idiom. The phrase "with their pointing out" uses
the preposition "with" and the pronoun "their" where the preposition "by" would be
idiomatic.

Explanation for Incorrect Answer B :


Choice (B) involves an improper idiom. The phrase "by them pointing" uses the
pronoun "them" where no pronoun is required.

Explanation for Incorrect Answer C :


Choice (C) involves an improper idiom. The phrase "through the pointing out" uses
the preposition "through" where "by" would be idiomatic.

Explanation for Incorrect Answer E :


Choice (E) creates an improper idiom. The sentence uses the preposition "through"
and the phrase "their having pointed" where the preposition "by" and the noun
"pointing" would be idiomatic.

re
d

Privacy Policy

te

Copyright 2006 The College Board. All rights reserved.

Back to Score Report

Terms of Use

Contact Us

is

eg

nR

file://E:\\b10.htm

2006-11-12

Potrebbero piacerti anche